Health Assessment

Ace your homework & exams now with Quizwiz!

A client is diagnosed with a scotoma. What question is appropriate for the nurse to ask to obtain more data about this condition?

"Are the blind spots constant or intermittent?" A scotoma is the presence of blind spots that can be constant or intermittent. If they are constant it may indicate retinal detachment. Intermittent blind spots may be due to vascular spasm or pressure on the optic nerve. Floaters are a common finding in individuals with myopia or in person over the age of 40 years and are a sign of normal aging. Redness or tearing is associated with allergies or inflammation of the eye. Night blindness is associated with optic nerve atrophy, glaucoma, or vitamin A deficiency.

A 61-year-old man has visited the clinic at the prompting of his wife, who states that his hearing is becoming less acute. Which of the following assessment questions is most useful in determining whether the man's hearing loss is sensorineural or conductive?

"Do you find that you have particular difficulty understanding people's speech?"

What is an appropriate question by the nurse to ask a client about the presence of temporomandibular joint dysfunction?

"Have you noticed a popping or grating sound when you chew?"

A student nurse is auscultating for bowel sounds on a client who returned from surgery 48 hours ago. The student tells the charge nurse that she cannot hear bowel sounds in the lower quadrants. What is the appropriate response by the charge nurse to this information?

"It takes about 3 to 5 days after surgery for the bowel sounds to return completely"

A nurse tells a client that the next step in the musculoskeletal assessment is to perform range of motion of the thoracic and lumbar spine. The nurse should demonstrate which movements for the client to facilitate the examination? Select all that apply.

*Flexion* Extension *Lateral bending Circumduction *Rotation* Exp: To assess the range of motion of the thoracic and lumbar spine, the client should be shown the muscle movements of flexion, lateral bending (right and left), rotation (twisting the shoulders one way then the other), and bending backwards (hyperextension). Circumduction is moving is a circular motion. P546

A school age client has been diagnosed with genu valgum. What is the other name for this disease?

*Knock kneed* P554 Flatfeet Bowlegs (genu varum) Clubfoot

The nurse is testing a client for carpal tunnel syndrome. The client flexes the wrists at an angle of 90° and holds the backs of the hands to each other for 60 seconds. The client tells the nurse that he is experiencing a burning pain as a result. Which test is the nurse performing on this patient?

*Phalen's* (for CT) McMurray's Tinel's (also for CT) Flick signal (also for CT) Ballottement

The nurse is assessing a client with joint pain and is trying to decide whether it is inflammatory or non-inflammatory. Which of the following symptoms is consistent with an inflammatory process?

*Tenderness* Ecchymosis Cool temperature Nodules Explanation: Tenderness implies an inflammatory process along with increased temperature. Nodules and ecchymosis are not typically associated with inflammatory processes. P542

The nurse instructs the patient to raise his arm out to the side and overhead. The nurse is asking the patient to adduct his arm.

*false* P528

The nurse is assessing the spine of an adult client and detects lateral curvature of the thoracic spine with an increase in convexity on the left curved side. The nurse suspects that the client is experiencing

*scoliosis* P563

A nurse examines a client with complaints of a sore throat and finds that the tonsils are enlarged and touching the uvula. Using a grading scale of 1+ to 4+, how should the nurse appropriately document the tonsils?

+3

During palpation of the client's knee, the nurse compresses the suprapatellar pouch against the client's femur with one hand while feeling on each side of the patella with the opposite hand. For which of the following problems is the nurse assessing?

-*Effusion in the knee joint* -Crepitus uteri flexion -Osteoarthritis -Ligament trauma Explanation: The balloon sign is indicative of a large effusion in the knee joint when fluid is palpable medial to the patella when the suprapatellar pouch is depressed. The presence of crepitus, osteoarthritis, or ligament damage is not directly suggested by a positive balloon sign. P556

A patient presents at the clinic with an enlarged, swollen, hot, and red metatarsophalangeal joint and bursa of the great toe. What medical diagnosis would the nurse suspect?

-*Gouty arthritis* P565, 557 -Hallux valgus -Hammer toe -Pes planus

An older adult client presents at the clinic, reporting otalgia in the right ear. Physical assessment reveals cerumen impacted in the client's ear. Removing this mechanical blockage may do what for this client? (Select all that apply.)

-Enhance socialization -Improve hearing -Prevent injury

what number correlates with non-pitting edema?

0+

Palpation: 5 points on the posterior thorax?

1. Assess for tenderness and sensation 2. Palpate for crepitus 3. Palpate for surface characteristics 4.Palpate for fremitus 5. palpate for symmetric expansion

In what order should you take these steps for physical examination of the face? 1. assess CNV sensory using a cotton wisp to test the ophthalmic, maxillary, and mandibular branches of the nerve (bilaterally touch forehead, cheeks, chin) 2. Inspect face for symmetry, expression, involuntary movements, or swelling by testing CNVII (ask patient to smile, frown, lift eyebrows, etc.) 3. palpate temporal and masseter muscles to test mastication as client clenches teeth, feel temporal as they clench, ask them to keep clenching then try to pull down chin

2, 1, 3

How many lobes does the right lung have?

3 lobes RUL RML RLL

The nurse examines the pharynx of a patient and records that the tonsils are touching the uvula. The nurse would grade the tonsils as

3+

An individual is considered obese when his or her BMI is:

30-39

Edema with very deep pitting (8mm), indentation that lasts a long time, and a leg that looks very swollen is noted as:

4+

Mole assessment:

A = asymmetry B = irregular borders C = change in color D = diameter >/= 6mm E = evolving

Myopathy:

A disease of muscle tissue

ABCDEs of melanoma

A: asymmetry B: border irregularity C: color D: diameter of more than 6 mm E: Evolution of lesion over time

Ataxia:

Abnormal motions

affects the lower lip and is characterized by scales, thickening, and eversion of the lip tissue.

Actinic cheilitis

A client reports experiencing chronic headache after a recent upper respiratory tract infection. On physical examination, the nurse notes tenderness when palpating over the sinuses. Which condition is likely?

Acute bacterial sinusitis.

A client diagnosed with goiter has undergone a thyroidectomy. Which statement from the client indicates understanding of post-operative care teaching?

After thyroidectomy, clients must be treated with exogenous thyroid hormone for the rest of their lives. Thyroid hormones are usually taken by mouth on a daily basis.

A nurse notes that a client looks much older than his chronologic age. Which of the following conditions would most likely contribute to this appearance?

Alcoholism A client may appear older than actual chronologic age due to a hard life, manual labor, chronic illness, alcoholism, or smoking. Parkinsons disease is associated with stiff, rigid movements. Marfan;s syndrome is associated with arm span being greater than height and pubis to sole measurement exceeding pubis to crown measurement. Cushing;s syndrome is associated with central body weight gain with excessive cervical obesity (Buffalos hump).

12. The nurse is planning to inspect an adult client's mouth, using a tongue depressor. The nurse should plan to

Ans: Depress the tongue blade slightly off center

The nurse would assess for positive Blumberg sign how?

Applying and releasing pressure to the abdomen

A 82 year old female presents with neck pain, decreased strength and sensation of the upper extremities. The nurse identifies that this could be related to what?

Arthritic changes of the cervical spine

A nurse is preparing to examine a client from Southeast Asia who has been experiencing chronic headaches. Which of the following should the nurse do in light of this client's cultural background?

Ask permission before palpating the head and neck

A 66-year-old woman has come to the clinic with complaints of increasing fatigue over the last several months. She claims to frequently feel lethargic and listless and states that, "I can never seem to get warm, no matter what the thermostat is set at." How should the nurse proceed with assessment?

Assess the woman for hypothyroidism.

Nursing process:

Assessment, diagnosis, planning, implementation, evaluation

During the physical examination of a client, a nurse notes that a client's trachea has been pushed toward the right side. The nurse recognizes that the pathophysiological cause for this finding is related to what disease process?

Atelectasis

A nurse is teaching a client who suffers from peptic ulcers how to reduce the risk of their recurrence. Which of the following should the nurse recommend?

Avoid excessive alcohol intake The nurse should recommend avoiding excessive alcohol intake, as this is a risk factor associated with peptic ulcer disease. The nurse should also recommend eating foods that have been cooked completely and taking pain medications with food. Antacid medications may relieve peptic ulcers.

Which of the following arteries can be palpated below the inguinal ligament between the anterior superior iliac spine and the symphysis pubis? A. Dorsalis pedis artery B. Femoral artery C. Ulnar artery D. Popliteal artery

B. Femoral artery Rationale: The femoral artery may be felt in the given location, while the popliteal and dorsalis pedis arteries are both distal to this point. The ulnar artery is located in the arm.

The nurse feels a small mass in the neck of a client. It is mobile in both the up-and-down and side-to-side directions. Which of the following is the nurse most likely feeling? A. Deep scar B. Lymph node C. Muscle D. Cancer

B. Lymph node Rationale: A useful way to discern lymph nodes from other masses in the neck is to check for their mobility in all directions. Many other masses are mobile in only two directions. Cancerous masses may also be fixed or immobile.

19-23 not at risk 15-18 low risk 13-14 moderate risk 10-12 high risk Less than or equal to 9 very high risk****

Braden Scale for Predicting Pressure Sore Risk

A new nurse on the long-term care unit is learning how to assess a patient's risk for skin breakdown. What would be the most likely instrument this nurse would use?

Braden scale Identifying risk for skin breakdown is especially important in hospitalized or inactive patients. Many health care facilities use the Braden Scale to assess risk in patients, with interventions based on the total score.

auscultation Have them say 99 as you listen over stethoscope. If you hear it clearly = bad

Broncophony

The nurse practitioner notes that the thyroid gland is enlarged and auscultates both lobes of the thyroid. For what is the nurse practitioner listening?

Bruit

After teaching a group of students about the important organs to be assessed during an abdominal assessment, the instructor determines that the teaching was successful when the students identify which organ as the largest solid organ in the body? A. Kidney B. Spleen C. Liver D. Pancreas

C. Liver Rationale: The liver is the largest solid organ in the body.

What tool is used for skinfold tests?

Calipers

The nurse enters a client's hospital room and the client asks the nurse to raise him up in the bed. What is the nurse's best action?

Call for help and use the draw sheet to move the client.

An adult client comes to the ED with a new onset of pain in his neck and jaw. What system requires emergency assessment? Cardiovascular Integumentary Respiratory Nervous

Cardiovascular

The nurse assess for kidney tenderness at what location?

Costovertebral angle

Hirsutism, or facial hair on females, is a characteristic of?

Cushing's disease and results from an imbalance of adrenal hormones.

An adult client is admitted to the hospital with severe diarrhea. When assessing the client, the nurse notes a round "moon" face, a buffalo hump at the nape of the neck, and a velvety discoloration around the neck. What are these signs indicative of? A. Myxedema B. Cushing's syndrome C. Scleroderma D. Bell's palsy

Cushing's syndrome

Assessment of an adult female client's face reveals a moon shape, increased hair distribution, and a reddened tone to the client's cheeks. What collaborative problem is most clearly suggested to the nurse by these assessment data?

Cushing's syndrome

What if you spot dark bluish-pink striae?

Cushing's syndrome

A nurse is assessing a child who got lost on a camping trip in November and was exposed all night to the elements. Which finding about the lips would support a diagnosis of hypoxia in this client?

Cyanotic lips are seen in cases of cold or hypoxia.

When assessing the skin of a patient, the nurse notices lesions that run together. What type of lesion is this? A- zosteriform B- grouped C- annular D- confluent

D (confluent)

How would you document a pressure ulcer that has full thickness, involves all skin layers, and extends into supporting tissues? This pressure ulcer is also exposing the patient's bone. A- stage I B- stage II C- stage III D- stage IV E- unstageable

D (stage IV)

The nurse is assessing a client's pain. Which of the following would lead the nurse to suspect that the client is experiencing pain? A. Regular, unlabored breathing B. Alert, talkative demeanor C. Sitting upright, hands on lap D. Facial grimacing, leaning forward

D. Facial grimacing, leaning forward Rationale: Facial expressions such as grimacing, and body positions such as leaning forward, suggest pain. Regular unlabored breathing, sitting upright with hands on the lap and an alert talkative demeanor suggest that he client is comfortable and relaxed.

All of the following skin lesions are papular except: A. warts B. acne C. moles D. herpes zoster

D. herpes zoster

A patient recovering from a burn injury is told by the health care provider that hair will no longer grow on the body part that was burned. When the patient questions why this is true, the nurse will base the response on what physiological event that occurred as a result of the burn?

Destruction of hair follicles located in the dermis layer Damage to hair follicles located in the dermis layer of the skin would result in the body's inability to regrow hair on burn damaged areas. The remaining options suggest correct information but none are associated with the regrowth of hair after a burn.

A nurse examines a client's retina during the ophthalmic examination and notices light-colored spots on the retinal background. The nurse should ask the client about a history of what disease process?

Diabetes Exudates appear as light-colored spots on the retinal background and occur in individuals with diabetes or hypertension. Anemia, renal insufficiency, and retinal detachment do not cause this appearance on the retina.

A nurse cares for a client of Asian descent and notices that the client sweats very little and produces no body odor. What is an appropriate action by the nurse in regards to this finding?

Document the findings in the client's record as normal Asians and Native Americans have fewer sweat glands than Caucasians and therefore produce less sweat and less body odor. Changes in sensation are not caused by alterations in sweat glands but are a circulation issue. Cystic fibrosis is an alteration in the exocrine glands that causes the production of thick mucus, especially in the lungs. Use of antiperspirants would be needed for excessive sweating, not a lack of sweating.

A nurse has performed a head and neck assessment of an adult patient and noted that the thyroid gland is not palpable. What is the nurse's most appropriate action?

Document this as an expected assessment finding

· ask the client to repeat letter "E" Normal- soft distinguishable "E" Abnormal- sounds like "A"

Egophony

Chelitis:

Erythema, scaly, shallow and painful fissures at the corners of the mouth occur with excess salivation and Candida infection. Often from poorly fitting dentures, causing folding in corners of mouth, which creates a warm, moist environment favoring growth of yeast

Infants and young children

Essential amino acids from protein for growth. Whole milk is recommended for children 2 and under.

Specialized examination occur how often when in the hospital?

Every 8 hours

When performing the cover test, a nurse notices that the client's left eye turns outward. How should the nurse document this finding in the client's record?

Exotropia With the cover test, the eyes of the client should remain fixed straight ahead. If the covered eye moves when uncovered to reestablish focus, it is abnormal. If the eye turns outward it is called exotropia. If the uncovered eye turns inward, it is called esotropia. Strabismus is constant malalignment of the eyes. Presbyopia is impaired near vision.

Delusions:

Firm, fixed, false beliefs; irrational; person clings to delusion despite objective evidence to contrary

sitting with backrest elevated 90 degrees Legs extended flat on table Exam of lower extremities/feet

Fowlers position

Gingivitis is an inflammation of the gums that often results in bleeding. Think bleeding!

Gingivitis

On palpation, the nurse notes that a client's thyroid gland is diffusely enlarged. Which of the following health problems would the nurse want to rule out? A. A tumour B. Hypothyroidism C. Graves' disease D. Nephrotic syndrome

Graves' disease

A six-month old male infant is brought to the emergency department by his parents for inconsolable crying and pulling at his right ear. When assessing this infant the nurse is aware that the tympanic membrane should be what color in a healthy ear?

Gray

Tension headaches:

Headache of musculoskeletal origin; may be a mild-to-moderate, less disabling form of migraine

Cluster headache:

Headache that is intermittent, excruciating, unilateral with autonomic signs

When assessing the head and neck, the nurse should realize that variations in skull or neck shape or size relate most to what?

Height and weight

What term would the nurse use to document a client's report of pain in the lower-middle area of the abdomen?

Hipogastic

Corneal Light Reflex:

Hold pen light ~ 2 ft. in front of patient and shine pen light into both eyes at the same time; want to make sure the light reflection is symmetrical in both eyes

The nurse notes that a client's capillary refill is 5 seconds. What should this finding indicate to the nurse?

Hypoxia

Hyperthyroidism/Graves Disease:

Increased production of thyroid hormone causes an increase metabolic rate. Manifest a goiter and exophthalmos. S/S include: nervousness, fatigue, weight loss, and heat intolerance/forceful tachycardia, SOB, excessive sweating, fine muscle tremor, thin and silky hair, and a staring appearance

XII (12): Hypoglossal

Inspect tongue (midline), voice should be clear and distinct (light, tight, dynamite)

Using the tongue depressor to keep the mouth open wide, inspect the tonsils for color, size, and presence of exudate or lesions. Grade the tonsils.

Inspect tonsils

Cody is a teenager with a history of leukemia and an enlarged spleen. Today he presents with fairly significant left upper quadrant pain. On examination of this area a rough grating noise is heard. What is this sound?

It is a splenic rub. A rough, grating noise over this area represents a splenic rub, which can accompany splenic infarction. Rubs also occur over the liver and pleura and pericardium.

Which of the following assessment findings most likely constitutes a secondary skin lesion?

Keloid formation at the site of an old incision A secondary lesion emerges from an existing primary lesion, such as the keloids that can emerge from the site of a healed wound. Acne and the lesions associated with psoriasis and herpes do not meet this criterion.

Cafe au Lait spot:

Large freckles; birth mark

A nurse auscultates for bowel sounds on a client admitted for nausea and vomiting and hears no gurgling in the right lower quadrant after 1 minute. What is an appropriate action by the nurse?

Listen for a total of 5 minutes

In reviewing a client's health history, the nurse notes that the client has had a history of TMJ pain. The nurse recognizes that which of the following bones is involved in this dysfunction?

Mandible

What activity is known to aggravate a tension headache?

Mandible

what element of COLDSPA? asks "when did it start?"

Onset (COLDSPA)

separation of the nail plate from the nail bed, fungal nails

Onycholysis

Carotemia:

Orange-pigmented color; too many vitamins in their system; won't cause harm to children or adults

Which of the following describes a condition characterized by abnormal spongy bone formation around the stapes?

Otosclerosis

The nurse assesses a client with lower abdominal pain who reports localized tenderness in the right lower quadrant with right flank pain. Which assessment should the nurse conduct next?

Palpate the right lower quadrant for rebound tenderness.

raised less than 1 cm

Papule (primary)

The nurse needs to assess the abdomen of a hospitalized client post gastrointestinal surgery. Place the following assessment steps in order as the nurse enters the client's room.

Perform a general survey of safety hazards. Inspect the abdomen. Auscultate all four quadrants. Palpate for tenderness. Document the findings.

Functions of the skin:

Protection, barrier, sensory perception (touch, pain, pressure, temp.), regulates body temp, ID, synthesize vitamin D, nonverbal communication, wound repair, excretion of metabolic waste (salt)

what test is being used when assesing pain in the RLQ when the right leg is hyperextended ( indicating irritation of the iliopsoas muscle due to appendicitis)

Psoas sign

Characterized by reddish-pink lesions covered with silvery scales. It commonly occurs on extensor surfaces such as the elbows and knees but can appear anywhere on the body.

Psoriasis

What action should the nurse implement using an otoscope when assessing the ear of an adult client?

Pull the auricle out, up and back

When testing the near reaction, an expected finding includes which of the following?

Pupillary constriction on near gaze; dilation on distant gaze During accommodation, pupils constrict with near gaze and dilate with far gaze.

The client would complain of pain in what quadrant if experiencing appendicitis?

RLQ With appendicitis, the client would experience pain in the RLQ.

To assess an adult client for possible appendicitis and a positive posts sign, the nurse should:

Raise the clients right leg from the hip

Wheals:

Raised area of edema in the upper epidermis; Ex. hives

Bullae:

Raised lesions larger than 1 cm in diameter and filled w/ fluid

The nurse suspects that a client has Cushing's syndrome. What assessment finding did the nurse use to make this clinical determination?

Red cheeks

Which characteristic feature of the tympanic membrane should a nurse anticipate finding in a client with otitis media?

Red, bulging with an absent light reflex

The nurse notes otitis media with effusion in the left ear of a 3-year-old child. Which assessment data is consistent with otitis media with effusion?

Redness and bulging of the eardrum

Crust:

Results from pus/exudate on the skin that dries and hardens which dries out the surrounding skin

Acute otitis media:

Results when middle ear fluid is infected; absent light reflex from increasing middle ear pressure is early sign

As part of an abdominal assessment, the nurse must palpate a client's liver. In which quadrant is this organ located?

Right Upper Quadrant (RUQ)

Tinea Capititis:

Ringworm can occur in the scalp

The nurse expects what change in a client's hair as a result of aging?

Sebaceous glands will secrete less causing hair to be drier.

A 58-year-old gardener comes to your office for evaluation of a new lesion on her upper chest. The lesion appears to be "stuck on" and is oval, brown, and slightly elevated with a flat surface. It has a rough, wartlike texture on palpation. Based on this description, what is your most likely diagnosis?

Seborrheic keratosis

Which organ that resides in the abdominal cavity stores red blood cells and platelets, produces new red blood cells and macrophages, and activates B and T lymphocytes?

Spleen

The nurse percusses the lowest interface in the left anterior axillary line, asks the client to take a deep breath, and percusses again. The nurse is assessing for which of the following?

Splenic percussion sign Explanation: A change in the percussion note from tympany to dullness on inspiration in this location suggests splenic enlargement. The given procedure is the correct technique for assessing for a positive splenic percussion sign, not kidney tenderness, liver palpation, or diaphragmatic displacement.

When asked to assess an area of broken skin on an older adult client in a long-term care facility, the nurse notes a break in the skin erythema and a small amount of serosanguineous drainage over the sacrum. The area appears blister-like. The nurse would interpret this finding as indicating which stage of pressure ulcer?

Stage II

Whisper test:

Stand behind patient; test one ear at a time; occlude opposite ear; whisper random letter/number series and have patient repeat back

A patient is diagnosed with otosclerosis, a condition in which the auditory ossicles develop a spongy consistency, which results in conductive hearing loss. It appears that the worst site is the inner most bone, which transmits sound waves through the oval window. Which bone is this?

Stapes

· Place hands at posterior chest wall with thumbs at T9 or T10 · Thumbs should move symmetrically 5-10 cm per respiratory cycle

Symmetric expansion

A 29-year-old computer programmer comes to the office for evaluation of a headache. The tightening sensation of moderate intensity is located all over the head. It used to last minutes, but this time it has lasted for 5 days. He denies photophobia and nausea. He spends several hours at a computer monitor/keyboard. He has tried over-the-counter medication; it has dulled the pain, but not taken it away. Based on this description, what is the most likely diagnosis? A. Tension B. Migraine C. Cluster D. Analgesic rebound

Tension

A client visits the clinic and tells the nurse that he is depressed because of a recent job loss. He complains of dull, aching, tight, and diffuse headaches that have lasted for several days. The nurse should recognize that these are symptoms of

Tension headaches

An adult client is having his skin assessed. The client tells the nurse he has been a heavy smoker for the last 40 years. The client has clubbing of the fingernails. What does this finding tell the nurse?

The client has chronic hypoxia Clubbing of the nails indicates chronic hypoxia. Clubbing is identified when the angle of the nail to the finger is more than 160 degrees. Melanoma does not present with the symptom of clubbing. The scenario described does not give enough information to indicate that the client has COPD or asthma.

What structure in the inner ear senses the position and movements of the head and helps to maintain balance?

The labyrinth

spleen function?

The spleen functions primarily to filter the blood of cellular debris, to digest microorganisms, and to return the breakdown products to the liver. (Weber 475)

A nurse is providing care for a client who has decreased mobility secondary to a recent stroke. Which assessment finding would be indicative of a stage I pressure ulcer?

There is a nonblanching reddened area on the client's coccyx region.

Palpation of a 15-year-old boy's submandibular lymph nodes reveals them to be enlarged and tender. What is the nurse's most reasonable interpretation of this assessment finding?

There is an infection in the area that these nodes drain.

Lichen:

Thickening of the epidermis; usually caused by chronic rubbing/scratching

During a pharmacology class the students are told that some drugs need to be closely monitored. What aspect should the nurse closely monitor for in clients who have been administered salicylates, loop diuretics, quinidine, quinine, or aminoglycosides?

Tinnitus and sensorineural hearing loss

How do you inspect the thyroid?

Tip head back slightly and swallow w/ sip of water

Midline bony growth in the hard palate that is fairly common in adults. Its size and lobulation vary.

Torus palatinus

Mrs. Anderson presents with an itchy raised rash that appears and disappears in various locations. Each lesion lasts for many minutes. Which most likely accounts for this rash?

Urticaria or hives This is a typical case of urticaria. The most unusual aspect of this condition is that the lesions move from place to place. This would be distinctly unusual for the other causes listed.

Neck serves as a conduit (a channel) for structures such as:

Vessels, muscles (sternomastoid/trapezius), nerves, lymphatics, viscera of the respiratory and GI system

What is Murphy's sign?

When you curl a finger under the rib cage right by the liver and then ask the patient to take a deep breath. If there is pain then that is a gallbladder or liver problem.

What light should the nurse use to inspect a lesion on the thigh of a client for the presence of fungus?

Wood's light

Jaundice:

Yellow-pigmented color; yellowing in the sclera/skin

What does CAGE screen for?

a screening tool for alcoholism

what does a pulse strength of 0 mean?

absent

Stasis dermatitis

affects lower legs and ankles, reddish brown build up of fluid under the skin causes circulation problems = fragile skin and poorly nourished causes ulcers and wounds

Presbyopia

age related farsightedness

what can cause?: Lungs lose their elasticity Skeletal muscles begin to weaken Bones lose their density Appearance of barreled chest and calcification of cartilage occur Alveoli become fibrotic

aging

Eustachian tube

anatomical feature that equalizes air pressure in the middle ear

Define: What kind of shape/configuration of lesions is characterized by circular lesions that begin in the center and spread to periphery (EX. tinea corporis (ringworm), tinea versicolor, pityriasis rosea)

annular

Wound inflammatory phase

begins 30 min after injury, lasts 2-3 days

While assessing the skin of an older adult client, the nurse observes that the client has small yellowish brown patches on her hands. The nurse should instruct the client that these spots are

caused by aging of the skin in older adults.

logical/illogical, coherent/incoherent, relevant

clarity

what is a result of malabsorption

clay color or fatty stools

When checking that the nail surface is smooth and regular you are checking for __________

consistency

The nurse is performing an assessment on a client that is on postop day 2. The abdominal wound has pulled apart and the contents are spilling out. The nurse recognizes this as a what? Hernia Dehiscence Infection Abscess

dehiscence

difficulty in swallowing

dysphagia

How would you describe an irregular heart rhythm?

dysrhythmia

high pitched popping sounds on inspiration, not cleared after coughing, Late in inspiration: restrictive diseases- pneumonia, CHF Early in inspiration: obstructive diseases- bronchitis, asthma, emphysema

fine sounds

An adult male visits the clinic and tells the nurse that he believes he has athlete's foot. The nurse observes that the client has linear cracks in the skin on both feet. The nurse should document the presence of

fissures

what kind of test is done to test for ascites

fluid wave test

Which of the following people need to be vaccinated for hepatitis A and B? Truck drivers Animal care workers Food-service workers Office personnel

food service

waist circumference

greater than 40 in males, greater than 35 in females increases risk for chronic illness

what is larger than 12 cm, enlarged liver due to cirrhosis, tumors and abssess

hepatomegaly

A client who is bedfast responds only to painful stimuli, never eats a complete meal, and moves occasionally in bed. Which term should the nurse use to describe this client's risk for skin breakdown?

high his client is at a high risk for skin breakdown because of activity (bedfast), poor nutritional status (never eats a complete meal), and immobility (occasionally moves in bed). A person who is independent with mobility and has a good nutritional status would have a mild or negligible risk for skin breakdown. A client who spends sometime in the same position and consumes half of required nutrients would have a moderate risk for skin breakdown.

The nurse is assessing a client's abdomen. For which reason should the nurse perform deep palpation?

identify abdominal organs

What are the functions of the liver?

including glucose storage, formation of blood plasma proteins and clotting factors, urea synthesis, cholesterol production, bile formation, destruction of red blood cells, storage of iron and vitamins, and detoxification. (Weber 474)

pain in the leg muscles that occurs during exercise and is relieved by rest "do you have pain in the back of your legs" "does rest relieve the pain"

intermittent claudication

Define: scars that form at the site of a wound and grow beyond the normal boundaries of the wound

keloid

client opens eyes, answers questions, then falls back to sleep

lethargy

Define: solely a color change, flat and circumscribed, less than 1cm. (freckles, measles, scarlett fever)

macule

What component of the general survey describes gait, range of motion, motor activity, and involuntary movements?

mobility

Name this lymph node (3)

occipital (node)

In general, a BMI 25-30 is considered __________

overweight

secretes a small amount of pericardial fluid that allows for smooth, friction-free movement of the heart

parietal pericardium

Name this gland

parotid gland

what will you see with gastronintestinal diseases

peptic ulcers duodenal ulcers GERD Chron's disease

Define: papules coalesce to form surface elevation wider than 1cm (psoriasis, lichen planus)

plaque

Which function of the skin does this describe: the skin is the surface on which uv light converts cholesterol into vitamin d

production of vitamin D

Define: difference between systolic and diastolic pressure

pulse pressure

A dark-skinned client visits the clinic because he "hasn't been feeling well." To assess the client's skin for jaundice, the nurse should inspect the client's

sclera.

PHQ-9 score of 20+ indicates?

severe- combination treatments and/or referral to behavioral health

What equipment do you need for an abdominal assessment?

small pillow -centimeter ruler -stethoscope -marking pen

For people age 18-59 a BP with a systolic 140-159 and diastolic 90-99 can be described as _______________

stage I hypertension (18-59)

What precautions should be put in place when measuring vital signs?

standard precautions (thermometer probe cover, sanitizing shared equipment with alcohol wipes, washing hands, use of PPE when necessary)

things a person tells you about that you cannot observe through your senses; symptoms

subjective data

Name this duct

sublingual duct

Define: wheals coalesce to form extensive reaction, intensely pruritic (itchy)

urticaria (hives)

Food frequency questionaire

use to track certain foods over time, quick and often combined with 24 hour recall, requires intact memory

Lower chambers of the heart thicker walls (left ventricle 3X thicker than right), pumps blood against much higher pressure

ventricles

To appropriately document ______ you would use a balance beam or electronic scale. If you are weighing your patient on a balance beam you will make sure the beam is at zero. Patient will remove shoes and step on the scale facing the balance beam. Slide large weight first then small weight forward until the beam is balanced, then document weight.

weight

usually pathological heard over the apical area with the pt in a supine or left lateral position sounds like TEN-nes-see

what does an S4 sound indicate? where is it best heard?

Define: superficial, raised, transient, and erythematous; slightly irregular shape from edema (mosquito bite, allergic reaction)

wheal

hive, edematous papule or plaque, lasts over 24 hours

wheal (primary)

a whispered phrase heard through the stethoscope that sounds faint and inaudible over normal lung tissue · ask the client to repeat "123" Normal- soft and muffled "123"

whispered pectoriloquy

Define: excessive dryness

xerosis

Define: What kind of shape/configuration of lesions is characterized by linear arrangement along a unilateral nerve route (EX. shingles, herpes zoster)

zosteriform (dermatomal)

In examining a client's external auditory canal with an otoscope, the nurse discovers impacted ear wax, known as cerumen. Which of the following is characteristic of cerumen? Select all that apply.

-Has a sticky consistency -Keeps the tympanic membrane soft -Has bacteriostatic properties -Serves as a defense against foreign bodies

A nurse practitioner is assessing a client in the ED following a motor vehicle accident. The client complains of ear pain. The nurse practitioner is performing an otoscopic examination. What would demonstrate the correct technique for using the otoscope?

-Holding the otoscope so that the thumb is by the window -Holding the clients ear at the helix -Rotating the otoscope slightly

A 55-year-old male presents to the health care clinic with reports of decrease hearing over the past year. Which subjective data in client's review of systems should the nurse recognize as risk factors for hearing loss? Select all that apply.

-Increased cerumen production -Repeated episodes of otitis media as a child -History of measles at age 3 years of age

A client visits the health care facility with reports of lumbar back pain that radiates down the back. The nurse performs the straight leg test to determine the origin of the pain. Which techniques should the nurse use to perform this test?

-Instruct the client to bend forward and touch the toes -Instruct the client to touch the chin to the chest -Palpate the spinous processes and the paravertebral muscles -*Ask the client to raise the leg to the point of pain and then dorsiflex the foot* P545

The nurse is working with a client who has leukemia, which affects the red marrow of the bones. The nurse understands that which of the following is characteristic of red marrow?

-Is composed mostly of fat -*Produces red blood cells* -Is hard and dense and makes up the shaft and outer layers -Covers the bones and contains osteoblasts and blood vessels

A nurse performs inspection and palpation of a client's knee and detects swelling. What is the appropriate test the nurse should perform next to determine the cause of the swelling?

-McMurray's test -*Bulge test* -Range of motion -Ballottement test Explanation: If swelling is detected in the knee, the nurse should perform the bulge test to determine if the swelling is due to an accumulation of fluid or soft-tissue swelling. The bulge test will help to detect small amounts of fluid in the knee. Ballottement is a knee test used to assess for the presence of large amounts of fluid in the knee. McMurray's test is useful to confirm a meniscal tear. Pain or clicking during the test is indicative of a torn meniscus of the knee. Range of motion is not useful in determining the cause of swelling.P555

A nurse is testing the range of motion of a client's wrist for supination. Which movement will this involve?

-Moving the tips of the fingers toward the forearm -Moving the tips of the fingers away from the forearm -*Turning the palm of the hand upward* -Turning the palm of the hand downward

Which glands are responsible for mouth drainage? Select all that apply.

-Parotid. -Sublingual. -Submandibular.

A client presents to the emergency department after falling off a ladder while doing some outside painting at home. The client's ankle appears swollen, out of alignment, and is painful to touch. What is the nurse's first action?

-Splint and immobilize the affected extremity. -Encourage early weight bearing and ambulation. -*Check for a pulse, color, temperature, and capillary refill.* -Apply an ice pack to the affected extremity. Exp: The first nursing actions include taking vital signs, monitoring pulses, and assessing color, temperature, and capillary refill distal to the injury to evaluate tissue perfusion. The ankle should then be immobilized after assessment. An ice pack may be applied after assessing for temperature and pulses, etc. The first action is no weight bearing until the ankle is fully assessed.

A college age athlete presents to the clinic with pain in the tibiotalar joint. It is a hinge joint limited to flexion and extension. The terms used to describe these movements are what?

-Supination and pronation -Rotation and supination -*Dorsiflexion and plantar flexion* -Adducting and abducting Exp: The terms used to describe the movements of the tibiotalar joint are dorsiflexion and plantar flexion. Adducting means to move a part of the body toward the midline. Abducting is moving a part of the body away from the midline. Supination is a motion where the foot or palm of the hand is moved to a surface up position. Pronation is a motion where the foot or palm of the hand is moved to a surface down position. Rotation is simply the movement of the joint. Rotation could be either internal or external in nature. P530

A nurse is caring for an adult client who has just undergone surgery to remove a thyroid tumor. The nurse is assessing for symptoms of hyperthyroidism. What are some of the symptoms of hypermetabolism? Select all that apply.

-Tachycardia -Diarrhea -Anxiety

A 32-year-old warehouse worker presents for evaluation of low back pain. He notes a sudden onset of pain after lifting a heavier-than-usual set of boxes. He also states that he has numbness and tingling in the left leg. What test should the nurse perform to assess for a herniated disc?

-Tinel's test -Phalen's test -*Straight leg raise test* -Leg length test Exp: The straight leg raise test involves having the client lie supine with the examiner raising the leg. If the client experiences a sharp pain radiating from the back down the leg in an L5 or S1 distribution, that suggests a herniated disc. Leg strength test, Tinel's test, and Phelan's test do not assess for a herniated disc. P545-546

Which nutrient deficiency should a nurse recognize as placing a client at risk for osteoporosis?

-Vitamin D -Protein -*Calcium* -Vitamin C

A client reports a decrease in appetite over the past month. What additional assessment should a nurse gather in relation to this data? Select all that apply.

-Weight -Food intake

A nurse is inspecting a client's gait. Which of the following would indicate an abnormal finding?

-Weight is evenly distributed -Arms swing in opposition -*Toes point out* -Posture is erect Exp: Abnormal findings in gait include the following: uneven weight bearing is evident; client cannot stand on heels or toes; toes point in or out; client limps, shuffles, propels forward, or has wide-based gait. Posture being erect, arms swinging in opposition, and weight being evenly distributed are all normal findings. P541

A client visits the clinic and tells the nurse that after playing softball yesterday, he thinks his knee is "locking up." The nurse should perform the McMurray test by asking the client to

-bend forward while trying to touch the toes. -move from a standing to a squatting position. -*flex the knee and hip while in a supine position.* P556 -raise his leg while in a supine position.

The external covering of the bone that contains osteoblasts and blood vessels is termed the

-connective tissue. -*periosteum.* P528 -cartilage. -synovial membrane.

An older adult client visits the clinic and tells the nurse that she has had shooting pain in both of her legs. The nurse should assess the client for signs and symptoms of

-rheumatoid arthritis. -metastases. -*herniated intervertebral disc.* -osteoporosis. Thirty-three bones: 7 concave-shaped cervical (C); 12 convexshaped thoracic (T); 5 concave-shaped lumbar (L); 5 sacral (S); and 3-4 coccygeal, connected in a vertical column. Bones are cushioned by elastic fibrocartilaginous plates (intervertebral discs) that provide flexibility and posture to the spine. Paravertebral muscles are positioned on both sides of vertebrae.

What is the deep tendon reflex scale:

0 - no response 1+ - somewhat diminished 2+ - average 3+- brisker than average 4+- very brisk/hyperactive (w/ clonus)

How would you document a pulse with a force that is diminished and barely palpable?

1+

what number correlates with mild pitting edema? (2mm depressions disappears rapidly)

1+

what is done during a physical exam

1. inspection 2.auscultation 3. percussion 4. palpation

Define: Providing false assurance or reassurance, giving unwanted advice, using authority, using avoidance language, engaging in distancing, using professional jargon, using leading/biased questions, talking too much, interrupting, using "why" questions

10 traps of interviewing

How many breaths per minute is normal for an adult at rest?

10-20

To calculate the ideal body weight for a woman, the nurse allows...?

100 pounds for 5 feet of height.

Describe the size of the kidney, location according to vertebrate and which kidney is lower?

10x5x2.5 T12-L3 The right kidney sits lower

what is the "normal" angle of attachment for nails

160 degrees abnormal: 180 degrees

A nurse is examining a client's neck and is preparing to palpate the thyroid gland. The nurse would most likely expect to palpate how many lobes?

2

How would you document a pulse with a force that is brisk and expected (normal)?

2+

Edema with moderate pitting (4mm) and indentation that subsides rapidly is noted as:

2+ (edema)

In what order should you take these steps for the physical examination of the skull and hair? 1. palpate the hair for texture 2. inspect the head/skull for size 3. inspect hair for quantity, color, and distribution 4. observe scalp for hygiene, scaliness, lesions, or pests 5. palpate the skull for shape, lumps, depressions, or tenderness as well as the temporal artery

2, 3, 1, 4, 5

A patient in the clinic where you work is considered legally blind. The nurse knows that this means the vision in his better eye, corrected by glasses, is what?

20/200 or less In the United States, a person is usually considered legally blind when vision in the better eye, corrected by glasses, is 20/200 or less.

A client is able to actively move the right arm against gravity. How should the nurse document this finding using the muscle strength grading scale?

3

A 72-year-old teacher comes to a skilled nursing facility for rehabilitation after being in the hospital for 6 weeks. She was treated for sepsis and respiratory failure and had to be on a ventilator for 3 weeks. The nurse is completing an initial assessment and evaluating the client's skin condition. On her sacrum there is full-thickness skin loss that is 5 cm in diameter with damage to the subcutaneous tissue. The underlying muscle is not affected. What is the stage of this pressure ulcer?

3 A stage III ulcer is a full-thickness skin loss with damage to or necrosis of subcutaneous tissue that may extend to, but not through, the underlying muscle.

How would you document a pulse with a force that is full and bounding?

3+

what number correlates with severe pitting edema (8mm depression lasts >2 minutes)

4+

An adult client visits the clinic complaining of a sore throat. After assessing the throat, the nurse documents the client's tonsils as 4+. The nurse should explain to the client that 4+ tonsils are present when the nurse observes tonsils that are

4+ Tonsils touch each other.

When calculating ideal body weight for women, the health care professional adds how many pounds for each inch over 5 feet?

5

What is the term for the following definition? Definition: recognizes the need for a set of skills necessary to care for people of different cultures A- cultural competence B- cultural sensibility C- cultural humility D- cultural vulnerability

A (cultural competence)

Which of the following is NOT a category when performing a general survey on a patient? A- thought processes B- mobility C- body structure D- physical appearance

A (thought processes)

Catatonia:

A behavioral syndrome marked by an inability to move normally

Upon inspection of a client's oral cavity, a nurse observes a bifid uvula. What should the nurse recognize about this finding?

A bifid or split uvula is a common finding in the Native American population. Clients with a bifid uvula may have a submucous cleft palate. Paralysis of cranial nerve X (vagus) often causes the uvula to deviate to one side and the palate to fail to rise. A CVA may cause asymmetrical or loss of movement of the uvula. Enlargement of the tonsils does not cause a bifid uvula.

A client presents at the emergency room reporting "the worst headache I have ever had." What are critical nursing behaviors for this client? (Select all that apply.)

A client with severe headaches may be unable to provide a complete history, but a focused history and physical examination looking for neurologic changes are critical nursing behaviors. Nursing behaviors do not include MRIs, CT scans, or EEGs

Which of the following assessment findings suggests a problem with the client's cranial nerves?

A client's extraocular movements are asymmetrical and she complains of diplopia. Deficits in cranial nerves III, IV, and VI can manifest as impaired extraocular movements or diplopia. Flashes of light are associated with retinal detachment, while intraocular bleeding and cataracts do not have a neurological etiology.

Schizophrenia:

A disorder that affects a person's ability to think, feel, and behave clearly

Preauricular tag:

A facial remnant or leftover of the embryologic branchial arch usually appears as as kin tag; one containing cartilage. Occurs most often in preauricular area, in front of the tragus

A nurse is assessing a small child who has lead poisoning. Which characteristic of the gums should the nurse expect this client?

A grey-white line.

The nurse notes a tophus of the ear of an older adult. Which assessment data is consistent with a tophus?

A hard nodule composed of uric acid crystals

A nurse finds crepitus when palpating over a client's maxillary sinuses. Which of the following should the nurse most suspect in this client?

A large amount of exudate in the sinuses.

You are assessing visual fields on a patient newly admitted for eye surgery. The patient's left eye repeatedly does not see your fingers until they have crossed the line of gaze. You would document that the patient has what?

A left temporal hemianopsia When the patient's left eye repeatedly does not see your fingers until they have crossed the line of gaze, a left temporal hemianopsia is present.

Thyroid nodule:

A lump in the thyroid, the butterfly-shaped gland at the base of the neck; can be cancerous/non-cancerous

Preauricular sinus:

A nodule, dent or dimple located anywhere adjacent to the external ear

The peritoneum is a serous membrane that contains which of the following?

A parietal layer

An increased risk of falls is dangerous for any patient. What patient would be at an increased risk of falls?

A patient with vertigo

You are teaching a class on diseases of the ear. What would you teach the class is the most characteristic symptom of otosclerosis?

A progressive, bilateral loss of hearing

The nurse notest that the client's tongue appears as shown. What should the nurse suspect is occurring with this client? (cracked at tip)

A smooth and often sore tongue that has lost its papillae, sometimes just in patches, suggests a deficiency in riboflavin, niacin, folic acid, vitamin B12, pyridoxine, or iron. Candidiasis or white patches would be on the tongue if a yeast infection is present. The tongue may have black areas if antibiotics were recently used. The tongue would deviate from the midline if CN XII is damaged.

A 55-year-old male client has just been diagnosed with presbycusis. In the interview with the client, the nurse should most expect the client to complain of having trouble hearing which of the following in the initial stages of this condition? a) The sound of his car engine starting b) A story his wife is telling him c) The bass speakers of his stereo system d) His son giving him directions to a restaurant

A story his wife is telling him Explanation: Presbycusis often begins with a loss of high-frequency sounds (woman's voice) followed later by the loss of low-frequency sounds. The bass speakers, his son's voice, and the engine starting would all have lower-frequency sounds than his wife's voice.

While the nurse is assessing a client for an unrelated health concern, the client experiences a sudden, severe headache with no known cause. He also complains of dizziness and trouble seeing out of one eye. What associated condition should the nurse suspect in this client?

A sudden, severe headache with no known cause may be a sign of impending stroke, particularly if accompanied by sudden trouble seeing in one or both eyes or sudden trouble walking, dizziness, and loss of balance or coordination. Only impending stroke is associated with all of these symptoms. Diabetes is not associated with headache or the other symptoms. A tumor-related headache is aching and steady and not necessarily associated with sudden onset. Hyperthyroidism is associated with goiter, bruit, and sudden weight loss, but not with any of the symptoms listed.

Which of the following clients is most likely to be diagnosed with migraine headaches? A. A woman whose headaches come on suddenly and are somewhat relieved by a quiet, dark room B. A woman who complains of recurrent headaches near the end of her workday spent at a computer station C. A man who has sought care for treatment of his episodic headaches that occur several times each day D. A man whose headaches are accompanied by severe light sensitivity but an absence of nausea

A woman whose headaches come on suddenly and are somewhat relieved by a quiet, dark room

What does the Melanoma ABCDE stand for?

A- asymmetry B- border irregularity C- color (blue/black) D-diameter (greater than 6mm or different from others) E-evolving (elevation or enlargement)

What are the 4 main headings of the mental status assessment?

A-B-C-T Appearance Behavior (LOC, posture/motor behavior, dress/hygiene, facial expression, and speech rate, loudness, fluency, rate) Cognition (A&O, attention, recent memory, remote memory, learning ability, vocabulary, ability to calculate, and abstract thinking) Thought process (mood, thoughts, perception, thought content, and judgement)

A nurse is interviewing a client whose chief complaint is temporomandibular joint pain. Which of the following questions should the nurse ask regarding a causative factor? A. "Do you grind your teeth?" B. "Do you drink alcohol?" C. "Is there a history of mouth cancer in your family?" D. "How often do you brush and use dental floss?"

A. "Do you grind your teeth?" Rationale: Grinding the teeth (bruxism) may be a sign of stress or of slight malocclusion. The practice may also precipitate temporomandibular joint (TMJ) problems and pain. The other answers are not causative factors associated with TMJ pain.

A nurse is interviewing a client who has recently been diagnosed with terminal disease. In covering the lifestyle and health practices profile, the nurse asks the client, "Are you close to any extended family members in the area?" The client objects to the question and asks why the nurse needs to know that. Which is the best rationale for the nurse posing this question? A. "I just wanted to see what kind of social support you might have to help care for you during your illness." B. "I'm just being friendly. We like to get to know our clients at this practice." C. "I just thought i might know them; I know pretty much everyone in this town." D. "With you having a terminal illness, you will need someone to help you plan your funeral." (Okay seriously wtf)

A. "I just wanted to see what kind of social support you might have to help care for you during your illness." Rationale: Ask clients to describe the composition of the family into which they were born and about past and current relationships with these family members. In this way, you can assess problems and potential support from them client's family or origin. Just being friendly and determining what acquaintances the nurse might have in common with the client are not proper rationales for asking this personal information. Mentioning plans for the client's funeral is blunt and would likely upset the client.

A nurse is assessing the blood pressure of a client who has come to the healthcare facility for the first time. Which of the following is the best site for obtaining the client's blood pressure reading? A. Arm B. Thigh C. Wrist D. Shoulder

A. Arm Rationale: The first time the blood pressure is measured, its is assessed in each arm. The two blood pressure measurements should not vary more than 5 to 10 mm Hg unless pathology (disease) is present. The blood pressure is not measured in shoulders, wrist or thighs of clients for the first time. Nurses use the thighs to assess the blood pressure when they cannot obtain readings in either of the client's arms.

The nurse recognizes the following to be a necessary component of performing an accurate assessment. (Select all that apply.) A. Collection and organization of data B. Inaccurate data C. Incomplete data D. Validation of data E. Documentation of data

A. Collection and organization of data D. Validation of data E. Documentation of data Rationale: Before beginning to analyze data, the nurse must make sure the assessment is accurately performed, which includes collection and organization, validation and documentation of the data. The nurse does not want to include any inaccurate or incomplete data — doing so will lead to a faulty assessment.

A client is brought to the ED in a confused state. Upon examination of the client's mouth, the nurse detects a fruity odor to the breath. The nurse recognizes this finding as a characteristic of what disease process? A. Diabetic ketoacidosis B. Small-bowel obstruction C. End-stage liver disease D. Respiratory infection

A. Diabetic ketoacidosis Rationale: The nurse should suspect the client to having diabetic ketoacidosis on the basis of the fruity smell of his breath. Clients with end-stage liver disease have a sulfur odor in their breath. Clients with small-bowel obstructions have a fecal smell, and clients with respiratory infection have foul odors in their breath.

After completing the physical examination of a client who is 12 weeks pregnant, a new nurse leaves the room only to realize she forgot to complete an examination of the skin. What should the nurse do? A. Go back in to complete a physical examination of the skin B. Ask a colleague who saw the client earlier C. Review the documented client history D. Omit this part of the physical examination

A. Go back in to complete a physical examination of the skin Rationale: It is common to forget part of the physical examination, especially at first. It is not unusual to go back to the client and ask to check one or two items that have been overlooked. Omitting this part of the physical examination could lead to missing important clinical data for planning client care. Reviewing the documented client history will not provide objective information about the current status of the client's skin. Asking a colleague who saw the client earlier is not an accurate way to collect information for a physical examination of the skin. The new nurse should not rely on the memory of her colleague alone for this information.

When caring for a client, what aspect is important in conducting a conversation? A. Keep the conversation at the knowledge level of the client B. Use medical terminology so that the client learns it C. Discuss only the least important topics first D. Include the family in all discussions

A. Keep the conversation at the knowledge level of the client Rationale: It is important to conduct a conversation at the knowledge level of the client. Medical terminology is used only if the client is already familiar with the terms. Discussing the least important topics first is not correct. Important or time sensitive topics should be prioritized and discussed first. The client's permission should always be asked before carrying out a conversation with family present.

As part of an abdominal assessment, the nurse must palpate a client's liver. In which quadrant is this organ located? A. Right upper quadrant B. Left lower quadrant C. Right lower quadrant D. Left upper quadrant

A. Right upper quadrant Rationale: The liver is the largest solid organ in the body. It is located below the diaphragm in the right upper quadrant of the abdomen.

The nurse is preparing to assess the lymph nodes of an adult client. The nurse should instruct the client to: A. Sit in an upright position B. Lie in a supine position C. Lie in a side-lying position D. Stand upright in front of the nurse

A. Sit in an upright position Rationale: Have the client remain seated upright. Then palpate the lymph nodes with your finger pads in a slow walking, gentle, circular motion.

The nurse places the back of the hand on the forehead of a client. What is the nurse assessing when using this part of the hand? A. Temperature B. Texture C. Vibration D. Pulses

A. Temperature Rationale: The dorsal or back surface of the hand is used to assess temperature. The pads of the fingers are used to assess pulses and texture. The ulnar or palmar surface of the hand is used to assess vibration.

While conducting the physical examination, which of the following assessments would require the nurse to auscultate the abdomen? A. To identify bowel sounds B. To identify the distribution of gas in the abdomen C. To identify abdominal tenderness D. To identify the edges of abdominal organs

A. To identify bowel sounds Rationale: Auscultation is used to identify bowel sounds when conducting the physical examination of the gastrointestinal system. Deep palpation is used to identify the edges of the liver, kidney and other abdominal masses. Light palpation is applied to identify abdominal tenderness along with muscular resistance and some superficial organs and masses. Percussion is used to identify the amount and distribution of gas in the abdomen.

When palpating the abdomen the nurse finds a large pulsating mass. The nurse would suspect this is what?

Abdominal aortic aneurysm Pulsation of the aorta may be increased and lateralized in an abdominal aortic aneurysm. Ascites is collection of fluid in the abdomen. Inflammation and tumors do not pulsate.

What are examples of cutaneous reflex?

Abdominal reflex Plantar reflex -Normal adult: toes curl up -Normal baby: toes fan out -Abnormal adult: toes fan out

How do many older adults define their health?

Ability to function independently

Where is the temporal artery palpated?

Above the cheek bone near the scalp line

Where is the temporal artery palpated? A. Above the cheek bone near the scalp line B. Just left of midline at the base of the neck C. Between the mandibular joint and the base of the ear D. Just left or right of the spine at the base of the skull

Above the cheek bone near the scalp line

A nurse examines a client with a paralytic ileus. Which alteration in bowel sounds should the nurse expect to find with auscultation of the client's abdomen?

Absent

A nurse examines a client with paralytic ileus. Which alteration in bowel sounds should the nurse expect to find with auscultation of the clients abdomen?

Absent

A client complains of a burning sensation in the esophagus after eating. Which associated condition should the nurse most suspect?

Acid reflux

You are using the Braden Scale to measure risk factors for pressure sores. What risk factors will you assess? Select all that apply.

Activity Moisture Nutrition The Braden Scale is a simple effective tool that evaluates levels of risk for ulcer development in the patient. With its high reliability, predictive validity, and ease of use, the Braden Scale can be used to assess patients as often as every shift if needed. Six factors are rated using a matrix scoring system: sensory perception, moisture, activity, mobility, nutrition, and friction and shear.

A 26-year-old sports store manager comes to the clinic with severe right-sided abdominal pain for 12 hours. He began having a stomach ache yesterday with a decreased appetite, but today the pain seems to be just on the lower right side. He has had some nausea and vomiting but no constipation or diarrhea. His last bowel movement was last night and was normal. He has had no fever or chills. He denies any recent illnesses or injuries. His past medical history is unremarkable. He is engaged. He denies any tobacco or drug use and drinks four to six beers per week. His mother has breast cancer and his father has coronary artery disease. On examination he appears ill and is lying on his right side. His temperature is 38.8° and his heart rate is 170. His bowel sounds are decreased, and he has rebound and involuntary guarding at one third of the way between the anterior superior iliac spine and the umbilicus in the right lower quadrant. His rectal, inguinal, prostate, penile, and testicular examinations are normal. What is the most likely cause of his pain?

Acute appendicitis

Delirium:

Acute confusional state, potentially preventable in hospitalized persons. Characterized by disorientation, disordered thinking and perceptions (illusions and hallucinations), defective memory, atitation, inattention. Sudden onset; reversible; often caused by infection or change in health/drug reaction

Upon examination of the head and neck of a client, a nurse notes that the submandibular nodes are tender and enlarged. The nurse should assess the client for further findings related to what condition?

Acute infection

Upon examination of the head and neck of a client, a nurse notes that the submandibular nodes are tender and enlarged. The nurse should assess the client for further findings related to what condition? A. Metastatic disease B. Chronic infection C. Acute infection D. Cushing's disease

Acute infection

After examining the client's tympanic membranes, the nurse documents "Right tympanic membrane, red and bulging with no light reflex." The nurse recognizes that these are signs of

Acute otitis media

The client has epigastric pain that is poorly localized and radiates to the back. What would be an important diagnosis to assess for?

Acute pancreatitis

Moving a part of the body away from the midline is called?

Adduction Rotation Extension *Abduction* P528

A client suffering from a headache complains of throbbing, severe, unilateral pain that feels worse when exposed to bright lights. The client also complains of nausea and vomiting. What is the nurse's best action?

Administer migraine medication

A client suffering from a headache complains of throbbing, severe, unilateral pain that feels worse when exposed to bright lights. The client also complains of nausea and vomiting. What is the nurse's best action? Administer narcotic pain medication Administer migraine medication Administer medication for common cold Prepare the client for a spinal tap

Administer migraine medication

Which abdominal finding in an elderly client should prompt a nurse to perform additional assessment to determine the cause?

An enlarged liver felt during palpation The liver normally decreases in size after age 50 years. An enlarged liver needs further assessment. Appetite decreases with age due to altered metabolism, decreased taste sensation, decreased mobility, and possibly depression. Tympany is a normal finding over the stomach. The fluid wave test should be negative unless fluid (ascites) is present in the abdomen.

While examining a client, the nurse observes that he appears to be nodding his head involuntarily. Which of the following conditions should the nurse additionally assess for, based on this finding?

An involuntary nodding movement may be seen in patients with aortic insufficiency. Neurologic disorders may cause a horizontal jerking movement. Head tilted to one side may indicate unilateral vision or hearing deficiency or shortening of the sternomastoid muscle. Acorn-shaped, enlarged skull bones are seen in Paget's disease of the bone.

Diplopia present with one eye covered can be caused by which of the following problems?

An irregularity in the cornea or lens Double vision in one eye alone points to a problem in "processing" the light rays of an incoming image. The other causes of diplopia result in a misalignment of the two eyes.

is a diffuse swelling of the dermis and subcutaneous tissue.

Angioedema

is the presence of fissures at the edges of the mouth.

Angular cheilitis

to determine early issues with arterial disease -ratio of the ankle systolic pressure to the brachial systolic pressure; an objective measurement of arterial disease that provides quantification of the degree of stenosis

Ankle-Brachial Index (ABI)

15. The nurse has assessed the nose of an adult client and has explained to the client about her thick yellowish nasal discharge. The nurse determines that the client understands the instructions when the client says that the yellowish discharge is most likely due to

Ans: An upper respiratory infection

2. An adult client visits the clinic complaining of recurrent ulcers in the mouth. The nurse assesses the client's mouth and observes a painful ulcer. The nurse should document the presence of

Ans: Aphthous stomatitis

11. The nurse is assessing an adult client's oral cavity for possible oral cancer. The nurse should explain to the client that the most common site of oral cancer is the

Ans: Area underneath the tongue

5. The nurse is assessing the mouth of an older adult and observes that the client appears to have poorly fitting dentures. The nurse should instruct the client that she may be at greater risk for

Ans: Aspiration

10. The nurse is assessing a client who has been taking antibiotics for an infection for 10 days. The nurse observes whitish curd-like patches in the client's mouth. The nurse should explain to the client that these spots are most likely

Ans: Candida albicans infection

7. The nurse is planning a presentation to a group of high school students about the risk factors for oral cancer. Which of the following should be included in the nurse's plan?

Ans: Diets low in fruits and vegetables are a possible risk for oral cancer

8. Before examining the mouth of an adult client, the nurse should first

Ans: Don clean gloves for the procedure

14. The nurse is preparing to inspect the nose of an adult client with an otoscope. The nurse plans to

Ans: Position the handle of the otoscope to one side

4. An adult client visits the clinic and tells the nurse that she has been experiencing frequent nosebleeds for the past month. The nurse should

Ans: Refer the client for further evaluation

3. A nurse assesses the mouth of an adult male client and observes a rough, crusty, eroded area. The nurse should

Ans: Refer the client foro further evaluation

9. A client visits the clinic and tells the nurse that she has painful cracking in the corners of her lips. The nurse should assess the client's diet for a deficiency of

Ans: Riboflavin

6. An adolescent client tells the nurse that her mother says she grinds her teeth when she sleeps. The nurse should explain to the client that grinding the teeth may be a sign of

Ans: Stress and anxiety

13. An adult client visits the clinic complaining of a sore throat. After assessing the throat, the nurse documents the client's tonsils as 4+. The nurse should explain to the client that 4+ tonsils are present when the nurse observes tonsils that are

Ans: Touching each other

When reviewing ear assessment, a student nurse would learn that the cone of light should be visible where on the tympanic membrane?

Anterior inferior quadrant

The Kiesselbach plexus is the most common site for what?

Anterior nosebleeds.

What medications can cause drying of the mucous membranes?

Antihistamines and antidepressants

2nd ICS, right of sternum -base

Aortic landmark

An emergency department nurse is caring for a teenage client who has severe pain in the umbilical area. Documentation shows that the client exhibits "Rovsing's sign." What medical diagnosis is associated with the assessment finding?

Appendicitis Rovsing's sign is an indicator of appendicitis. It is not a sign of gastroenteritis, liver disease, or an enlarged spleen.

An emergency department nurse is caring for a teenage client who has severe pain in the umbilical area. Documentation shows that the client exhibits "Rovsing's sign." What might this client's medical diagnosis be?

Appendicitis Rovsing's sign is an indicator of appendicitis. It is not a sign of gastroenteritis, liver disease, or an enlarged spleen.

The nurse notes that a client experiencing right lower quadrant abdominal pain when the hip and knee are flexed, and the leg is rotated internally and externally. What should the nurse suspect is occurring with this client?

Appendicitis The client is demonstrating a positive obturator sign that causes pain in the right lower abdominal quadrant when the hip and knees are flexed and the leg is rotated internally and externally. Rebound tenderness occurs with peritoneal irritation. There is no specific sign for liver inflammation. Pain that occurs when pressure is applied under the liver border at the right costal margin indicates an inflamed gall bladder.

Chris is a 20-year-old college student who has had abdominal pain for 3 days. It started at his umbilicus and was associated with nausea and vomiting. He was unable to find a comfortable position. Yesterday, the pain became more severe and constant. Now, he hesitates to walk, because any motion makes the pain much worse. It is localized just medial and inferior to his iliac crest on the right. Which of the following is most likely?

Appendicitis This is a classic history for appendicitis. Notice that the pain has changed from visceral to parietal. It is well localized to the right lower quadrant, making appendicitis a strong consideration.

The nurse is assessing an older adult client who has lost albs since her last visit one year ago. The client tells the nurse that her husband died 2 months ago. The nurse should further assess the client for:

Appetite changes

The nurse would assess for positive Blumberg sign how? Applying blunt pressure at the costovertebral angle (CVA) Having the client breathe deeply Applying and releasing pressure to the abdomen Applying blunt pressure that the midclavicular line (MCL)

Applying and releasing pressure to the abdomen

The nurse is inspecting the cornea and lens of an elderly client and notices a white arc around the limbus of the client's eye. The nurse recognizes this condition, common in older adults, as which of the following?

Arcus senilis Arcus senilis, a normal condition in older clients, appears as a white arc around the limbus. The condition has no effect on vision. Presbyopia, which is impaired near vision, is caused by decreased accommodation and is a common condition in clients over 45 years of age. Ectropion is when the lower eyelids evert, causing exposure and drying of the conjunctiva. This is a normal finding in the older client. Myopia is impaired far vision.

A 82 year old female presents with neck pain, decreased strength and sensation of the upper extremities. The nurse identifies that this could be related to what? A. Arthritic changes of the cervical spine B. Bacterial thyroiditis C. Cranial damage D. Muscle tension

Arthritic changes of the cervical spine

A client complains of a unilateral headache near the scalp line and double vision. The nurse palpates the space above the cheekbone near the scalp line on the affected side, and the client complains of tenderness on palpation. What is the nurse's next action?

Arthritis

An 81-year-old client complains of neck pain and demonstrates decreased range of motion on examination. Which of the following causes should the nurse most suspect in this client?

Arthritis

An 81-year-old client complains of neck pain and demonstrates decreased range of motion on examination. Which of the following causes should the nurse most suspect in this client? A. Meningeal inflammation B. Stress C. Injury to the sternomastoid D. Arthritis

Arthritis

To assess for anemia in a dark-skinned client, the nurse should observe the client's skin for a color that appears. Pallor (loss of color) is seen in arterial insufficiency, decreased blood supply, and anemia. Pallid tones vary from pale to ashen without underlying pink.

Ashen

A nurse is preparing to examine a client from Southeast Asia who has been experiencing chronic headaches. Which of the following should the nurse do in light of this client's cultural background? A. Avoid asking the client to remove her clothes for the examination B. Have a nurse who is the same sex as the client perform the examination C. Ask permission before palpating the head and neck D. Palpate the client's feet before palpating the head

Ask permission before palpating the head and neck

As the nurse assesses the temporomandibular joint (TMJ), an audible click is heard and palpated. What is nurse's best action?

Ask the client if painful to move jaw side to side.

The nurse is preparing to perform a head and neck assessment of an adult client who has immigrated to the United States from Cambodia. The nurse should first

Ask the client if toughing the head is permissible

The nurse is planing to assess the abdomen of an adult male client. Before the nurse begins the assessment, the nurse should:

Ask the client to empty his bladder

What do you do ask the patient to do before you perform an abdominal assessment?

Ask the client to empty the bladder before beginning the examination to eliminate bladder distention and interference with an accurate examination. (Weber 484)

A nurse inspects a client's abdomen and notices that a bulge is present in the right lower quadrant. How should the nurse further assess this finding using inspection?

Ask the client to raise the head off the bed

A client presents complaining of nausea, vomiting, and acute abdominal pain. What is the nurse's first action?

Ask the client when the pain began

A mother of a small child calls the clinica and asks to schedule an appointment for ear tube removal. The call is transferred to the nurse. What is the nurse's best action?

Ask the mother how long the tubes have been in place

The nurse suspects an enlarged thyroid in a patient during the physical examination of the head and neck. What should the nurse first? Displace the trachea to the right. Listen over the thyroid with a stethoscope. Ask the patient to sip and swallow water. Ask the client to lie down for further assessment

Ask the patient to sip and swallow water.

Upon inspection of an elderly client's ears using an otoscope, the nurse observes a cloudy appearance to the tympanic membranes with prominent landmarks. What action should the nurse perform first? a) Assess for balance using the Romberg test b) Perform hearing test to assess for conductive hearing loss c) Document these findings as normal changes in the older adult d) Ask whether the client is experiencing ear pain or pressure

Ask whether the client is experiencing ear pain or pressure Explanation: The older client's eardrums may appear cloudy with prominent landmarks due to the atrophy of the tympanic membranes associated with normal aging. The nurse should ask the client about ear pain because prominent landmarks can also be a sign of negative pressure associated with obstructed eustachian tubes. An ear infection should be ruled out and problems with hearing and balance assessed before making a clinical judgment that this is a normal finding. All objective information should be documented by the nurse.

After a physical assessment, the nurse determines that a client has full range of motion of the temporomandibular joint. Which of the following assessments did the nurse complete with the client? (Select all that apply.) A Asked the client to swallow B Asked the client to open and close the mouth C Asked the client to extend the tongue D Asked the client to rock the jaw laterally E Asked the client to jut the jaw forward

Asked the client to open and close the mouth Asked the client to rock the jaw laterally Asked the client to jut the jaw forward

Which action by the nurse is appropriate to provide a clear view of the uvula for observation?

Asking the client to say "aaah" and instructing him or her to open the mouth wide makes the uvula more clear for observation. The nurse should depress the client's tongue slightly off center to prevent the gag reflex during observation of the uvula. Depressing the back of the tongue would elicit the gag reflex. Having the client stick out the tongue would not provide a clear view of the uvula.

Which technique by the nurse demonstrates proper use of the ophthalmoscope?

Asks the client to fix the gaze upon an object and look straight ahead. After turning on the ophthalmoscope, the nurse should ask the client to gaze straight ahead and slightly upward. Ask the client to remove glasses but keep contact lens in place. The nurse should use the right eye to examine the right eye & left eye to examine the client's left eye. This allows the nurse to get as close as possible to the client's eye. Begin about 10-15 inches from the client at a 15 degree angle. The nurse should keep the ophthalmoscope still & ask the client to look into the light to view the fovea and macula.

A client with a cervical spine injury has chronic pain. What would be the most appropriate initial nursing intervention for this client? Work with medical team to evaluate possible surgery Discuss pharmacologic interventions Chronic pain related to cervical spine injury Assess characteristics

Assess characteristics

A client with a cervical spine injury has chronic pain. What would be the most appropriate initial nursing intervention for this client? A. Work with medical team to evaluate possible surgery B. Discuss pharmacologic interventions C. Chronic pain related to cervical spine injury D. Assess characteristics

Assess characteristics

A 57-year-old client reports, "I am having the worst headache I have ever experienced." Which action should the nurse perform next? Assess the client's blood pressure. Provide medication for pain relief. Inquire about family history of headaches. Review the client's medical record.

Assess the client's blood pressure.

what element of COLDSPA is "any other symptoms associated with daily activities/positions"

Associated factors (COLDSPA)

Trichotillomania:

Associated with OCD; when people are stressed, they pull out hair

As a part of the ear examination for hearing loss, a nurse conducts a Weber test on a client. To accurately perform this test, the nurse should place the base of the tuning fork in which of the following locations?

At the center of the client's forehead

During the physical examination of a client, a nurse notes that a client's trachea has been pushed toward the right side. The nurse recognizes that the pathophysiologic cause for this finding is related to what disease process?

Atelectasis

During the physical examination of a client, a nurse notes that a client's trachea has been pushed toward the right side. The nurse recognizes that the pathophysiologic cause for this finding is related to what disease process? A. Endocarditis B. Bronchitis C. Atelectasis D. Tuberculosis

Atelectasis

During the physical examination of a client, a nurse notes that a client's trachea has been pushed toward the right side. The nurse recognizes that the pathophysiological cause for this finding is related to what disease process? A. Endocarditis B. Bronchitis C. Atelectasis D. Tuberculosis

Atelectasis

When palpating the neck, performing which of the following techniques will help differentiate lymph nodes from a band of muscles? Applying pressure and assessing for induration Attempting to roll the structure up and down and side to side Palpating for lateral movement when the client swallows a sip of water Observing for hypertrophy when the client turns the head against resistance

Attempting to roll the structure up and down and side to side

Inspect/palpate the external ear for:

Auricle (move up and down for pain), tragus (press for pain), lobes, deformities, lumps, lesions, inflammation, foreign bodies, discharge

If thyroid is enlarged, what else would you do?

Auscultate for a bruit

A nurse palpates an elderly client's thyroid and detects an enlargement over the right lateral lobe. What action should the nurse take first? Immediately notify the health care provider Document the findings in the nurse notes Auscultate with the bell over the lateral lobes Ask the client about past history of hypothyroidism

Auscultate with the bell over the lateral lobes

A nurse palpates an elderly client's thyroid and detects an enlargement over the right lateral lobe. What action should the nurse take first? A. Immediately notify the health care provider B. Document the findings in the nurse notes C. Auscultate with the bell over the lateral lobes D. Ask the client about past history of hypothyroidism

Auscultate with the bell over the lateral lobes

A nurse is assessing a client with hyperthyroidism for the presence of a bruit. Which assessment technique should the nurse use? A. Inspection B. Palpation C. Auscultation D. Percussion

Auscultation

Which assessment technique should a nurse use to assess for the presence of a bruit in a client with hyperthyroidism?

Auscultation

Which assessment technique should a nurse use to assess for the presence of a bruit in a client with hyperthyroidism? A. Inspection B. Palpation C. Auscultation D. Percussion

Auscultation

The goal of __________________ is to reduce health disparities among ethnic or cultural groups in the United States A- human resources B- healthy people 2020 C- department of health care services D- USDA food and nutrition service

B (healthy people 2020)

A nurse assesses the radial pulse of a client. Which pulse rate would the nurse document as bradycardia? A. 94 beats/minute B. 56 beats/minute C. 64 beats/minute D. 88 beats/minute

B. 56 beats/minute Rationale: Bradycardia is a rate less than 60 beats/minute. Rates of 64, 88 and 94 would be considered within the normal range of pulse rates.

Acute pain can be differentiated from chronic pain because... A. Acute pain is not treated and left to subside on its own, whereas chronic pain is referred for treatment B. Acute pain is associated with a recent onset of illness or injury with a duration of less than 6 months, whereas chronic pain persists longer than 6 months C. Acute pain always scores more on the visual analog scale than chronic pain D. Acute pain occurs only in persons aged less than 45 years, whereas chronic pain occurs in persons aged 46 or above

B. Acute pain is associated with a recent onset of illness or injury with a duration of less than 6 months, whereas chronic pain persists longer than 6 months Rationale: Acute pain is usually associated with a recent injury or illness and lasts less than 6 months.

A client reports pain and rates it as a 9 on a scale of 0 to 10. The nurse administers pain medication as ordered and returns 20 minutes later to assess the severity of the client's pain. To assess the severity, the nurse would: A. Ask about the location of the pain B. Ask the client to rate the pain on a scale of 0 to 10 C. Ask the client if he or she needs anything D. Ask the client what makes the pain worse

B. Ask the client to rate the pain on a scale of 0 to 10 Rationale: When assessing a client's pain, the nurse should ask about location, duration, intensity, quality, alleviating/aggravating factors, management goal and functional goal. To assess severity or intensity, the nurse should ask the client to rate the pain on a scale of 0 to 10 or 1 to 10.

The student nurse is studying therapeutic communication and realizes that it is an important aspect of collecting a health history. When gathering information about the client's current illness, the student nurse understands that she should use which technique? A. Asking yes and no questions B. Asking open-ended questions C. Showing disapproval D. Avoiding eye contact

B. Asking open-ended questions Rationale: The nurse should use the therapeutic technique of asking open-ended questions to allow the client to verbalize. Yes and no questions may not elicit a proper response or allow the client to give comprehensive information. Showing disapproval and avoiding eye contact are examples of nontherapeutic techniques.

A client performs the test for distant visual acuity and scores 20/50. How should the nurse most accurately interpret this finding? A. Client did not wear his glasses for this test and therefore it is not accurate B. At 20 feet from the chart, the client sees what a person with good vision can see at 50 feet C. When 50 feet from the chart, the client can see better than a person standing at 20 feet D. Client can read the 20/50 line correctly and two other letters on the line above

B. At 20 feet from the chart, the client sees what a person with good vision can see at 50 feet Rationale: The Shelley chart tests distant visual acuity by seeing how far the client can read the letters standing 20 feet from the chart. The top number is how far the client is from the chart and the bottom number refers to the last line the client can read. A reading of 20/50 means the client sees at 20 feet what a person with normal vision can see at 50 feet. The minus number is the number of letters missed on the last line the client can distinguish.

During the introduction phase of the interview, the client begins to talk nonstop about health problems, family issues and fears related to illness. What can the nurse do to control the interview process? A. Tap the pen on the paper while the client talks B. Courteously interrupt the client to clarify some information C. Leave the interview and contact security D. Glance at the clock at the wall

B. Courteously interrupt the client to clarify some information Rationale: The nurse should only interrupt the client when necessary in a courteous manner. The purpose of the interruption should be to clarify some information that the client provided while talking. Glancing at the clock on the wall and tapping the pen on the paper are signs of impatience and should not be done when working with a talkative client. Contacting security is not necessary when interviewing a talkative client.

What characteristic nail color should the nurse recognize as an indication of hypoxia? A. Greenish B. Cyanotic C. Yellowish D. Pink

B. Cyanotic Rationale: Pale or cyanotic nails indicate hypoxia. A normal healthy client would have pink tones in their nail beds. Yellowish nails are observed in clients with hepatitis. Greenish nails are indicative of localized fungal infection.

A nurse is examining a client's nose. Which characteristics of the nasal mucosa should the nurse expect to find if the client is healthy? A. Red, swollen, with purulent discharge B. Dark pink, moist, and free of discharge C. Pale pink, swollen, with watery exudate D. Bluish-gray, swollen, with watery exudate

B. Dark pink, moist, and free of discharge Rationale: Dark pink, moist nasal mucosa which is free of exudate is a normal finding. The nurse should find red, swollen nasal mucosa with purulent discharge in the client diagnosed with upper respiratory tract infection. Pale pink, swollen nasal mucosa with watery exudate and bluish-gray, swollen nasal mucosa with watery exudate is found in cases of allergy.

The nurse is preparing to examine the ears of an adult client with an otoscope. The nurse should plan to: A. Use a speculum that measures 10 mm in diameter B. Firmly pull the auricle out, up and back C. Ask the client to tilt the head slightly forward D. Release the auricle during the examination

B. Firmly pull the auricle out, up and back Rationale: Use the thumb and fingers of your opposite hand to grasp the client's auricle firmly but gently. Pull out, up and back to straighten the external auditory canal. Do not alter this positioning at any time during the otoscope examination.

The nurse performs an assessment by gently pressing up under the brow bone with each thumb on both sides of the nose. What is the nurse assessing in this client? A. Nasal septum B. Frontal sinus C. Maxillary sinus D. Nasal cavity

B. Frontal sinus Rationale: The frontal sinuses are located above the eyebrow bone. A speculum is required to assess the nasal cavity and septum. The maxillary sinuses are located just below the cheekbones.

Which of the following would the nurse suspect when a client with a cardiac condition complains of not sleeping well and having to get up frequently at night to urinate? A. The client most likely sleeps without a pillow at night B. Increased urination at rest may indicate heart failure C. The is indicates the heart is working efficiently D. The client has decreased performance levels of activities of daily living

B. Increased urination at rest may indicate heart failure Rationale: With heart failure, increased renal perfusion during periods of rest or recumbency may cause nocturia. Nocturia does not indicate that the heart is working efficiently. Depending on the client's fatigue level from not sleeping well, as well as other complaints, the client's ability to perform activities of daily living may be affected. If the client is experiencing dyspnea at night, he or she will likely be sleeping on more than one pillow at night.

A group of students is reviewing the structures of the heart, noting that the thickest layer of the heart is made up of contractile muscle cells. How would the students identify this layer? A. Epicardium B. Myocardium C. Pericardium D. Endocardium

B. Myocardium Rationale: The myocardium is the thickest layer of the heart and is made up of contractile cardiac muscle cells. The epicardium is the serous membrane that covers the outer surface of the heart; the endocardium is a thin layer of endothelial tissue that forms the innermost layer of the heart. The pericardium is a tough non-dispensable loose-fitting fibroserous sac that attaches to the greatest vessels and thereby surrounds the heart.

Loss of bone density that occurs with greatest frequency in postmenopausal women is called? A. Kyphosis B. Osteoporosis C. Scoliosis D. Lordosis

B. Osteoporosis Rationale: Loss of bone density is termed osteoporosis. Some osteoporosis occurs in all people, but it is most evident in women with small bone frames. Women experience rapid loss of bone density for the first 5 to 7 years after menopause. Lordosis, kyphosis and scoliosis are conditions that affect the spinal alignment.

A student in the vascular surgery clinic is asked to perform a physical examination on a client with known peripheral vascular disease in the legs. Which of the following aspects are most important to note? A. Lower extremity strength B. Size, symmetry and skin color C. Nodules in joints D. Muscle bulk and tone

B. Size, symmetry and skin color Rationale: Size, symmetry and skin color are important aspects to note in physical examination. Swelling in the legs, cyanosis and lack of appropriate hair growth are all signs of peripheral vascular disease.

A nurse is assessing the mouth of a client and finds that she has a smooth, red, shiny tongue without papillae. The nurse should recognize this as indicative of a loss of which vitamin?

B12.

To document ___ you would either use a table or calculate using patient's height (in meters) and weight (in kilograms) using the formula kg/m^2. After recording the results, then you will determine the range of weight they fall under.

BMI (body mass index)

Deep, aching back pain has prompted a 31-year-old woman to visit her primary care provider. The client claims that spinal movements do not exacerbate her pain and assessment reveals no deficits in range of motion. Which of the following etiologies would the clinician first suspect?

Back pain that is referred from the abdomen or pelvis.

While auscultating a client's abdomen, the nurse hears the client's stomach growling. The nurse knows that this is which type of bowel sound?

Borborygmus The nurse should find that bowel sounds are absent in a client with paralytic ileus. Paralytic ileus is a condition characterized by absence of bowel sounds, not normal bowel sounds. Hyperactive bowel sounds may be caused by diarrhea, gastroenteritis, and early bowel obstruction. Hypoactive bowel sounds may be due to surgery or late bowel obstruction. Hyperactive bowel sounds referred to as "borborygmus" may also be heard. These are the loud, prolonged gurgles characteristic of one's "stomach growling." Erratic is not a type of bowel sound.

Upon examination of an elderly client, the nurse finds hard, painless nodules over the distal interphalangeal joints. What is the appropriate term the nurse should use to document this finding in the client's medical records?

Bouchard's nodes *Heberden's nodes* Painful corns Inflamed bursa Exp: The nurse should document the hard, painless nodules over the distal interphalangeal joints as Heberden's nodes. Inflamed bursa is an inappropriate term because bursae are not found in interphalangeal joints. Bouchard's nodes are seen over the proximal interphalangeal joints. Painful corns are thickenings of the skin that occur over bony prominences and at pressure points. P552

When conducting the physical examination of a client's abdomen, the nurse auscultates 20 clicks and gurgles over 1 minute. Which of the following statements would accurately describe this finding?

Bowel sounds normal

A client presents to a primary care office with a complaint of double vision (diplopia). On questioning, the client claims to have not suffered any head injuries. Which of the following underlying conditions should the nurse most suspect in this client?

Brain tumor Double vision (diplopia) may indicate increased intracranial pressure due to injury or a tumor. Vitamin A deficiency is a cause of night blindness. Allergies are usually indicated by burning or itching pain in the eye. Viral infection is usually indicated by redness or swelling of the eye.

Functions of the nose:

Breathing, warms/moistens the air we breathe, helps w/ speech resonance, and is a sensory organ (smell)

Palpating the nose and sinuses:

Bridge, nares, frontal and maxillary sinus (firm pressure w/ no pain), nasal cavity (divided by septum and have bony turbinates in the inside)

What is the mini mental status exam?

Brief screening tool to assess cognition and change over time

Symptoms of malnutrition

Brittle hair, Swollen glands, dry patchy skin, delayed wound healing, edema in lower legs, enlarged liver

A nurse cares for a client with a stage II pressure ulcer on the right hip. The nurse anticipates finding what type of appearance to the skin over this area?

Broken with the presence of a blister

· ask the client to repeat "99" Normal- muffled "99" Abnormal- hearing "99 clearly"

Bronchophony

The nurse is auscultating the abdomen and notes a swishing sound in the abdominal area. The nurse would document this sounds as a what?

Bruit

A client expresses to the nurse that he has a "giving in" or "locking" sensation in the knee. Which test should the nurse perform to elicit related findings of a possible tear in the meniscus of the client's knee?

Bulge Ballottement *McMurray's* P556 Phalen's

Serous-filled lesion 1cm or larger, fluid filled (ex; burn)

Bulla (primary)

Paresthesia:

Burning or prickling sensation

A client visits the clinic for a routine examination. The client tells the nurse that she has become constipated because she is taking iron tablets prescribed for anemia. The nurse has instructed the client about the use of iron preparations and possible constipation. The nurse determines that the client has understood the instructions when she says: A. "I should discontinue the iron tablets and eat foods that are high in iron." B. "I should cut down on the number of iron tablets I am taking each day." C. "I can decrease the constipation if I eat foods high in fiber and drink water." D. "Constipation should decrease if I take the iron tablets with milk."

C. "I can decrease the constipation if I eat foods high in fiber and drink water." Rationale: High iron intake may lead to chronic constipation.

A client has presented for care with complaints of persistent lower back pain. When assessing the client's pain, which statement, made by the nurse, would be most appropriate? A. "Did either of your parents have back pain?" B. "Does this pain really bother you every day?" C. "What makes your pain better or worse?" D. "Heating pads usually help relieve my pain."

C. "What makes your pain better or worse?" Rationale: The nurse would assess the client's pain pattern by asking what makes a sign or symptom better or worse. A client's level of pain is subjective and the nurse would not question this level. Pain is not hereditary. The nurse would not state what makes their pain better, but rather focus on assessing the client's pain.

A nurse is interviewing a client complaining of abdominal pain for the last 2 weeks. Why is a history of the present illness vital to treating this client? A. It is only important to know when it started and the level of pain on a scale of 0-10 B. If the nurse knows where the pain is, he or she does not need a complete history of present illness C. A complete description of the present illness is essential to an accurate diagnosis D. This is essential so that the nurse can diagnose the problem before the client sees the doctor

C. A complete description of the present illness is essential to an accurate diagnosis Rationale: The nurse collects information about the present illness by beginning with open-ended questions and have the client explain symptoms. A complete description of the present illness is essential to an accurate diagnosis. Nurses do not diagnose the problem for which the client is seeking medical help; even if knowing where the pain is, it is necessary for a nurse to take a complete health history. It is important to know not only where the pain started but also the quality and intensity of the pain as well as what aggravates or alleviates the pain.

A client with peripheral vascular disease is discharged from the health care facility. Which risk-reduction teaching tip should the nurse discuss during discharge teaching? A. Limit physical activity B. Decrease dietary fiber intake C. Avoid smoking D. Eat a low-protein diet

C. Avoid smoking Rationale: The nurse should encourage the client to stop smoking, because it causes vasoconstriction (also contributes to further plaque formation), which increases the complications brought about by peripheral vascular disease. The nurse should ask the client to increase, not decrease, dietary fiber intake, and to eat a low-fat diet, not a low-protein diet. The nurse should ask the client to get regular exercise and maintain a moderate level of physical activity rather than avoid physical activity. Regular exercise improves peripheral vascular circulation and decreases stress, pulse rate and blood pressure, thereby decreasing the risk for developing peripheral vascular disease.

The nurse is assessing a 69-year-old woman's risks for lung disease. The woman states, "It shouldn't be a problem for me. My husband smokes quite heavily but I've been a lifelong nonsmoker." The nurse should recognize the need to teach the client about what topic? A. Genetic causes of lung cancer B. Age-related changes to respiratory function C. Health risks of secondhand smoke D. Strategies for making her husband quit smoking

C. Health risks of secondhand smoke Rationale: Second-hand smoke puts clients at risk for COPD (including emphysema and chronic bronchitis) or lung cancer later in life. The relationship between genetics and lung disease is not a high priority, and the husband himself must be motivated for smoking cessation. Age-related respiratory changes are not likely to be a priority in this woman's respiratory health.

A nurse is reviewing the four basic physical examination techniques and their sequence prior to receiving a new client from post-anesthetic recovery. The nurse should plan to perform which technique first? A. Auscultation B. Palpation C. Inspection D. Percussion

C. Inspection Rationale: Inspection is always done first before palpation, percussion and auscultation because the latter techniques can potentially alter the appearance of what is being inspected.

A nurse needs to position a client in the supine position for the physical examination. The nurse should ask the client to: A. Lie down, with knees bent, legs separated, and feet flat on the table B. Kneel on the table with weight of body supported by chest and knee C. Lie on the back with the legs together on the examining table D. Place the chest and abdomen on the table with the head to the side

C. Lie on the back with the legs together on the examining table Rationale: The nurse should assist the client to a supine position by instructing him or her to lie down with legs together on the examining table. To get the client into the dorsal recumbent position, the nurse instructs the client to lie down, with knees bent, legs separated and feet flat on the table. In the prone position, the lies down on his abdomen with head to the side. In the knee-chest position, the client kneels on the table with the weight of the body supported by the chest and the knees.

Which of the following findings are considered an expected change in the skin in older adults? A. Solar lentigines B. Actinic keratoses C. Loss of subcutaneous fat D. Photoaging

C. Loss of subcutaneous fat

A nurse is collecting a thorough and accurate subjective history of a client's nail problems. The client asks why this is necessary. Which of the following should the nurse mention in response? A. Abnormalities may be a sign of poor hygiene B. Local irritation can cause damage to the nail bed C. Nail problems can be caused by an underlying systemic illness D. Nail problems may affect a person's body image negatively

C. Nail problems can be caused by an underlying systemic illness Rationale: Diseases or disorders of the nails can be a local problem or they may be a sign of an underlying systemic disease that needs to be assessed. A nurse should be sensitive when interviewing a client with nail problems because they can be damaging to a person's self-image. A nurse should ask questions in a nonjudgmental manner if the client has abnormalities of the nails that are due to poor hygiene.

A nurse asks a supine client to raise his knee partially. The nurse then places the thumbs on the knee while positioning the fingers deep in the bend of the knee. The nurse is palpating the pulse of which artery? A. Femoral B. Posterior tibial C. Popliteal D. Dorsalis pedis

C. Popliteal Rationale: The femoral artery is the major supplier of blood to the legs. Its pulse can be palpated just under the inguinal ligament. This artery travels down the front of the thigh then crosses to the back of the thigh, where it is termed the popliteal artery. The popliteal pulse can be palpated behind the knee. The popliteal artery divides below the knee into anterior and posterior branches. The anterior branch descends down the top of the foot, where it becomes the dorsalis pedis artery. Its pulse can be palpated on the great toe side of the top of the foot. The posterior branch is called the posterior tibial artery.

The nurse performs the action of pinching the skin on the client's chest and pulling it straight up during the assessment of a client. What is the nurse assessing? A. Carotid pulse B. Intercostal spaces C. Skin turgor D. Lymph nodes

C. Skin turgor Rationale: Turgor refers to the skin's elasticity and how quickly the skin returns to its original shape after being pinched. Pinching the skin is not performed when assessing the carotid pulse, lymph nodes or intercostal spaces.

A client reports chest pain that occurs with exercise but subsides with rest. The nurse recognizes this as what type of data? A. Reflective B. Objective C. Subjective D. Introspective

C. Subjective Rationale: Subjective data includes the following: sensations or symptoms, feelings, perceptions, desires, preferences, beliefs, ideas, values and personal information collected from the client. This information can be elicited and verified only by the client. Introspection and reflection are not types of data collection but ways in which a nurse can assist a client to work toward changing behaviors. Objective data are obtained by the nurse through observation using the four physical assessment techniques.

A client with a 5-day history of constipation describes a sensation of "burning" in the perianal area. This information is considered which part of the assessment data? A. Physical examination B. Health history C. Subjective data D. Objective data

C. Subjective data Rationale: The client is describing a problem from their own perspective; therefore, this data is considered subjective. The objective data set comprises the client signs, or what the nurse observes in the assessment. The health history and physical exam are also part of the objective data set.

A nurse is interviewing a client regarding her lifestyle and health practices to obtain subjective information to assist in her assessment of her skin. She asks her, "Do you spend long periods of time sitting or lying in one position?" Which of the following is the best rationale for asking this question? A. To determine the client's risk for dehydration B. To determine the client's risk for skin cancer C. To determine the client's risk for pressure ulcers D. To determine the client's risk for herpes zoster

C. To determine the client's risk for pressure ulcers Rationale: Older, disabled or immobile clients who spend long periods of time in one position are at risk for pressure ulcers. Spending long periods of time sitting or lying in one position is not associated with increased risk for skin cancer, dehydration or herpes zoster.

The nurse is asking the client about the health of her parents, siblings and grandparents. This is part of the health history and is done for what reason? A. To get to know the client better B. To assess the client's quality of life C. To identify diseases for which the client may be at risk D. To establish personal rapport with the client

C. To identify diseases for which the client may be at risk Rationale: Family history is taken to identify those diseases for which the client may be at risk. Getting to know the client and establishing rapport are important but that is not why the family history is taken. Taking family history does not help to assess the client's quality of life.

A 30-year-old client arrives at the community healthcare center complaining of dizziness and a feeling of the room spinning. Based on the client's symptoms, which condition best describes what the client is most likely experiencing? A. Presbycusis B. Tinnitus C. Vertigo D. Inner ear infection

C. Vertigo Rationale: The feeling of dizziness and the room spinning that is experienced by the client is vertigo. The client is not experiencing an inner ear infection, or presbycusis, the degenerative loss in hearing, which often begins at about 60 years of age. The feeling of dizziness and the feeling of the room spinning are not the symptoms of presbycusis, an inner ear infection, or tinnitus.

Which action by a nurse is appropriate before beginning a physical examination of a client? A. Approach the client from the left side of the examination table B. Remove gloves only after examination is over C. Wash hands before examination in the examination room D. Recap used needles and place in puncture-resistant containers

C. Wash hands before examination in the examination room Rationale: The nurse should wash hands before examination room in front of the client to ensure the client that his or her safety is first priority. To avoid injury, the nurse should not recap used needles, and all disposable needles and blades should be placed in puncture-resistant containers. The nurse should always approach the client from the right-hand side of the examination table, not the left-hand side, because most examination techniques are performed with the examiner's right hand. The nurse should change gloves if they become soiled at any time during the examination and apply new pair of clean or sterile gloves.

When inspecting the mouth, the nurse focuses on lateral and vertical surfaces of the tongue and its base, because these are regions where:

Cancers often occur. It is important to inspect the sides and undersurface of the tongue and the floor of the mouth, because these are areas where cancer most often develops.

During a physical examination the nurse observes the condition shown on a client's hard palate. How should the nurse document this finding?

Candidiasis Thrush or candidiasis is a yeast infection on the palate, although it may appear elsewhere in the mouth. It is characterized by thick, white plaques that are somewhat adherent to the underlying mucosa.

A golden yellow pigment that is heavily keratinized and is found in subcutaneous fat is called what?

Carotene.

The nurse is palpating a client's neck as part of a physical assessment. Which of the following blood vessels should the nurse be especially careful to avoid bilaterally compressing during the assessment?

Carotid artery

Cultural influences on nutrition

Catholic: avoid meat on fridays during lent Hindu: avoid beef, pork, and alcohol Mormon: Avoid coffee, tea, alcohol, and tobacco Seventh Day Adventist: lacto-ovo-vegetarians Buddism: lacto-ovo- vegetarians Judaism: Kosher meat, avoid milk and meat in same meal Islam: avoid pork, fast during ramadan

Cerebral cortex:

Center of functioning for thought, memory, voluntary movement, sensory, and reasoning; damage to specific cortical areas there is a loss of function such as: motor weakness, paralysis, loss of sensation, impaired ability to understand and process language; area can be damaged by a stroke (occluded artery)

A nurse asks the client to describe the pain associated with a headache by rating the pain on a scale from 1 to 10. This subjective data should be documented in which section of the assessment?

Characteristic symptoms include having the client rate the level of pain as this provides information about the severity. This subjective information is categorized as a characteristic symptom. Information about anything else that the client may be experiencing during the headache (for example, nausea or blurred vision) should be documented in associated manifestations. Relieving factors provides information about anything that the client has attempted to relieve the symptoms. The location provides subjective information about where the headache is localized and pain radiates.

Dementia:

Chronic progressive loss of cognitive and intellectual functions, althought, perception and consciousness are intact. Characterized by disorientation, impaired judgment, memory loss. Slow onset; NOT reversible; usually caused by anatomic changes in the brain

Cachectic:

Chronic wasting disease such as cancer, dehydration, and starvation. Features include: sunken eyes, hollow cheeks, and exhausted, defeated expression

When palpating a client's abdomen, the nurse notes that the liver has a firm edge. What is the likely cause of his abnormal characteristic?

Cirrhosis

The nurse is assessing a client and notes dullness to percussion in the lowest point of the abdomen. When rolling the client to the left, the nurse notes that there is now dullness on the left side. This indicates ascites, which can be caused by

Cirrhosis and nephrosis

On examination of a client, the nurse detects a fecal odor to the breath. The nurse recognizes this finding as characteristic of what disease process?

Clients with small bowel obstructions have a fecal smell to their breath. The nurse should suspect the client of having diabetic ketoacidosis if there is a fruity smell on the breath. Clients with end-stage liver disease have a sulfur odor in their breath. Clients with respiratory infection have foul odors in their breath.

A nurse notices that a client has decreased range of motion with lateral bending of the cervical spine to the left side. What should the nurse do next in relation to this finding?

Compare this finding to the range of motion to the right side

A nurse is preparing to assess an adult client's carotid pulses. Which of the following actions would be contraindicated?

Compressing the arteries bilaterally

While the nurse examines a patient's pupillary response to light in the right eye, the pupil in the left eye is constricted. What does this finding suggest to the nurse?

Consensual reaction The consensual reaction is when the pupil constricts in the opposite eye. Myopia is impaired far vision. Presbyopia is impaired near vision often seen in middle-aged and older patients. The direct reaction is when the pupil constricts in the same eye.

A 14-year-old boy has a rash at his ankles. There is no history of exposures to ill people or environmental agents. He has a slight fever. The rash consists of small, bright red marks. When they are pressed, the red colour remains. What should the nurse do?

Consider admitting the client to the hospital. Although this rash may not be impressive, the fact that they do not blanch with pressure is concerning. This generally means that there is pinpoint bleeding under the skin; while this can be benign, it can be associated with life-threatening illnesses like meningococcemia and low platelet counts (thrombocytopenia) associated with serious blood disorders like leukemia. The nurse should always report this feature of a rash immediately.

A college student presents to the health care clinic with reports of no bowel movement for 4 days, bloating, and generalized abdominal discomfort. She states that she has not been eating and drinking correctly and is stressed because she has a final exam in 2 days. A nurse assesses the abdomen and finds positive bowel sounds in all four quadrants and tenderness in the left lower quadrant with a few small, round, firm masses. The Rovsing's sign and Psoas sign are negative. What nursing diagnosis can the nurse confirm for this client?

Constipation related to decrease in fluid intake

A college student presents to the health care clinic with reports of no bowel movement for 4 days, bloating, and generalized abdominal discomfort. She states that she has not been eating and drinking correctly and is stressed because she has a final exam in 2 days. A nurse assesses the abdomen and finds positive bowel sounds in all four quadrants and tenderness in the left lower quadrant with a few small, round, firm, masses. The Roving's sign and Psoas sign are negative. What nursing diagnosis can the nurse confirm for this client?

Constipation related to decrease in fluid intake

A nurse is inspecting the ears of an Asian client and observes that her earlobes appear soldered, or tightly attached to adjacent skin with no apparent lobe. Which of the following should the nurse do next?

Continue with the examination

Corticospinal pyramidal tract fibers:

Control voluntary movement; skilled, discrete, purposeful movement (ex. writing, typing); ~10% do NOT cross over for cross representation

A client reports that he has been experiencing diarrhea for the past week. What question by the nurse will assist in determining if this client is truly experiencing an alteration in bowel pattern? "Have you changed your food intake this week?" "How many times a day are you having a bowel movement?" "What is the consistency of your stools??" "Do you have a bowel movement every day?"

"How many times a day are you having a bowel movement?"

A client reports that he has been experiencing diarrhea for the past week. What question by the nurse will assist in determining whether this client is truly experiencing an alteration in bowel pattern?21- A client reports that he has been experiencing diarrhea for the past week. What question by the nurse will assist in determining whether this client is truly experiencing an alteration in bowel pattern?

"How many times a day are you having a bowel movement?"

A nurse observes striae on the abdomen of a middle-aged female client during the examination of the abdomen. What is an appropriate question to ask this client in regards to this finding? "Do you have high blood pressure?" "How many times have you been pregnant?" "Are you experiencing any abdominal pain?' "Have you noticed any color change to the skin?'

"How many times have you been pregnant?"

A client visits the clinic for a routine examination. The client tells the nurse that she has become constipated because she is taking iron tables prescribed for anemia. The nurse has instructed the client about the use of iron preparations and possible constipation. The nurse determines that the client has understood the instructions when she says:

"I can decrease the constipation if I eat foods high in fiber and drink water"

A 52-year-old patient with myopia calls the ophthalmology clinic very upset. Shetells the nurse, "I keep seeing semi-clear spots floating across my vision. What is wrong with me?" What would be the most appropriate response by the nurse?

"It is not an uncommon finding in people older than 40 years for this to happen. They are called 'floaters'." Floaters (translucent specks that drift across the visual field) are common in people older than 40 years of age and nearsighted patients; no additional follow-up is needed.

A student nurse is auscultating for bowels sounds on a client who returned from surgery 48 hours ago. The student tells the charge nurse that she cannot hear bowel sounds in the lower quadrants. What is the appropriate response by the charge nurse to this information?

"It takes about 3 to 5 days after surgery for the bowel sounds to return completely"

The mother of a small child with tubes in both eardrums asks the nurse if it is okay if the child travels by airplane. What is the nurse's best response?

"It's safe to fly because the tubes will equalize pressure"

A nurse inspects the gums and teeth of a client and notices a brownish tint to the teeth. What questions should the nurse ask the client to determine the cause of this finding? Select all that apply.

-"Do you drink a lot of coffee or tea?" -"How many cigarettes do you smoke daily?"

A nurse conducts a physical examination of the musculoskeletal system of a client who reports upper arm pain. Which instruction should the nurse provide the client when assessing flexion of the elbow?

-"Turn your palms down." -*"Bend your elbow."* -"With palms down, point your fingers toward the floor." -"Straighten your elbow." Exp: Asking the client to bend the elbow assesses for flexion. Asking the client to straighten the elbow assesses for extension. Asking the client turn the palms down assesses for pronation. Asking the client to turn the palms down and point fingers to the floor assesses flexion of the elbow. P548

When assessing a client's strength, it is necessary to

-*Compare one side to the other* -Assess the extremities at the same time -Assess upper and lower extremities at the same time -Compare upper and lower extremities

When the client performs straight leg flexion, the client complains of pain that radiates down his leg. The nurse understands that this may indicate what?

-*Herniated disc* -Hip fracture -Degenerative joint disease -Arthritis Exp: Straight leg flexion that produces back and leg pain radiating down the leg may indicate a herniated disc. One leg longer than the other may indicate a hip fracture. Arthritis is accompanied by pain and stiffness. Asymmetry, discomfort when touched, or crepitus during movement may occur with degenerative joint disease. P545

On inspection of the spine of a 79-year-old man, the nurse might expect to find a(n):

-*increased thoracic curve* -decreased lumbar curve -decreased cervical curve -increased cervical curve Exp: An exaggerated thoracic curve (kyphosis) is common with aging. P543

How would the nurse document normal muscle strength?

-2&2 -*5/5* -1:1 -4+ Explanation Scale for grading muscle strength: muscle strength is graded on a 0 to 5 scale: 0: No muscular contraction detected 1: A barely detectable flicker or trace of contraction 2: Active movement of the body part with gravity eliminated 3: Active movement against gravity 4: Active movement against gravity and some resistance 5: Active movement against full resistance without evident fatigue. This is normal muscle strength. P540

A nurse has just finished assessing a client's spine and neck muscles. How would the nurse document normal findings?

-All findings within normal limits -C8 and T1 spinous processes prominent. Paravertebral, sternocleidomastoid, and trapezius muscles fully developed, symmetrical, and nontender -*C7 and T1 spinous processes prominent. Paravertebral, sternocleidomastoid, and trapezius muscles fully developed, symmetrical, and nontender* P543 -Neck assessment WNL

Katz Index of ADL

-Assessment for evaluation of activities of daily living -Focus: assessment of level of independence functioning and type of assistance required in six areas of ADL: 1) bathing 2) dressing 3) toileting 4) transferring 5) continence 6) feeding -Method: evaluator observes activity performance or interviews the individual about performance -Materials: rating scale, pencil and common task objects if activity is actually performed -Scoring and interpretation: 1) evaluator rates each of the six activities as independent, some assistance required, or dependent 2) specific criteria for each rating are provided for each activity 3) the individual ratings for the six activities are converted into global letter score: A = independent in all 6 activities; B = independent in any 5 activities; C = indpendent in all but bathing and one other activity; D = independent in all but bathing, dressing and one other activity; E = independent in all but bathing, dressing, toileting, and one other activity; F = independent in all but bathing, dressing, toileting, transfers, and one other activity; G = dependent in all activities; Other = individual's functional performance cannot be classified in A-G categories -Scores: can be used to evaluate intervention outcomes and prognosis in a broad, general manner -Population: adults and elders with chronic illness

A client presents to the health care clinic with reports of onset of neck pain 3 days ago. The nurse recognizes that the most common cause of neck pain is what condition?

-Cervical disc degenerative disease -Cervical spinal cord compression -Compression fractures -*Cervical strain* P543

A client presents to the health care clinic with reports of a swollen, tender, reddened joint in the left big toe. The nurse recognizes this finding as an indication of what inflammatory process?

-Degenerative joint disease -Verruca vulgaris (warts) -*Gouty arthritis* -Rheumatoid arthritis Explanation: Tender, painful, reddened, hot, and swollen metatarsophalangeal joint in the great (big) toe is seen in gouty arthritis. This is an inflammatory condition caused by an abnormal buildup of uric acid in the body that becomes deposited in the joints. Rheumatoid arthritis can occur in any joint but usually affects the hands first. Verruca vulgaris (warts) is a painful wart that occurs under a callus. Degenerative joint disease does not typically cause the joints to be reddened and hot because it is not an inflammatory process. P557

A client waiting to be seen for a clinic appointment is observed periodically shaking the left wrist. On what should the nurse focus when assessing this client?

-Fractured wrist -Paralysis -Dupuytren contracture -*Carpal tunnel syndrome* Explanation: A motion that resembles shaking a thermometer could indicate the presence of carpal tunnel syndrome. The wrist and hand would not be mobile if the limb is paralyzed. Moving the hand and wrist would produce excruciating pain if the wrist is fractured. The inability to extend the ring and little finger is associated with a Dupuytren contracture. P552

A nurse observes the abdomen of a client and notices it to be distended below the umbilicus. The nurse recognizes that this can be caused by which of these conditions? Select all that apply.

-Full bladder -Uterine enlargement -Impacted colon -Ovarian tumor

The nurse plans to assess an adult clients kidneys for tenderness. The nurse should assess the area at the:

Costovertebral angle

The nurse notes unilateral facial drooping and reports the finding immediately to the healthcare provider. The client is diagnosed with Bell palsy. The nurse should include assessment of which affected cranial nerve in the client's head and neck assessment?

Cranial nerve VII

The nurse notes unilateral facial drooping and reports the finding immediately to the healthcare provider. The client is diagnosed with Bell palsy. The nurse should include assessment of which affected cranial nerve in the client's head and neck assessment? A. Cranial nerve V B. Cranial nerve VI C. Cranial nerve VII D. Cranial nerve VIII

Cranial nerve VII

A nurse is working with a client who has an impaired ability to move the tongue. He explains that he was in an automobile accident many years ago and suffered nerve damage that resulted in this condition. Which nerve should the nurse suspect was damaged in this client?

Cranial nerve XII (hypoglossal)

Decreased tongue strength may occur with a defect of the ____________ cranial nerve—_________________—or with a shortened frenulum that limits motion.

Cranial nerve XII (hypoglossal)

a medical term to describe the grating, crackling or popping sounds and sensations experienced under the skin and joints or a crackling sensation due to the presence of air in the subcutaneous tissue.

Crepitus

A client complains of abdominal pain with cramping diarrhea, nausea, vomiting, weight loss, and loss of energy. The nurse should suspect which of the following as the underlying cause?

Crohn's disease

A client complains of abdominal pain with cramping diarrhea, nausea, vomiting, weight loss, and loss of energy. The nurse should suspect which of the following as the underlying cause? Crohn's disease Gastric ulcer Pancreatitis Gastroesophageal reflux

Crohn's disease

An adult client is admitted to the hospital with severe diarrhea. When assessing the client, the nurse notes a round "moon" face, a buffalo hump at the nape of the neck, and a velvety discoloration around the neck. What are these signs indicative of?

Cushing's Syndrome

A nurse observes the presence of hirsutism on a female client. The nurse should perform further assessment on this client for findings associated with which disease process?

Cushing's disease Hirsutism, or facial hair on females, is a characteristic of Cushing's disease and results from an imbalance of adrenal hormones. Iron deficiency anemia is associated with spoon-shaped nails but not with excessive hair. Carcinoma of the skin causes lesions but not facial hair. Lupus erythematosus causes patchy hair loss but does not cause excessive facial hair.

What are the characteristics of normal lymph nodes? A- large, clumped, tender B- matted, fixed, tender C- non-tender, matted, fixed D- mobile, soft, nontender

D (mobile, soft, nontender)

A nurse is performing a physical examination on a patient. They are aware that the two pairs of salivary glands that are accessible to examine are the: A- parotid and thyroid B- sublingual and occipital C- submandibular and submandibular D- parotid and submandibular

D (parotid and submandibular)

A nurse is performing a physical examination. They find that the patient has swollen nodes behind the ear. Which node is this? A- submental B- pre-auricular C- jugulodigastric D- post-auricular

D (post-auricular)

A nurse is assessing a patient's pain. The nurse knows that the most reliable indicator of the patient's pain would be the: A- results of a CT scan B- physical examination C- patient's vital signs D- subjective report

D (subjective report)

A nurse is preparing to conduct a health history on a patient. Which of the following statements best describes the purpose of a health history? A- to provide a form for obtaining a patient's biographic information B- to provide an opportunity for interaction to occur between the nurse and patient C- to document normal and abnormal findings of a physical assessment D- to provide a database of subjective information about the patient's past and current health

D (to provide a database of subjective information about the patient's past and current health)

Considering the acronym OLDCART, the nurse is asking a newly admitted client questions during the assessment process. The client is a 35-year-old man who presents with pain in the upper arm since lifting weights 3 days ago. What question would be appropriate to ask that would give information for the "D" in the acronym? A. "Can you point to where the pain is located?" B. "Is there anything that makes the pain worse?" C. "Has anything helped relieve the pain?" D. "Does the pain come and go or is it constant?"

D. "Does the pain come and go or is it constant?" Rationale: The "D" in OLDCART represents the duration of the symptom. Asking if the pain comes and goes provides an answer to that question.

Temporary heart pain, resolving in less than 20 minutes, aggravated by physical activity and stress is known as what? A. Gastrointestinal B. Crushing C. Musculoskeletal D. Angina

D. Angina Rationale: Angina is temporary heart pain, resolving in less than 20 minutes. It can be aggravated by physical activity and stress, or there may be no triggers (unstable angina). This type of pain is not musculoskeletal, gastrointestinal or crushing.

The nurse observes a client crying while staring out the window. What should the nurse do first? A. Leave the client alone B. Complete a pain assessment C. Suggest the client "cheer up" D. Assess the reason for the client's emotions

D. Assess the reason for the client's emotions Rationale: The nurse needs to further assess the client to determine the reason for the sadness and crying. The client may be experiencing a physical problem that is causing moral or spiritual distress. Leaving the client alone is not appropriate since this does not provide support to the client when it is obviously needed. Suggesting that the client "cheer up" is inappropriate since the reason for the client's display of emotion is unknown. Completing a pain assessment assumes that crying is because the client is experiencing pain.

A nurse is preparing to perform a physical examination on a young man who appears anxious about the procedure. Which of the following should the nurse do to ease this client's anxiety? A. Have him urinate before the examination B. Have him undress and put on an examination gown C. Perform the genital assessment first to get it over with D. Before performing each procedure, explain what it involves and its purpose

D. Before performing each procedure, explain what it involves and its purpose Rationale: Explaining what you are doing and why helps to ease your client's anxiety. If a urine sample is not necessary, ask the client to urinate before the examination to promote an easier and more comfortable examination of the abdominal and genital areas, although it will not likely ease the client's anxiety. Begin the examination with the less intrusive procedures such as measuring the client's vital signs, height and weight. These nonthreatening/nonintrusive procedures allow the client to feel more comfortable with you and help to ease client anxiety about the examination. Having the client undress and put on an examination gown, although required, is not likely to ease his anxiety.

Upon entering the examination room, a nurse observes that the client is leaning forward with arms supporting the body weight. The nurse would most likely suspect the client is compensating for what pathophysiological disorder? A. Heart failure B. System lupus erythematosus C. Diabetes mellitus D. Chronic obstructive pulmonary disease

D. Chronic obstructive pulmonary disease Rationale: The client is assuming the tripod position which is often seen in COPD. A client with heart failure would most likely assume an orthopneic position to ease any breathing difficulties. The tripod position is usually not associated with diabetes or systemic lupus.

A client is concerned because the sclera of the right eye has been pink in color for several days and tearing. What should the nurse suspect is occurring with this client? A. Exophthalmos B. Hyphema C. Anisocoria D. Conjunctivitis

D. Conjunctivitis Rationale: Pink-colored sclera with tearing is associated with conjunctivitis which can be caused by allergies, or bacterial or viral infections. Hyphema is blood in the anterior chamber of the eye which is usually caused by blunt trauma. Anisocoria is a term used to describe pupils of unequal size. Exophthalmos is protrusion of the eye ball usually caused by a problem with the thyroid gland.

The nurse is using a goniometer while conducting the physical examination of a client's musculoskeletal status. What will the nurse use this device to measure? A. Length of extremities B. Amount of subcutaneous tissue C. Ease of ambulatory D. Degree of joint motion

D. Degree of joint motion Rationale: The goniometer is used to measure the degrees of join motion. A tape measure is used to measure extremity length. No device is used to measure the ease of ambulatory. Skinfold caliper is used to measure the amount of subcutaneous tissue.

A client complains of temporomandibular joint (TMJ) pain. What would the nurse most likely assess? A. Recent weight gain B. Knife-like pain C. History of fracture D. Difficulty chewing

D. Difficulty chewing Rationale: A client with temporomandibular joint problems may describe the jaw "getting locked" or difficulty chewing. Jaw tenderness, pain or clicking sound may be present with range of motion. Knife-like pain, history of fracture and recent weight gain are not associated with TMJ pain.

When using the PERRLA acronym, the nurse is assessing which body part(s)? A. Ears B. Nose C. Neck D. Eyes

D. Eyes Rationale: PERRLA stands for pupils equal, round and reactive to light and accommodation.

The nurse is preparing to assess a client's apical impulse. The nurse would palpate at which location? A. Third intercostal space, left axillary line B. Fourth intercostal space, left sternal border C. Second intercostal space, left sternal border D. Fifth intercostal space, left midclavicular line

D. Fifth intercostal space, left midclavicular line Rationale: The apical impulse is palpated at the fourth or fifth intercostal space at the midclavicular line.

The nurse is beginning the review of systems with a client. Which approach would ensure that all major body systems are included in this assessment? A. In a circle B. Alphabetical C. Right to left D. Head to toe

D. Head to toe Rationale: Reviewing the body systems from head to toe is one way to ensure that all areas are included. Following a circle, right to left or alphabetical are not identified as patterns to complete the review of systems.

When preparing an education session for a group of women who have been identified as postmenopausal, the nurse should include which teaching point? A. Drink two or three glasses of red wine per day B. Stop taking proton pump inhibitor medications C. Minimize weight lifting exercises D. Increase intake of vitamin D and calcium

D. Increase intake of vitamin D and calcium Rationale: Dietary intake of vitamin D and calcium promotes bone strength by increasing bone mineralization and density. Muscle strengthening exercise is encourages as it appears to maintain and possibly increase bone mass. Although moderate alcohol consumption can be beneficial in the postmenopausal years, taking more than one to two alcoholic drinks per day can promote bone loss. If a client is required to take a proton pump inhibitor, the client should not be told to take it for bone health. Instead, the client should be advised to take a calcium citrate supplement to support normal acid production leading to decreased bone loss in association with this medication.

A 50-year-old man has sought care because of the intense shoulder pain that resulted when he threw a baseball to home plate from the outfield the previous evening. The client states that he has never had problems with his shoulder previously. The nurse has asked the client to slowly abduct his affected arm to shoulder level and maintain the position. Which of the following shoulder problems does the nurse suspect? A. Bicipital tendinitis B. Adhesive capsulitis C. Calcific tendinitis D. Rotator cuff tear

D. Rotator cuff tear Rationale: A rotator cuff tear is often the result of a strong, single throwing motion and is assessed for using the drop arm test. Calcific tendinitis, adhesive capsulitis and bicipital tendinitis are degenerative diseases that typically have a more gradual onset.

The nurse is assessing a client's gait. Which finding would alert the nurse to the need for a referral for further evaluation? A. Arms swinging in opposition B. Stands on heels and toes C. Weight evenly distributed D. Shuffling of feet

D. Shuffling of feet Rationale: Shuffling of the feet suggest a problem that would most likely require a referral for further evaluation. Evenly distributed weight, ability to stand on heels and toes and arms swinging in opposition are considered normal findings.

What tool does the nurse use to auscultate the client's abdomen? A. None B. Otoscope C. Fetoscope D. Stethoscope

D. Stethoscope Rationale: The nurse uses a stethoscope to perform auscultation, in which movements of air or fluid are heard in the body over the lungs and abdomen. A fetoscope is used to hear the fetal heartbeat. An otoscope is used to view portions of the ear.

The client presents to the nurse stating that his jaws feel "stuck". What joint should the nurse assess? A. Subtalar B. Sternoclavicular C. Radioulnar D. Temporomandibular

D. Temporomandibular Rationale: The temporomandibular is where the mandible and temporal joint articulate. The sternoclavicular is at the junction of the manubrium and clavicle. The radioulnar is at the radius and ulna. The subtalar is in the foot.

When the nurse is preparing to assess the thyroid gland of a client with suspected hypothyroidism, why is it important to bring a cup of water to the physical examination? A. To prevent further dehydration B. To assist the client to feel more comfortable C. To promote the nurse-client relationship D. To observe the movement of the thyroid gland

D. To observe the movement of the thyroid gland Rationale: Although providing the client with water may help the client feel more at ease during the assessment and promote development of the nurse-client relationship, the significance of bringing a cup of water into the assessment is to help observe the movement of the gland. As the client swallows, the nurse can visualize upward movement, contours and symmetry of the thyroid. Dehydration can be a feature of hypothyroidism; however, the nurse is conducting the assessment, not providing supportive management or treatment.

A nurse is preparing to conduct an initial interview on a client from a different culture. Which of the following is important for the nurse to consider before beginning the interview? A. Specific talents of the client B. Workforce issues C. Economic status of the client D. Verbal and nonverbal communication

D. Verbal and nonverbal communication Rationale: When getting ready to conduct an initial interview with a client from a different culture, many different things must be considered. All communication is culturally based, and verbal communication can have many variables based on both language differences and usual tone of voice. Nonverbal communication is most frequently misinterpreted. Workforce issues are important but do not have to be considered in the initial interview. The client's economic status and specific talents also are good factors to consider but are not necessary information to obtain in the initial interview.

A son brings his 80-year-old father into the clinic. The son is concerned because he feels as if his father is growing weak, losing interest in things he used to care about, and no longer coming to dinner on Sundays. The nurse would know that the father is at risk for what?

Depression

Which layer of the skin contains blood vessels, nerves, sebaceous glands, lymphatic vessels, hair follicles, and sweat glands?

Dermis The second layer, the dermis, functions as support for the epidermis. The dermis contains blood vessels, nerves, sebaceous glands, lymphatic vessels, hair follicles, and sweat glands, which support the nutritional needs of the epidermis and provide support for its protective function. the top layer of the skin is the dermis layer outermost skin layer, and serves as the body's first line of defense against pathogens, chemical irritants, and moisture loss. The subcutaneous layer provides insulation, storage of caloric reserves, and cushioning against external forces. Composed mainly of fat and loose connective tissue, it also contributes to the skin's mobility. The connective layer is a distracter to the question.

Vitamin B/folate

Dietary sources are leafy greens, lentils, seeds, liver, orange juice, grains, cereal. Groups at risk: acoholics, older adults, those that follow popular diets and people of low socioeconomic status

The throat is dull red, and a gray exudate is present on the uvula, pharynx, and tongue.

Diphtheria

The terms "generalized," "exposed surfaces," "upper arm," and "skin folds" are used to describe which major characteristic of skin lesions?

Distribution The given terms denote anatomic location, or distribution, of skin lesions over the body.

The nurse is caring for a female client during her first postoperative day after a temporary colostomy. The client refuses to look at the colostomy bag or the area. A priority nursing diagnosis for this client is:

Disturbed body image related to temporary colostomy

Sternocleoidmastoid muscle

Divides the neck into anterior and posterior triangles

A nurse has performed a head and neck assessment of an adult patient and noted that the thyroid gland is not palpable. What is the nurse's most appropriate action? Document this as an expected assessment finding Refer the patient to the primary care provider promptly Perform a focused endocrine assessment Position the patient supine and reattempt palpation

Document this as an expected assessment finding

The nurse selects the chart shown here to assess a client's vision. Which client characteristic caused the nurse to select this chart? (Snellen E Chart)

Does not speak English The Snellen E chart can be used for clients who do not speak English. This chart is not used for clients being treated for glaucoma, color blindness, or recovering from cataract surgery.

The nurse is beginning the examination of the skin of a 25-year-old teacher. She previously visited the office for evaluation of fatigue, weight gain, and hair loss. The previous clinician had a strong suspicion that the client has hypothyroidism. What is the expected moisture and texture of the skin of a client with hypothyroidism?

Dry and rough A client with hypothyroidism is expected to have dry and rough skin. This is a good example of how the skin can give clues to systemic diseases.

Aging adult ears, nose, throat:

Ear: cilia become coarse and stiff; noise pollution; nerve degeneration, increased auditory rxn time Nose & throat: nose hairs grow stiff and coarse, decreased sense of smell, loss of taste buds, decrease salivation/sense of taste

You are caring for a patient with hypertension. What dietary change would be appropriate to recommend?

Eat more spinach

Also known as atopic dermatitis, is characterized by itchy, pink macular or papular lesions, commonly located on flexural areas such as the inner elbows or posterior knees. Can occur anywhere on the body.

Eczema

Teenagers doing community service following arrest for driving under the influence are working at the rehabilitation hospital with clients who have paraplegia. These clients have been paralyzed by drunk drivers. How would the nurses who care for these clients best use the time spent with these teenagers?

Educating them about not drinking and driving

Pustules:

Elevated lesions that contain pus (infectious material)

The apocrine glands are stimulated by what?

Emotional stress Explanation:The eccrine glands are widely distributed, open directly onto the skin surface, and by their sweat production help to control body temperature. In contrast, the apocrine glands are found chiefly in the axillary and genital regions, usually open into hair follicles, and are stimulated by emotional stress.

The nurse is discharging an adult client who received 18 staples for a head laceration received while mountain biking. What can the nurse focus on while doing discharge teaching?

Encourage the use of safety equipment

The nurse is discharging an adult client who received 18 staples for a head laceration received while mountain biking. What can the nurse focus on while doing discharge teaching? A. Encourage the use of safety equipment B. Encourage proper nutrition to promote healing C. Encourage the client to take a safety course D. Teach proper posture, bending, and lifting

Encourage the use of safety equipment

A nurse is educating a client about the function of the parts of the auditory system. Which is the function of the eustachian tube

Equalizes the pressure in the middle ear with atmospheric pressure

A nurse is educating a client about the function of the parts of the auditory system. Which is the function of the eustachian tube?

Equalizes the pressure in the middle ear with atmospheric pressure

A 46-year-old former salesman presents to the ER complaining of black stools for the past few weeks. His past medical history is significant for cirrhosis. He has gained weight recently, especially around his abdomen. He has smoked two packs of cigarettes a day for 30 years and has drunk approximately 10 alcoholic beverages a day for 25 years. He has used IV heroin and smoked crack in the past. He denies any recent use. He is currently unemployed and has never been married. Examination shows a man appearing older than his stated age. His skin has a yellowish tint and he is thin with a prominent abdomen. Multiple "spider angiomas" are at the base of his neck. Otherwise his heart and lung examinations are normal. On inspection he has dilated veins around his umbilicus. Increased bowel sounds are heard during auscultation. Palpation reveals diffuse tenderness that is more severe in the epigastric area. His liver is small and hard to palpation and he has a positive fluid wave. He is positive for occult blood on his rectal examination. What cause of black stools most likely describes his symptoms and signs?

Esophageal varices

The nursing student hopefuls are taking a pre-nursing anatomy and physiology class. What will they learn is the anatomical feature that equalizes air pressure in the middle ear?

Eustachian tube

Inspect the eyes for: (cont.)

Eversion of upper lid. lacrimal apparatus (swelling, eye crusting, tearing), cornea and lens (shine light, clear, no opacities), pupil (size, shape, symmetry)

Macules:

Ex. freckles; flat, change in the color of the skin; if > 1 inch = patch, Ex. cafe au lait spot;

Acromegaly:

Excessive secretion of growth hormone from the pituitary gland after puberty creates an enlarged skull and thickened cranial bones. Elongated head, massive face, over-growth of nose and lower jaw, heavy eyebrow ridge, and coarse facial features

What is a characteristic symptom of Graves hyperthyroidism?

Exophthalmos In exophthalmos the eyeball protrudes forward. When bilateral, it suggests the infiltrative ophthalmopathy of Graves hyperthyroidism.

A 29-year-old physical therapist presents for evaluation of an eyelid problem. On observation, the right eyeball appears to be protruding forward. Based on this description, what is the most likely diagnosis?

Exophthalmos In exophthalmos, the eyeball protrudes forward. If it is bilateral, it suggests the presence of Graves' disease, although unilateral exophthalmos could still be caused by Graves' disease. Alternative causes include a tumour and inflammation in the orbit.

The nurse is preparing to examine a client's skin. What would the nurse do next?

Expose only the body part that is being examined.

Inspect the nose, mouth, throat for:

External nose, nasal cavity, lips, teeth and gums, tongue, buccal mucosa, palate and uvula, and pharyngeal wall and tonsils

Sensory testing:

Eyes usually closed; sharp vs. dull, light touch, proprioception, tactile discrimination (two point discrimination)

Inspect/palpate the face for:

Facial expression, involuntary movements, and shape and symmetry of facial structures (eyebrows, sides of mouth)/TMJ

Primary headaches are more worrisome than secondary headaches.

False

The frontal sinuses are the only ones readily accessible to clinical examination.

False

When examining the skin, you should NOT thoroughly inspect skin folds such as under large breasts, the abdomen, and the groin. True or False?

False

When assessing risk of colon cancer, which of the following health-history components should the nurse prioritize?

Family history; dietary habits

A client exhibits purulent drainage in the right external ear canal. The client complains of pain that increases when the ear is touched. Which client teaching instructions should the nurse provide?

Finish the entire course of the antibiotic therapy

The nurse is preparing to examine the ears of an adult client with an otoscope. The nurse should plan to

Firmly pull the auricle out, up and back

Linear cracks in the skin that may extend to the dermis and may be painful. Examples include chapped lips or hands and athlete's foot.

Fissures

A nurse performs palpation of a client's lymph nodes. Which finding should be reported to the health care provider? A. Mobile from side to side B. Soft consistency C. Fixed to underlying tissue D. Round and 8mm in size

Fixed to underlying tissue

A nurse performs palpation of a client's lymph nodes. Which finding should be reported to the health care provider? A. Mobile from side to side B. Soft in consistency C. Fixed to underlying tissue D. Round and 8 mm in size

Fixed to underlying tissue

Which action by the nurse will facilitate relaxation of the abdominal muscles during examination of the abdomen?

Flex the client's legs by placing a pillow under the knees

What range of motion is the nurse testing by asking a client to stoop to pick an object off the floor?

Flexion

The nurse notes that a client's abdominal skin is pale and taut. What should the nurse suspect is causing this finding?

Fluid accumulating in the abdominal cavity

On inspection of the abdomen, a nurse notes that the client's skin appears pale and taut. The nurse recognizes that this finding is most likely due to what process occurring within the abdominal cavity?

Fluid accumulation

A nurse at a long-term care facility is completing the nutrition assessment of a man who has just moved to the facility. After determining the client's venous filling and emptying each take approximately 10 seconds, the nurse would perform further assessments related to what health problem?

Fluid volume deficit

A nurse assesses the parallel alignment of a client's eyes by testing the corneal light reflex. Where should the nurse shine the penlight to obtain an accurate result?

Focused on the bridge of the nose When testing the corneal light reflex, the nurse should shine the light toward the bridge of the nose. At the same time, the client is instructed to stare straight ahead. This facilitates a parallel image on the cornea. The eye response upon shining the light toward the eye may interfere with the assessment. The light should not be shined toward the forehead or on an object on the wall.

In order to effectively assess the oral mucosa, the nurse should have which assessment tools available?

For the assessment of the oral mucosa, the nurse needs to have available gloves to prevent the possible transmission of infection, a penlight to optimize visibility of the oral cavity, and a tongue depressor to prevent the tongue from obstructing view of the posterior oral cavity and throat.

A client has been receiving intravenous antibiotics for several weeks. Which prevention strategy would be best for the nurse to recommend for this client?

Formal hearing test

The tongue is attached to the hyoid bone and styloid process of the temporal bone and is connected to the floor of the mouth by the

Frenulum The tongue is a mass of muscle, attached to the hyoid bone and styloid process of the temporal bone. It is connected to the floor of the mouth by a fold of tissue called the frenulum.

An adult client has been diagnosed with carpal tunnel syndrome. What type of working conditions may have contributed to this diagnosis?

Frequent Repetitive Movements

Skull bones:

Frontal, parietal (2), occipital, and temporal (2)

Name the 4 lobes of the cerebrum:

Frontal, parietal, temporal, and occipital

Stage III pressure ulcer:

Full-thickness extending into sub-q resembling a crater. Might not see muscle, bone, or tendon.

what is a frequent heart burn, dry cough, asthma symptoms or acid indigestion behind the breast bone that is worst with supine position

GERD -gastroesphageal disease

Inspect the fundus (of the eye) for:

General background; macula; optic disc color, shape, margins; number, color, A:V ratio, pulsations, tortuosity of retinal vessels

During an assessment the nurse observes the condition shown in the client's mouth. What should the nurse suspect is occurring with this client? Think overgrowth of gum tissue!

Gingival hyperplasia. This is gingival hyperplasia, or an overgrowth of gum tissue. It is seen in pregnancy, puberty, leukemia, and medications such as phenytoin. Gum tissue would not be overgrown if dentures are worn. This is not periodontal disease or evidence of significant dental caries.

An adult client tells the nurse that her peripheral vision is not what it used to be and she has a blind spot in her left eye. The nurse should refer the client for evaluation of possible

Glaucoma A scotoma is a blind spot that is surrounded by either normal or slightly diminished peripheral vision. It may be from glaucoma.

A client reports the appearance of rings around lights. A nurse should perform further assessment to confirm the onset of what disorder?

Glaucoma Seeing rings around lights or halos is associated with narrow angle glaucoma. Diabetes produces change in the retina that can cause blurred vision. Cataracts are caused by clouding of the lens of the eyes. Hypertension affects the blood vessels of the eyes which may not cause any eye symptoms until the damage is severe.

When documenting the findings from a physical examination of the head and neck, what will the nurse include when describing the client's head? A. Sclera color B. Hair color C. Nasal mucosa color D. Facial skin color

Hair Color

When inspecting the tympanic membrane, which of the following structures does the nurse expect to identify? a) Pars tensa, pars flaccida, vestibule, cone of light b) Handle of malleus, short process of malleus, cone of light c) Pars tensa, umbo, handle of malleus, ossicles d) Cone of light, incus, umbo, cochlea

Handle of malleus, short process of malleus, cone of light Explanation: Visualization of the tympanic membrane using an otoscope includes inspection of the cone of light, the short process of the malleus, and the handle of the maleus. The cochlea, vestibule, and stapes (part of the ossicles) are not normally visualizable

The nurse is preparing to perform a physical examination of a client who is an Orthodox Jew. Which of the following accommodations should the nurse be prepared to make for this client, based on his religious beliefs?

Have a nurse who is the same sex as the client examine him Clients from conservative religious groups (e.g., Orthodox Jews or Muslims) may require that the nurse be the same sex as the client. The client must still undress and put on an examination gown. It is not likely that the client will want to pray before the examination, and it is not necessary to avoid asking questions regarding his lifestyle.

A nurse begins the eye examination on a client who presents to the health care clinic for a routine examination. What is the correct action by the nurse to perform the test for near visual acuity?

Have the client hold the Jaeger card 14 inches from the face and read with one eye at a time Near vision is tested with a Jaeger card, Snellen card, or comparable card), held 14 inches from the face. Have the client cover one eye with an opaque card before reading from top to bottom. Sitting the client in front of the examiner, extending one arm, and slowly move one finger upward until it is seen by both the client and the examiner is a test for gross peripheral vision. If the client wears glasses, they should be left on for the test. Placing the client 20 feet from the chart and record the smallest line the client can read is the test for distant acuity.

Migraines:

Headache of genetically transmitted vascular origin; headache plus prodrome, aura, other symptoms (?????????)

A client presents to the health care clinic with reports of a 12-pound unintentional weight loss despite being hungry all the time, profuse sweating, and swelling around the anterior neck area. The client states that she does not have insurance and cannot afford to see a regular health care provider. What nursing diagnosis can the nurse confirm from this data?

Health seeking behaviors

A female client visits the clinic and tells the nurse that she wants to "stay healthy." The nurse observes that the client has diffuse neck enlargement, is perspiring, and is quite fidgety. The client tells the nurse that she is "hungry all the time, but I have lost weight." A priority nursing diagnosis for the client is

Health-seeking behaviors related to verbalization of wanting to stay healthy and concerns over weight loss.

a solid swelling of clotted blood within the tissues. localized bleeding outside of blood vessels, due to either disease or trauma including injury or surgery and may involve blood continuing to seep from broken capillaries

Hematoma

A patient presents to the clinic with "sores around the mouth." The nurse notes vesicular lesions on the upper lip and right corner of the lips. The patient describes these as painful. The nurse suspects what condition?

Herpes simplex.

A nurse is working with an older client who has had diarrhea for the past week and is dehydrated. The nurse understands that older clients are especially at risk for potential complications with diarrhea due to which of the following factors? Decreased sensitivity to pain Large numbers of medications taken Tendency to have an inadequate fluid intake Higher fat-to-lean muscle ratio

Higher fat-to-lean muscle ratio Older adult clients are especially at risk for potential complications with diarrhea, such as fluid volume deficit, dehydration, electrolyte, and acid-base imbalances, because they have a higher fat-to-lean muscle ratio. because they are fat!

A nurse is working with an older client who has had diarrhea for the pas t week and is dehydrated. The nurse understands that older clients are especially at risk for potential complications with diarrhea due to which of the following factors?

Hight fat-to-lean muscle ratio

You are caring for a patient in the outpatient clinic with suspicion of cancer due to recent weight losses for unidentifiable reasons. The patient a 25-year history of smoking. You perform an assessment and ask the patient about symptoms related to laryngeal cancer. What is an early symptom associated with laryngeal cancer?

Hoarseness is an early symptom of laryngeal cancer. Dyspnea, dysphagia, and lumps are later signs of laryngeal cancer.

Which of the following is a priority patient teaching topic related to the ears?

How to prevent skin cancer

A staff educator from the hospital is providing an event for the hospital staff. The educator is talking about health promotion activities for people with diseases of the nose, mouth, throat, and sinuses. What would the educator include in the presentation?

How to reduce periodontal disease Major risk reduction and health promotion goals in assessment of the nose, sinuses, mouth, and throat are related to various issues, including tobacco use, obstructive sleep apnea, oral health, and cancer. Health goals include reducing periodontal disease.

Visual fields:

Humans have binocular vision; what the right/left eye see overlaps; blind spot in each eye because the optic disc doesn't have the necessary elements to see

What structure is found midline in the tracheal area just beneath the mandible? A. Cricoid cartilage B. Hyoid bone C. Thyroid cartilage D. Adam's apple

Hyoid bone

When identifying the midline structures of the neck from the mandible to the sternal notch, the nurse notes the structures in what order? A. Cricoid cartilage, hyoid bone, tracheal rings, thyroid isthmus B. Thyroid cartilage, thyroid isthmus, cricoid cartilage, hyoid bone C. Hyoid bone, thyroid cartilage, cricoid cartilage, isthmus of the thyroid D. Hyoid bone, tracheal rings, cricoid cartilage, lobes of the thyroid gland

Hyoid bone, thyroid cartilage, cricoid cartilage, isthmus of the thyroid

during an auscultation what is a high pitched, rapid motility, and bowel obstruction indicate

Hyperactive bowel sounds

Hyper/hypo reflexia:

Hyperreflexia: exaggerated reflex seen when the monosynaptic reflex arc is released from the usually inhibiting influence of higher cortical levels; occurs with upper motor neuron lesions (ex. stroke) Hyporeflexia: absence of a reflex, is a lower motor neuron problem. Occurs with interruption of sensory afferents or destruction of motor efferents and anterior horn cells (ex. spinal cord injury)

The nurse does a health history. The patient states he has lost 30 pounds in the last couple months without really trying. The patient also states he feels warm all the time and sometimes feels like he has heart palpitations. The nurse would anticipate orders to evaluate the patient fo

Hyperthyroidism

The nurses assesses the thyroid gland of a client with recent weight loss. On auscultation, a low, soft, rushing sound is heard over the lateral lobes. Which condition is most likely?

Hyperthyroidism

A client has an edematous face, hands, and legs. Which health problem should the nurse suspect this client is experiencing?

Hypothyroidism

A client presents at the clinic for a routine check-up. The nurse notes that she is dressed in warm clothing even though the temperature outside is 73°F. The nurse also notes that the patient has gained 10 pounds since her last visit 9 months ago. What might the nurse suspect?

Hypothyroidism

A client presents at the clinic for a routine check-up. The nurse notes that she is dressed in warm clothing even though the temperature outside is 73°F. The nurse also notes that the patient has gained 10 pounds since her last visit 9 months ago. What might the nurse suspect? A. Effects of age-related changes B. Brain tumor C. Hyperthyroidism D. Hypothyroidism

Hypothyroidism

Hoarseness lasting longer than two weeks accompanied by the additional reported symptoms of fatigue and weight gain suggest hypothyroidism.

Hypothyroidism

A client presents to the health care clinic with reports of new onset of generalized hair loss for the past 2 months. The client denies the use of any new shampoos or other hair care products and claims not to be taking any new medications. The nurse should ask the client questions related to the onset of which disease process?

Hypothyroidism Generalized hair loss can be a finding in hypothyroidism. (None of the other conditions listed is associated with generalized hair loss. Diabetes is a problem with glucose regulation. Crohns disease is an inflammatory process in the large intestines. Liver disease results in many problems with fluid regulation, metabolism of drugs, and storage of glucose.)

A client diagnosed with goiter has undergone a thyroidectomy. Which statement from the client indicates understanding of post-operative care teaching? A. I must take thyroid hormone replacement medication for the rest of my life. B. I will complete the entire course of thyroid hormone replacement over six weeks. C. I must keep my follow up appointments to receive my thyroid hormone injections. D. I will take my thyroid hormone replacement medication once every week.

I must take thyroid hormone replacement medication for the rest of my life.

The nurse is developing a plan of care for a client found to have a strength problem. What would be an appropriate nursing diagnosis for this client?

Impaired walking Self-care deficit Activity intolerance *Impaired physical mobility* Exp: The most appropriate diagnosis would be impaired physical mobility related to reduced strength and ROM. Activity intolerance and self-care deficit would not correctly identify the situation at hand. Impaired walking is incorrect. P560

The client is having a Weber test. During a Weber test, where should the tuning fork be placed?

In the midline of the client's skull or in the center of the forehead

The nurse assesses a client's submental lymph nodes. In which area of the client's head should the nurse palpate these lymph nodes?

In the sideline, a few centimeters behind the tip of the mandible

Glaucoma:

Increase in intraoccular pressure which decreases peripheral vision

While assessing the ears of an adult client, the nurse observes that the tympanic membrane is completely immobile. The nurse should further assess the client for signs and symptoms of

Infection

Paronychia:

Infection along the side of fingernails, usually from picking at the nail

Tinea Veriscolor:

Infection from a fungus; with light skinned individuals, the spots are dark; with darker skinned individuals, the spots are lighter

Otitis externa:

Infection of the more outer ear canal; severe swelling of canal, inflammation, and tenderness

The nurse as elicited a positive Murphy sign. The knows this is indicates what?

Inflammation of the gallbladder

what is peritonitis?

Inflammation of the membrane lining the abdominal wall and covering the abdominal organs.

A client tells the nurse about a raised lesion on the client's leg. What is the nurse's first nursing action?

Inspect the area If the client has a specific concern about the skin, the nurse should inspect the area/lesion first and ask other questions second. It would not be appropriate to ask further questions, document the statement, or move on to the next body system until the lesion has been inspected.

When performing the abdominal assessment for a client, which assessment technique should the nurse perform first?

Inspection

What is the order of a Health Assessment:

Inspection, palpation, percussion, auscultation (exception w/ abdomen = IAPaPe)

Stage I pressure ulcer:

Intact skin appears red, but unbroken. Skin does NOT blanch

A client presents to the emergency department with reports of new onset of abdominal pain for the past 3 days. The client states there is also a pulling feeling on the right side. Upon examination, the nurse notices a 5-cm transverse scar in the right lower quadrant. The nurse recognizes that this client may be experiencing what type of process?

Internal adhesions from previous surgery

A client presents to the emergency department with reports of new onset of abdominal pain for the past three (3) days. The client states there is also a pulling feeling on the right side. Upon examination the nurse notices a 5cm transverse scar in the right lower quadrant. The nurse recognizes that this client may be experiencing what type of process?

Internal adhesions from previous surgery

A client presents to the emergency department with reports of new onset of abdominal pain for the past three (3) days. The client states there is also a pulling feeling on the right side. Upon examination the nurse notices a 5cm transverse scar in the right lower quadrant. The nurse recognizes that this client may be experiencing what type of process? Intestinal obstruction at the sigmoid colon Internal adhesions from previous surgery Peritonitis from a ruptured diverticulum Acute onset of appendicitis with possible rupture

Internal adhesions from previous surgery

The nurse is assessing the abdomen of an adult client and observes a purple discoloration at the flanks. The nurse should refer the client to a physician for possible

Internal bleeding

While assessing the abdominal sounds of an adult client, the nurse hears high pitched tingling sounds throughout the distended abdomen. The nurse should refer the client to a physician for possible:

Intestinal obstruction

Stage IV pressure ulcer:

Involves all skin layers and extending into support tissue. Exposes muscle, tendon, or bone, and may slough (stringy matter in wound bed) OR eschar (black/brown necrotic tissue)

Associated with spoon-shaped nails

Iron deficiency anemia

A 77-year-old retired bus driver presents at his wife's request to the clinic for a physical examination. He has recently been losing weight and has felt very fatigued. He has had no chest pain, shortness of breath, nausea, vomiting, or fever. His past medical history includes colon cancer, for which he had surgery, and arthritis. He has been married for more than 40 years. He denies any tobacco or drug use and has not drunk alcohol since getting married. His parents both died of cancer in their 60s. On examination his vital signs are in expected ranges. His head, cardiac, and pulmonary examinations are unremarkable. Abdominal examination reveals normal bowel sounds. Results of palpation of the liver are abnormal. His rectal examination is positive for occult blood. What further abnormality of the liver was likely found on examination?

Irregular, large liver

A 21-year-old receptionist comes to the clinic reporting frequent diarrhea. She states that the stools are very loose and there is some cramping beforehand. She states this has occurred on and off since she was in high school. She denies any nausea, vomiting, or blood in her stool. Occasionally she has periods of constipation but that is rare. She thinks the diarrhea is much worse when she is nervous. Her past medical history is not significant. She is single and a university student majoring in accounting. She smokes when she drinks alcohol but denies any illegal drugs. Both of her parents are healthy. Her entire physical examination is unremarkable. What cause of diarrhea is the most likely etiology?

Irritable bowel syndrome Irritable bowel syndrome will cause loose bowel movements with cramps, but no systemic symptoms of fever, weight loss, or malaise. This syndrome is more likely found in young women with alternating symptoms of loose stools and constipation. Stress usually makes the symptoms worse as well as certain foods.

A nurse is preparing a client for a physical examination of his skin, hair, and nails. Which of the following interventions should the nurse implement? Select all that apply.

Keep the room door closed Wear gloves when palpating lesions Use sunlight, if possible, to inspect the skin Have the client remove his toupee To prepare for the skin, hair, and nail examination, ask the client to remove all clothing and jewelry and put on an examination gown. In addition, ask the client to remove nail enamel, artificial nails, wigs, toupees, or hairpieces as appropriate. The client may remain in a sitting position for most of the examination. If available, sunlight is best for inspecting the skin. Wear gloves when palpating any lesions because you may be exposed to drainage. Keep the room door closed or the bed curtain drawn to provide privacy as necessary.

A nurse is providing client teaching to the parents of a preschooler who experiences chronic epistaxis. What would the nurse identify as the area where most nosebleeds originate?

Kiesselbach's plexus The anterior portion of the nasal septum has a rich vascular supply known as Kiesselbach's plexus. It is often the site of origination for nosebleeds. The other options are distracters for the question.

A client frequently experiences dry, irritated eyes. These findings are consistent with a problem in what part of the eye?

Lacrimal apparatus The lacrimal apparatus (which consists of the lacrimal gland, punctum, lacrimal sac, and nasolacrimal duct) protects and lubricates the cornea and conjunctiva by producing and draining tears.

How does the image of a Schizophrenic differ from those w/out?

Larger ventricles

A nurse tells a client that the next step in the musculoskeletal assessment is to perform range of motion of the thoracic and lumbar spine. The nurse should demonstrate which movements for the client to facilitate the examination? SELECT ALL THAT APPLY A Circumduction B Extension C Lateral bending D Rotation E Flexion

Lateral bending Rotation Flexion

a sign of volume overload, displaced laterally and found over a wider area

Laterally Displaced Apical Impulse

The brain has "crossed representation" meaning...

Left cerebral cortex is getting sensory info from and controls motor functions on the right side; vice versa w/ right cerebral cortex

To palpate for tenderness of an adult clients appendix, the nurse should begin the abdominal assessment at the clients

Left lower quadrant

A nurse should assist an elderly client to assume which position to facilitate the examination of the anus and rectum?

Left side-lying

To palpate the spleen of an adult client, the nurse should begin the abdominal assessment of the client at the:

Left upper quadrant

Zosteriform:

Lesion that follows a dermatome; usually unilateral; ex. shingles

Gyrate:

Lesion that is snake-like/twisted

Clustered/grouped:

Lesions are near each other on a body part; but not confluent

During an oral assessment, the nurse identifies that client has white patches in his mouth. How would this be documented in the medical record?

Leukoplakia is white patches inside of the mouth. Gingivitis is inflamed gums that bleed. Small red spots occur with petechiae. Fordyce granules are while or yellow papules appearing on the cheeks, tongue and lips.

Anatomy of the lacrimal apparatus:

Lies w/in the bony orbit; tears keep the eye moist and clear of microbes; glands create the tears, wash across the eye, and down into the sinuses

Fundus of the eye:

Lighter circle on left is the optic disc; fovea is the dark spot whic is surrounded by the macula

A nurse is inspecting the abdomen of a young, fit client who has well-defined abdominal muscles. The nurse recognizes the vertical line that appears in the center of the client's abdomen as which of the following?

Linea alba

Burrows:

Linear lesions caused by infestation of mites that makes tunnels under the skin; Ex. scabies or other larvae under the skin

A client is admitted to a health care facility with new onset of abdominal pain, fatigue, and low back pain. The client relates a 10-year history of high blood pressure. When auscultating the client's abdomen for bowel sounds, what other assessment should the nurse perform at this time?

Listen with the bell of the stethoscope for vascular sounds

A nurse performs percussion by placing the left hand flat against the client's lower rib cage and striking it with the ulnar side of the right fist. The client reports tenderness. The nurse recognizes this as an abnormal finding for which organ?

Liver Percussion for liver tenderness is elicited by placing the left hand flat against the lower rib cage and striking it with the ulnar side of the right fist. The costovertebral angles are located at the twelfth rib posteriorly. Tenderness of the costovertebral angles indicates a kidney problem such as infection (pyelonephritis), renal calculi, or hydronephrosis. Percussion of the spleen begins in the left midaxillary line and progresses downward until the sound changes from lung resonance to splenic dullness. The gall bladder is not percussed.

The nurse performs the assessment technique shown. What is the nurse assessing in this client?

Liver size

Wernicke's Aphasia:

Located in the upper temporal lobe; wenicke's region controls language comprehension; when damaged, patient hears the sound, but has no meaning to them; similar to hearing a new language

what element of COLDSPA asks "does it radiate/spread or stay local"

Location (COLDSPA)

During your physical examination of the patient you note an enlarged tender tonsillar lymph node. What would you do?

Look for a source such as infection in the area that it drains

During your physical examination of the patient you note an enlarged tender tonsillar lymph node. What would you do? A. Assess for meningitis B. Look for involvement of other regions of the body C. Look for a source such as infection in the area that it drains D. Assess for dietary changes

Look for a source such as infection in the area that it drains

Vitiligo:

Loss of pigmentation of the skin; get white spots; more noticeable in darker skin individuals

Entropion/ectropion:

Lower lid rolls in because of a spasm that led to scar tissue/lower lid is loose and rolls out

Bell's Palsy:

Lower motor neuron lesion, producing rapid onset of cranial nerve VII paralysis of facial muscles; almost always unilateral. May be reactivation of herpes simplex virus latent since childhood. Usually present with smooth forehead, wide palpebral fissure, drooling and pain behind the ear.

The nurse feels a small mass in the neck of a client. It is mobile in both the up-and-down and side-to-side directions. Which of the following is the nurse most likely feeling?

Lymph node

Which type of vessels filter pathogens from the body and drain the fluid that has moved outside of the circulation back into the vessels?

Lymphatic

A client visits the clinic for a routine physical examination. The nurse prepares to assess the client's skin. The nurse asks the client if there is a family history of skin cancer and should explain to the client that there is a genetic component with skin cancer, especially?

Malignant melanoma

An older adult client who wears dentures reports having soreness of the gums. Which intervention should the nurse recommend to the client to alleviate this problem?

Massage the gums daily. Regular massage of the gums relieves soreness and pressure from dentures on the underlying soft tissue. Avoiding excessive intake of sugary foods is a strategy to reduce the risk of cariogenic bacteria. The client with dentures will not use toothpaste but rather a cleanser specific for dentures. The client with dentures should have a dental examination every year; however, this intervention will not provide immediate relief from the gum soreness.

An older patient who wears dentures complains of having sore gums. What can the nurse instruct the patient to help with this problem?

Massage the gums every day.

While assessing an adult client's abdomen, the nurse observes that the client's umbilicus is deviated to the left. The nurse should refer the client to a physician for possible

Masses

While assessing an adult clients abdomen, the nurse observes that the clients umbilicus is deviated to the left. The nurse should refer the client to a physician for possible:

Masses

Older adults

May add additional sugar leading to diabetes, hypertension, and heart disease. Malnutrition and dehydration are common due limited food options and reduced sense of thirst.

A nurse cares for a client with duodenal ulcer. The nurse knows that which characteristic of pain is generally associated with the client's condition?

May awaken the client at night

Southeast Asian practices

May involve coining and cupping, results in bruising and abrasions that is often mistaken as abuse.

A nurse cares for a client with a distended abdomen due to peritonitis. Which parameter should the nurse measure to assess improvement?

Measure abdominal girth

A nurse receives an order to measure the abdominal girth daily on a client admitted with ascites. How should the nurse best implement this order?

Measure at the same time each day, ideally in the morning after voiding

A nurse notices that a client's flexibility of the right elbow is less than the left elbow. What is an appropriate action by the nurse in regard to this finding?

Measure movement with a goniometer.

A client presents to the emergency department with reports of neck pain and a sudden onset of a headache. Upon examination, the nurse finds that the client has an increased temperature and nuchal rigidity. The nurse recognizes these findings as most likely to be caused by what condition?

Meningeal Inflammation

A client presents to the emergency department with reports of neck pain and a sudden onset of a headache. Upon examination, the nurse finds that the client has an increased temperature and neck stiffness. The nurse recognizes these findings as most likely to be caused by what condition? A. Migraine headache B. Meningeal inflammation C. Trigeminal neuralgia D. Parkinson's disease

Meningeal inflammation

A client presents to the emergency department with reports of neck pain and a sudden onset of a headache. Upon examination, the nurse finds that the client has an increased temperature and neck stiffness. The nurse recognizes these findings as most likely to be caused by what condition?

Meningeal inflammation is a likely cause of this condition, which manifests as sudden headache, neck pain with stiffness, and fever. Migraine headaches are accompanied by nausea, vomiting, and sensitivity to noise or light, not by fever and neck stiffness. Trigeminal neuralgia is manifested by sharp, shooting, piercing facial pains that last from seconds to minutes. Parkinson's disease is not manifested by headache and neck pain.

A nurse is caring for a patient admitted with neck pain. The patient is febrile. What is the most likely medical diagnosis for this patient?

Meningitis

A 38-year-old accountant comes to the clinic for evaluation of a headache. The throbbing sensation is located in the right temporal region, and is an 8 on a pain scale of 1 to 10. It started a few hours ago, and she has noted nausea with sensitivity to light; she has had headaches like this in the past, usually less than one per week, but not as severe. She does not know of any inciting factors. There has been no change in the frequency of her headaches. She usually takes an over-the-counter analgesic, which results in resolution of the headache. Based on this description, what is the most likely diagnosis of the type of headache?

Migraine

A female client visits the clinic and tells the nurse that she frequently experiences severe recurring headaches that sometimes last for several days and are accompanied by nausea and vomiting. The nurse determines that the type of headache the client is describing is a

Migraine headache

A client reports right-sided temporal headache accompanied by nausea and vomiting. A nurse recognizes that which condition is likely to produce these symptoms?

Migraine headaches are usually located around the eyes, temples, cheeks, and forehead. They are often accompanied by nausea and vomiting. Bell's palsy is a one sided facial paralysis caused by inflammation of the facial nerve. A tension headache usually presents with stress, anxiety, or tension and is located in the frontal, temporal, or occipital region. Temporal arteritis produces pain around the temple but no nausea or vomiting.

When preparing to provide education regarding the prevention of head injuries from motor vehicle accidents, the nurse should be sure to include which point?

Mobile phones should only be used if there is a hands-free option available.

The nurse is planning a presentation on osteoporosis to a group of high school students. Which of the following should the nurse plan to include in the presentation?

Moderate strenuous exercise tends to increase bone density.

Which risk factor for traumatic brain injury should a nurse include in a discussion about prevention for a group of adolescents?

Modes of transportation are the leading cause

Blue-ish black macules appearing over the buttocks and/or thighs of darker skinned neonates, can be mistaken for mild bruising

Mongolian spots

There are millions of sensory receptors relaying information from the skin, mucous membranes, muscles and tendons, and organs of the body and they are doing what?

Monitoring pain, temperature, body position, and touch

Extrapyramidal tract:

Motor nerve fibers that originate in the motor cortex, basal ganglia, brain stem and spinal cord, outside of the corticospinal pyramidal tracts. Controls ability to walk.

You are assessing a patient for acute cholecystitis. What sign would you assess for?

Murphy sign

A client reports the onset of discomfort and pain in the right upper quadrant of the abdomen after eating. The nurse should assess this finding using which test?

Murphy's

During the abdominal examination, a nurse presses her fingers at the client's right costal margin and tells the client to inhale. At this point, the client holds his breath as a result of experiencing a sharp pain where the nurse is pressing. This test is positive for which sign?

Murphy's

A young adult client has just had X-rays and computed tomography scanning of the head and neck following a mountain bicycling accident. All results are negative. What should the nurse assess for next?

Musculoskeletal injury or disease can be confirmed with an X-ray, CT, or MRI. If test results are negative, the nurse should assess for complete range of motion of the neck, looking for any muscle tension, loss of mobility, or pain. According to the scenario, the nurse would not assess for headache, shortness of breath, or ROM of the arms and shoulders next.

The nurse asks the client to perform the action pictured. What is the nurse assessing?

Near vision The client is using the Jaeger chart which is used to assess near vision. The Snellen chart is used to assess distant vision. The nurse would not assess intraocular pressure. Ishihara cards are used to assess color discrimination.

A nurse performs palpation of a client's lymph nodes. Which finding should be reported to the health care provider?

Normally lymph nodes are round and soft, less than 1 cm in size, mobile from side to side, soft in consistency, and nontender. A fixed lymph node may be seen in metastatic disease.

Anatomy of the ear:

Note the cochlea, cochlear nerve, malleus, incus, stapes, auricle (tunnels sound into ear canal), tympanic membrane (ear drum), auditory tube

Anatomy of the mouth/tongue:

Note the gums, lips, frenulums, 32 teeth as an adult, soft/hard palate, uvula, bucal mucousa, pharynx, and tonsil/papillae are the little bumps on the tongue (taste buds)

A client complains of a unilateral headache near the scalp line and double vision. The nurse palpates the space above the cheekbone near the scalp line on the affected side, and the client complains of tenderness on palpation. What is the nurse's next action? A. Notify the healthcare provider immediately. B. Administer intravenous pain medication. C. Palpate the carotid pulses bilaterally at the same time. D. Prepare the client for a temporal artery biopsy.

Notify the healthcare provider immediately.

Direct observation

Nurses usually count calories and documents intake and output

Oncholysis:

Occurs on toes or fingers; discoloration of the nail; fungal infection

Why are old people more prone to UTI's

Older adult clients are prone to UTIs because the activity of protective bacteria in the urinary tract declines with age. (Weber 479)

Where should a nurse place the hands to palpate the submandibular lymph nodes? A. On the medial border of the mandible B. At the angle of the mandible on the anterior edge C. At the posterior base of the skull bone D. A few centimeters behind the tip of the mandible

On the medial border of the mandible

Vitamin B12

Only water-soluble vitamin not found in plants, maintains myelin sheath around nerves. Sources are beef, lamb, organ meats, shell fish, canned tuna, salmon, diary products

Which instruction to the client will help facilitate examination of the temporomandibular joint by the nurse

Open the mouth

The oropharynx is the common channel for the respiratory and digestive systems.

Oropharynx

The client is complaining that his lower joints are increasingly painful as the day progresses. The nurse suspects the client is experiencing what musculoskeletal disorder?

Osteoarthritis

A client with hearing loss by whisper test is further examined with a tuning fork, using the Weber and Rinne maneuvres. The abnormal results are as follows: bone conduction is greater than air on the left, and the client hears the tuning fork better on the left. Which of the following is most likely?

Ostesclerosis of the ear

A nurse obtains an objective assessment on a child who presents to the clinic with reports of right ear pain. The nurse observes the following: painful movement of the pinna and tragus; ear canal is red and swollen with presence of purulent discharge from the external canal; temperature 101.8°F. The mother states that the family was on vacation at the beach last week. The nurse recognizes these findings as an indication of what acute ear condition?

Otitis externa

While positioning the head to examine the ears the client expresses pain when the area behind the ears is touched. On which health problem should the nurse focus during the assessment of this client?

Otitis media

A client presents to the health care clinic complaining of a sore throat. In examining the client's mouth and throat, the nurse notices that the tonsils on both sides of the oropharynx at the end of the soft palate are swollen. Which tonsils are these?

Palatine Explanation: Masses of lymphoid tissue referred to as the palatine tonsils are located on both sides of the oropharynx at the end of the soft palate between the anterior and posterior pillars. The lingual tonsils lie at the base of the tongue. Pharyngeal tonsils or adenoids are found high in the nasopharynx. Paranasal refers to sinuses, not tonsils.

A client has been brought to the emergency unit of a health care facility following an automobile accident. Which finding about the lips supports the diagnosis of anemia and shock?

Pallor.

The nurse splaying to asses a clients abdomen for rebound tenderness. The nurse should:

Palpate deeply while quickly releasing pressure *Blumberg's sign (aka rebound tenderness)- It is indicative of peritonitis. It refers to pain upon removal of pressure rather than application of pressure to the abdomen.

Which action by a nurse is a correct method for performing Tinel's test to determine the presence of carpel tunnel syndrome?

Palpate the hollow area on the back of the wrist -*Percuss lightly on the inner aspect of the wrist* P551 -Ask the client to bend the wrist down and back -Perform wrist movements against resistance

Normal: Left 5th intercostal space, midclavicular line (displacement is normal in pregnant women), finger width = 1 cm, accepted 1-2 cm Abnormal: Displaced (enlarged heart), Enlarged (ventricular enlargement, left ventricular hypertrophy LVH, indicative of heart failure

Palpating the Precordium: Apical pulse

A client reports the onset of pain in the left upper quadrant of the abdomen with the ingestion of alcohol. The nurse recognizes that alteration in function of which organ is most likely to be the cause of pain?

Pancreas

The nurse is assessing the bowel sounds of an adult client. After listening to each quadrant, the nurse determines that bowel sounds are not present. The nurse should refer the client to a physician for possible

Paralytic ileus

The nurse is assessing the bowel sounds of an adult client. After listening to each quadrant, the nurse determines that bowel sounds are not present. The nurse should refer the client to a physician for possible:

Paralytic ileus *Inability of the intestine (bowel) to contract normally and move waste out of the body.

A 73-year-old woman comes to the office for evaluation of new onset of tremors. She is not taking any medications, herbs, or supplements. She has no chronic medical conditions. She does not smoke or drink alcohol. She walks into the examination room with slow, shuffling steps. She has decreased facial mobility with a blunt expression without any changes in hair distribution on her face. Based on this description, what is the most likely reason for the client's symptoms?

Parkinson's disease

Name the three salivary glands:

Parotid (near the ear) Submandibular Sublingual

Upon inspection of a client with reports of a fever, the nurse notices that the client's earlobes are asymmetrical in appearance. The nurse recognizes that the most common cause for the asymmetry of the earlobes is what condition? Bell's palsy Acute pharyngitis Thyroid enlargement Parotid enlargement

Parotid enlargement

Stage II pressure ulcer:

Partial-thickness skin erosion with loss of epidermis/dermis. Superficial ulcer looks shallow like and abrasion or open blister (red-pink wound)

Lupus erythematosus causes:

Patchy hair loss

An adult male client visits the outpatient center and tells the nurse that he has been experiencing patchy hair loss. The nurse should further assess the client for

Patchy hair loss may accompany infections, stress, hairstyles that put stress on hair roots, and some types of chemotherapy.

The client presents at the clinic with a history of cerebral palsy. When examining the patient the nurse notes increased resistance that is rate dependent and increases with rapid movement. What would the nurse chart about this patient?

Patient demonstrates spasticity.

Pressured Speech:

Patient has so much they are trying to say at once

Lethargic:

Patient is super drowsy; they can wake up, talk to you, but they'll probably fall back asleep

Comatose:

Patient is unresponsive to pain

Obtunded:

Patient may open their eyes, but they will respond slowly/confused

Three-day food diary

Patient writes down right after consumption, can be time-consuming

what element of COLDSPA ask "does anything make it better or worse?

Pattern (COLDSPA)

Which statement reflects accurate documentation by the nurse of a normal, left tympanic membrane?

Pearly gray, translucent, with cone of light at 7 o'clock position

Transmission of sound waves in the inner ear is known as

Perceptive hearing

Which action by a nurse is a correct method for performing the Tinel's test to determine the presence of carpel tunnel syndrome?

Percuss lightly on the inner aspect of the wrist.

Normal: Resonance- percussion tone elicited over normal lung tissue Flat tones are percussed over bones and borders of heart and liver Abnormal: Hyperresonance- elicited in cases of trapped air (COPD, Pneumothorax) 9 points of percussion

Percussion: performed to assess obstruction or consolidation of lung tissue

A client has been diagnosed with conductive hearing loss. The nurse understands that which of the following could be the cause of this type of hearing loss? a) Injury to the organ of Corti b) Damage to cranial nerve VIII c) Dysfunction of the temporal lobe of the brain d) Perforated eardrum

Perforated eardrum Explanation: The transmission of sound waves through the external and middle ear is referred to as "conductive hearing," and the transmission of sound waves in the inner ear is referred to as "perceptive" or "sensorineural hearing." Therefore, a conductive hearing loss would be related to a dysfunction of the external or middle ear (e.g., impacted ear wax, otitis media, foreign object, perforated eardrum, drainage in the middle ear, or otosclerosis). A "sensorineural loss" would be related to dysfunction of the inner ear (i.e., cranial nerve VIII, temporal lobe of brain, or organ of Corti).

On a health history, a client reports no visual disturbances, last eye exam being 2 years ago, and not wearing glasses. The nurse notices that the client squints when signing the consent for treatment form and holds the paper close to the face. What should the nurse do next?

Perform both the distant and near visual acuity tests The first thing the nurse should do is perform both the distant and near visual acuity exams to assess for loss of far and near vision. Testing the pupil is important to assess reaction to light. The findings must be documented in the client's record. If abnormalities are found upon assessment, the client should be referred for a complete eye examination.

Telangiectasia:

Permeant dilation of the superficial blood vessels in the skin; can be random; broken blood vessels; secondary lesions; blanchable

General Survey consist of four distinct areas:

Physical appearance (age, LOC, skin color) Mobility (gait, motor activity) Behavior (facial appearance, hygiene) Body structure (height, weight, symmetry, posture)

Define Shift Assessment:

Physical assessment and any changes from admission, q4h depending on severity, do BEFORE giving meds/treatment

approach: head to toe, anterior, posterior, lateral/regional techniques: inspection and palpation maintain standard precautions general survey with VS and head to toe scan

Physical assessment; Integumentary system

A nurse assesses the pupillary reaction to light for a client. Which precaution should the nurse follow to get an accurate result of consensual response?

Place an opaque card in between the eyes of the client The nurse should place an opaque card in between the eyes of the client when assessing the client for consensual response to avoid inaccurate results. The light should not be focused directly into the eye to be tested; it should be focused obliquely into one eye, and the response should be checked in the other eye. The client should not be instructed to close the other eye not focused with light because the response is checked in the other eye.

How should the nurse perform blunt percussion over the liver? Place left hand on right lower rib cage, strike it with radial side of right fist Place right hand on mid of the rib cage; strike it with ulnar side of left fist Place left hand on right lower rib cage, strike it with ulnar side of right fist Place right hand on mid of the rib cage; strike it with ulnar side of left fist

Place left hand on right lower rib cage, strike it with ulnar side of right fist

Stereognosis:

Place small object in patients hand w/ their eyes closed, and they should ID object in 5 secs or so

A nurse is attempting to palpate the abdomen of a 6-year-old girl, but the girl is so ticklish that the nurse cannot proceed. Which of the following should the nurse do?

Place the client's hand under the nurse's hand for a few moments

A nurse is attempting to palpate the abdomen of a 6 year old girl, but the girl is ticklish and the nurse cannot proceed. Which of the following should then nurse do?

Place the client's hand under the nurses hand for a few moments

To palpate the spleen of an adult client, the nurse should:

Place the right hand below the left costal margin

What precaution should the nurse take when measuring a client's abdominal girth to screen for cardiovascular risk factors? Inform the client that the pen mark on the abdomen should not be washed off Ask the client to be seated and relaxed when taking the measurement Ensure that the client has had a full meal before measuring the abdomen Place the tape measure behind the client and measure at the umbilicus

Place the tape measure behind the client and measure at the umbilicus

Which of the following principles should guide the nurse's assessment of clients' oral health?

Poor oral health has both physical and psychosocial implications. Poor oral health constitutes a risk for infection and can aggravate atherosclerosis and lung disease, in addition to reducing social and psychological well-being. It should be assessed in all clients, but it would be erroneous and presumptive to assume its presence.

A female client who works as an administrative assistant which involves a lot of typing as part of her daily duties. She presents to the clinic with complaints of burning, pain, and numbness in both hands. What teaching should the nurse prepare to provide for this client?

Post-operative healing time after carpal tunnel release.

Which terms refers to the progressive hearing loss associated with aging?

Prebycusis (Presbycusis results from gradual degeneration of nerves and sensory hair cells of the organ of Corti and may be related either aging or use of ototoxic drugs)

The nurse observes a white patchy area in the pharyngeal fossa of a client. What is the nurse's best action?

Prepare client for a biopsy of the lesion. The phayngeal fossa is the most common site of oral cancer. A whitish area is a suspicous finding and will likely be biopsied.

The nurse assesses a client's indwelling urinary catheter bag and observes cloudy urine. The client also complains of lower back pain. What is the nurse's best action?

Prepare to obtain a urine specimen for culture.

The nurse is assessing the hearing of an older adult. Which type of hearing problem might the nurse expect to find in the older adult?

Presbycusis

The nurse observes a middle-aged colleague fully extending her arm to read the label on a vial of medication. Which of the following age-related changes is the nurse likely to have observed?

Presbyopia Prebyopia denotes an age-related deficit in close vision. It is less likely that cataracts, macular degeneration, or loss of convergence underlie the colleague's visual changes.

nonblanchable erythema of intact skin not going to turn white/return to color

Pressure Ulcers Stage 1

A client reports severe pain in the posterior region of the neck and difficulty turning the head to the right. What additional information should the nurse collect? A. Previous injuries to the head and neck B. Difficulty with swallowing C. Changes in sleeping habits D. Stiffness in the right shoulder

Previous injuries to the head and neck

A client reports severe pain in the posterior region of the neck and difficulty turning the head to the right. What additional information should the nurse collect?

Previous injury to the head or neck

Sebaceous glands:

Produce sebum which is an adipose like-liquid that comes out of the follicles that provide moisture to skin

Ear wax (cerumen):

Produced or impacted because of narrow, tortuous canal or poor cleaning methods. May be round balls that can obscure eardrum. Ear can be 90-95% blocked with normal earing, but last 5-10% person hears "fullness"

A nurse examines the ear of a client diagnosed with an obstructed eustachian tube. What finding should the nurse anticipate upon assessment?

Prominent landmarks on the tympatic membrane

Some clients with acoustic neuromas have vertigo. What is a priority nursing action with clients with vertigo?q

Protect the client from injury

Parents bring a child to the clinic and report a "rash" on her knee. On assessment, the nurse practitioner notes the area to be a reddish-pink lesion covered with silvery scales. What would the nurse practitioner chart?

Psoriasis

What action should the nurse implement when assessing the ear of an adult client using an otoscope?

Pull the surgical out, up, and back.

How do you move the auricle for an adult when using an otoscope? A child?

Pull up and back for an adult Pull down and back for a child under the age of 3

Vesicles:

Raised lesions less than 1 cm in diameter and filled w/ fluid; Ex. hand, foot, and mouth disease OR blister

Plaques:

Raised, solid and flat topped; greater than 1 cm in diameter; Ex. cirrhoses

A young adult client has just had X-rays and computed tomography scanning of the head and neck following a mountain bicycling accident. All results are negative. What should the nurse assess for next? A. Range of motion of the neck B. Headache C. Shortness of breath D. Range of motion of the arms and shoulders

Range of motion of the neck

Motor tests: coordination =

Rapid alternating movements (RAM), point to point movements (finger to nose), observe gait

Myoclonus:

Rapid, sudden jerk or a short series of jerks at fairly regular intervals; looks like a startle reflex

what tests are done to test for appendicitis

Rebound tenderness/ blumbergs sign Referred rebound tenderness/ rovsings sign psoas sign obturator sign skin hypersensitiviy

A nurse is assessing the mouth of an older client. Which of the following findings is common among older adults?

Receding and ischemic gums The gums recede, become ischemic, and undergo fibrotic changes as a person ages.

Are located in the nose. These receptors are related to the sense of smell.

Receptors of cranial nerve I (olfactory)

Which characteristic feature of the tympanic membrane should a nurse anticipate finding in a client with acute otitis media?

Red, bulging, with an absent light reflex

The nurse assesses an adult client's head and neck. While examining the carotid arteries, the nurse assesses each artery individually to prevent:

Reduction of the blood supply to the brain

While assessing an older adult's neck, the nurse observes that the client's trachea is pulled to the left side. The nurse should

Refer the client to a physician for further evaluation

A client complains of a sudden onset of pain in the back. On questioning the client further, the nurse learns that the cause of the pain is acute pancreatitis. The nurse recognizes that this type of pain is which of the following?

Referred Pain

A client reports using pain medication and sitting in a dark room on the onset of a migraine headache. In which part of the subjective section of the physical examination should the nurse document this information?

Relieving factors includes anything the client subjectively reports they have tried to make the migraine go away. Onset refers to when the migraine started. Location helps determine what part of the client's head the pain is localized within or where it radiates. Treatment refers to any assessment, support, or care the client has received from various health care providers.

What finding should a nurse expect when performing the Phalen's test on a client with suspected carpal tunnel syndrome?

Reports of tingling, numbness, and pain in the involved wrist

Secondary lesions:

Results from changes over time caused by disease or treatment for disease (scratching, picking, rubbing)

Diagnostic tests completed validate that a client has an obstruction of the ascending and transverse colon. Where should the nurse assess for bowel sounds around the obstruction?

Right upper quadrant

To percuss the liver of an adult client, the nurse should begin the abdominal assessment at the clients:

Right upper quadrant

The nurse is admitting a client who is in hypertensive crisis. The doctor's notes indicate that bruits that are both systolic and diastolic have been noted and renal artery stenosis is suspected as the cause of the hypertension. Where would the nurse auscultate the patient's abdomen to hear these bruits? (Select all that apply.)

Right upper quadrant, Epigastrium, constovertbral angles

A nurse performs a Rinne test on a client who relates a history of decreased hearing in the right ear. The test demonstrates that the client has conductive hearing loss in the right ear. What is the correct documentation of this test by the nurse? a) Right: AC> BC; left BC> AC b) Right: AC> BC; left AC= BC c) Right: BC> AC; left BC= AC d) Right: BC> AC; left AC> BC

Right: BC> AC; left AC> BC Explanation: With normal hearing, air conduction should be heard longer than bone conduction. With hearing loss, bone conduction becomes longer than or equal to air conduction. The correct documentation of this finding is Right: BC> AC; left AC> BC.

A client who is taking antibiotics for a sinus infection presents with a white coating on the tongue and complains of a burning sensation on the tongue. Which instructions are most appropriate for this client?

Rinse mouth with antifungal medication as prescribed.

A client has a sore throat and difficulty swallowing that has lasted for months. There are no lesions on the lips. The nurse suspects that the client may have oropharyngeal cancer. What are the risk factors the nurse should assess for in this client?

Risk factors associated with oropharyngeal cancer are as follows: - using tobacco products (including cigarettes, cigars, pipes, and smokeless and chewing tobacco) - heavy alcohol use - chewing betel nuts - infection with a certain type of human papillomavirus - being exposed to sunlight (lip cancer only) - being male - fair skin - poor oral hygiene - poor diet/nutrition - weakened immune system.

A client with advanced presbycusis admits to the nurse that he was nearly involved in a car accident because he could not hear the siren of an ambulance that was crossing an intersection through which he was driving. The client says that he lives alone and has no one else to drive him. Which of the following diagnoses can the nurse make at this time?

Risk for injury related to hearing impairment

A client admitted to the health care facility is diagnosed with vertigo. Which test is appropriate for the nurse to perform to assess for equilibrium in the client?

Romberg

A client presents at the clinic complaining of a loss of balance. What test should the nurse expect the physician to carry out on a client with a loss of balance?

Romberg test

Motor tests: (cont.)

Romberg test (patient stands still w/ feet together and eyes closed to see if they can hold position; be prepared to catch patient) Pronator drift (hold two bowls of soup; close eyes; abnormal is when hand rotates)

During the abdominal examination, a nurse performs deep palpation in the left lower quadrant. At this point, the client reports pain. This test is positive for which sign?

Rovsing's

What does SADPERSONS stand for?

S: Male sex A: Age (<19 or >45 years) D: Depression P: Previous attempt E: Excess alcohol or substance use R: Rational thinking loss S: Social supports lacking O: Organized plan N: No spouse S: Sickness

Which of the following is a symptom of the eye?

Scotomas Scotomas are specks in the vision or areas where the client cannot see; therefore, this is a common and concerning symptom of the eye. Tinnitus is a ringing in the ears, dysphagia is difficulty swallowing, and rhinorrhea is a "runny nose."

What does SBIRT stand for?

Screening Brief Intervention Referral to Treatment

Malnutrition Universal Screening Tool (MUST)

Screening tool which assesses 1. BMI 2. Unintentional weight loss 3. Effect of acute disease on intake for >5 days

An inflammatory skin disorder characterized by macular lesions that may be pink, red, or orange-yellow and may or may not have a fine scale. Distribution is usually on the face, scalp, and ears.

Seborrhea

what element of COLDSPA asks "on a scale of 1-10"

Serverity (COLDSPA)

A client is assigned a visual acuity of 20/100 in her left eye. Which of the following is true?

She can see at 20 feet what a normal person could see at 100 feet. The denominator of an acuity score represents the line on the chart the client can read. In the example above, the client could read the larger letters corresponding with what a normal person could see at 100 feet.

A nurse is providing nutritional instruction at a health fair. She instructs passersby on the characteristics of a nutrient that is the body's first source of energy, sparing use of other nutrients for this purpose, that raises the blood glucose level, is found in fruit juices, and that can be converted quickly into energy. To which of the following nutrients is the nurse referring?

Simple carbs

The nurse is preparing to assess the lymph nodes of an adult client. The nurse should instruct the client to

Sit in an upright position

A nurse should assist a client to assume what position in order to best assess the mouth, nose, and sinuses?

Sit with the head erect and at the eye level of the nurse.

A nurse should assist a client to assume what position to best assess the mouth, nose, and sinuses?

Sitting with the head erect and at the eye level of the nurse The nurse should ask the client to assume a sitting position with the head erect and at the eye level of the examiner. Tilting the head backwards and a semi-recumbent position with the chin lifted will make it more difficult to visualize the mouth and nose. The prone position will make transillumination and palpation of the sinuses more difficult for the examiner.

Special assessments include:

Skin Risk, hourly rounds, fall risk, IV/catheter assessments, neuro assessment

The nurse is presenting an educational event for gardeners. When discussing the ears, what would be an important topic to cover?

Skin cancer prevention

The nurse is assessing a 79-year-old man who experienced an ischemic CVA 7 weeks prior and has a consequent loss of mobility. Because the client spends so much time immobilized, the nurse recognizes the importance of screening for pressure ulcers. Which of the following assessment findings would signal to the nurse an early sign of skin breakdown?

Skin that feels boggy on palpation

A client presents to the health care clinic with reports of changes in the skin. Which data should the nurse document as objective with regards to the skin?

Skin warm and dry to the touch

The nurse is caring for a client suffering from a nutritional deficiency. The nurse expects that the client has a dysfunction of which abdominal body part?

Small intestine Absorption of nutrients takes place almost exclusively in the small intestine. The esophagus propels the food bolus by means of slow peristaltic movements into the stomach. The descending colon is part of the large intestine. Mastication occurs in the mouth, then food moves into the oropharynx and esophagus for food propulsion through the digestive tract.

A 23-year-old woman has presented to the clinician to follow up her recent diagnosis of psoriasis. Which of the following assessments of the client's nails would be consistent with the client's diagnosis?

Small pits in the nails are an early sign of, though not specific for, psoriasis.

Exostoses:

Small, bony, hard, rounded nodules of hypertrophic bone covered with normal epithelium; arise near the drum but usually do not obstruct view of drum; occurs typically in cold-water swimmers; no treatment; may cause accumulation of cerumen

A client reports the feeling of increased gas in the abdomen. The nurse recognizes that which organs may be difficult to percuss due an increase in air or intestinal gas? Select all that apply.

Spleen

A nurse performs percussion beginning along the left midaxillary line and progressing downward until the sound changes from lung resonance to splenic dullness. The client reports tenderness. The nurse recognizes this as an abnormal finding for which organ?

Spleen

The ossicles contained in the middle ear include the malleus, the incus, and the

Stapes

Peripheral vision/Confrontation exam:

Start at temporal fields; bring fingers forward until they can see both fingers in their periphery

Stenson's ducts are visible on the buccal mucosa across from the second upper molars.

Stenson's ducts

A client is having trouble turning her head to the side. Which of the following muscles should the nurse most suspect as being involved?

Sternocleidomastoid

A client visits the clinic because she experienced bright hematemesis yesterday. The nurse should refer the client to a physician because this symptom is indicative of:

Stomach ulcers

A nurse performs light palpation of the abdomen and feels a prominent, nontender, pulsating mass above the umbilicus that measures approximately 6 cm. What is an appropriate action by the nurse?

Stop the palpation and notify the health care provider

A nurse performs light palpation of the abdomen and feels a prominent, nontender, pulsating mass above the umbilicus that measures approximately 6cm. What is an appropriate action by the nurse?

Stop the palpation and notify the health care provider

The primary function of the gallbladder is to:

Store and secrete bile

When the nurse is obtaining a health history of the urinary system, the client reports "leaking" urine when coughing or laughing. Which of the following problems is the client likely experiencing?

Stress incontinence

what is the term for stretch marks?

Striae

The nurse is planning to conduct the Weber test on an adult male client. To perform this test, the nurse should plan to

Strike a tuning fork and place it on the center of the clients head or forehead

A 12-year-old presents to the clinic with his father for evaluation of a painful lump in the left eye. It started this morning. The client denies any trauma or injury. There is no visual disturbance. Upon physical examination, there is a red raised area at the margin of the eyelid that is tender to palpation; no tearing occurs with palpation of the lesion. Based on this description, what is the most likely diagnosis?

Stye A hordeolum or stye is a painful, tender, erythematous infection in a gland at the margin of the eyelid.

Cerebellar system:

Subconscious; controls coordination, equilibrium, and posture

What type of data is this? Hx of heart problems, rheumatic fever, hypertension, heart murmurs? Hx of EKG of other heart test results? Concerning symptoms: chest pain, palpitations, SOB, DOE: dyspnea on exertion Orthopnea: SOB, or PND (Paraxysmal Nocternal Dyspnea) difficulty breathing intermittently while sleeping Edema Family hx of hypertension or life-style?

Subjective Data Collection

When inspecting the abdomen, which of the following client positions facilitates correct examination technique?

Supine with arms at sides or folded across chest A supine position with pillows under the client's head and knees is most conducive to accurate examination and is preferable to a sitting, Trendelenberg, or semi-Fowler's position.

Common in local or systemic allergic reaction.

Swelling of the lips

Inspect the eyes for:

Symmetry of eyes, eyebrows; parasites; redness, swelling, inflammation; discharge; lesions; sunken appearance; protrusion; conjunctiva/sclera are clear

Inspect/palpate the neck for:

Symmetry, ROM, midline trachea, thyroid, skin (lesions, lumps, color)

Inspect and palpate the muscles for:

Symmetry, size, atrophy, strength (1-5; 1 no muscle contraction, 5 = normal), tone, spasticity, flaccidity, tics

Which of the following is an important function of the skin?

Synthesis of vitamin D A vital role of the skin is the synthesis of vitamin D. Carotene exists in sebaceous fat, and melanin deposits are a normal component of skin. Skin does not significantly contribute to pH maintenance.

-Ventricular repolarization -Ventricles return to a resting state

T wave

While assessing an adult client's jaw, the nurse hears a clicking popping sound, and the client expresses pain in the joint. The nurse should further assess the client for

TMJ dysfunction

The client is having a thyroid crisis. What symptoms would the nurse assess for? Select all that apply.

Tachypnea Nausea Anxiety

The nurse is preparing to palpate the client's spleen. What should the nurse instruct the client to do?

Take a deep breath and exhale

Mrs. Fletcher presents to the office with chronic unilateral pain when chewing. She does not have facial or scalp tenderness. Which of the following is the most likely cause of her pain? a) Trigeminal neuralgia b) Temporal arteritis c) Tumour of the mandible d) Temporomandibular joint syndrome

Temporomandibular joint syndrome

Which of the following would the nurse document as an abnormal finding with lymph node assessment?

Tender

A client comes to the trauma unit in respiratory distress following a motor vehicle accident. On examination, the nurse notices that the trachea is deviated from the midline. What does this finding indicate? Tension pneumothorax Cardiac tamponade Flail chest Severe neck fracture

Tension pneumothorax

Which of the following findings related to hair would the nurse most likely assess in an older adult female client?

Terminal hair growth on chin

Diencephalon ("between brain"):

Thalamus: controls pain, perception of cold, perception of pleasant/unpleasant Hypothalamus: perception of hunger, thirst, and sexual arousal

Are found chiefly in the axillary and genital regions, usually open into hair follicles, and are stimulated by emotional stress. This type of gland does not secret on locations identified by the other options.

The apocrine glands

The nurse is assessing an elderly woman who reports lethargy and weakness. During the examination, the nurse notices that the client's face is drooping slightly on the right side. Why does this finding require emergency intervention?

The client could be experiencing a stroke

A nurse is admitting a client to the hospital. When reviewing the client's medical record, the nurse notes that this client had abnormal findings during the Weber test. What would the nurse know this means?

The client has unilateral hearing loss

Which of the following statements provides the most accurate guide to the assessment of the gallbladder?

The gallbladder is deep to the liver and cannot normally be distinguished from the liver clinically Because the gallbladder is deep to the liver, it is normally not amenable to direct examination by auscultation, palpation, or percussion. This does not mean, however, that cholecystitis and cholelithiasis cannot be assessed for a thorough history. The gallbladder and the spleen are not proximate.

What structure in the inner ear senses the position and movements of the head and helps to maintain balance?

The labyrinth

A client reports severe pain in the left lower quadrant of 3 days duration. How should the nurse conduct palpation of the abdomen due to this history?

The left lower quadrant is palpated last

The health care provider documented that a client is experiencing edema of the left maxillary sinus. Where on the diagram provided should the nurse expect to observe this swelling?

The maxillary sinuses are located in the upper jaw. (cheek under eye kinda)

A patient comes to the clinic and reports nosebleeds. What area of the nose is the bleeding most likely coming from?

The most common site of nasal bleeding is the Kiesselbach plexus on the anterior septum.

The nurse is caring for a patient who comes to the clinic reporting a lump by her ear. What are the symptoms of a cancerous lymph node?

The node is fixed and rubbery

The nursing instructor is discussing assessment of the head and neck with the class. What identifying characteristic would the instructor use for the thyroid cartilage? A. Its position just below the mandible B. The curve on its inferior edge C. Its relation to the cricoid cartilage D. The notch on its superior edge

The notch on its superior edge

Which action by the nurse is consistent with Weber's test?

The nurse activates the tuning fork and places it on the midline of the parietal bone in line with both ears

A nurse examines a client with complaints of a sore throat and finds that the tonsils are enlarged and touching one another. Using a grading scale of 1+ to 4+, how should the nurse appropriately document the tonsils?

The nurse should document the tonsillar grading as 4+ because the tonsils are so large that they are touching one another. Grade 2 tonsils are midway between the tonsillar pillars and the uvula. Grade 1 tonsils are ones that are barely visible. Tonsils that touch the uvula are graded 3+.

A nurse is examining the nose of a client diagnosed with an upper respiratory tract infection. Which characteristics of the nasal mucosa should the nurse expect to find during assessment of a client with an upper respiratory tract infection?

The nurse should find red, swollen nasal mucosa with purulent discharge in the client diagnosed with upper respiratory tract infection. Dark pink, moist nasal mucosa which is free of exudate is a normal finding. Pale pink, swollen nasal mucosa with watery exudate and bluish gray, swollen nasal mucosa with watery exudate is found in cases of allergy.

During assessment of the oral cavity, the nurse examines the salivary glands. Which area of the mouth should the nurse assess to inspect for the Wharton's ducts?

The nurse should inspect the Wharton's ducts on either side of the frenulum on the floor of the mouth.

A client presents to the health care clinic with reports of a 3-day history of fever, sore throat, and trouble swallowing. The nurse notes that the client is febrile, with a temperature of 101.5°F, tonsils are 2+ and red, and transillumination of the sinuses is normal. Which nursing diagnosis should the nurse confirm based on this data?

The nursing diagnosis of Acute Pain can be confirmed because it meets the major defining characteristic of verbalization of sore throat. Impaired swallowing is not related to impaired neurologic or neuromuscular function. There is no criterion to confirm that this client cannot maintain health maintenance because this is an acute problem. No data exist to confirm the nursing diagnoses of Self-Care Deficit or Hopelessness.

The nurse is admitting a 79-year-old man for outpatient surgery. The patient has bruises in various stages of healing all over his body. Why is it important for the nurse to promptly document and report these findings?

The patient may have been abused. Multiple ecchymoses may be from repeated trauma (falls), clotting disorder, or physical abuse.

A 52-year-old patient fails the Romberg test. The nurse explains that this might indicate a dysfunction in what part of the ear?

The vestibular portion of the inner ear (Failure of the Romberg test may indicate dysfunction in the vestibular portion of the inner ear, semicircular canals, and vestibule.)

Which of the following findings from the health history of a 70-year-old woman with tinnitus is likely most significant to her diagnosis? a) The woman was diagnosed with oral cancer 12 years prior, which responded well to treatment. b) The woman has experienced transient ischemic attacks in recent years. c) The woman takes a beta-blocker to treat hypertension. d) The woman takes aspirin 4 times daily to treat her rheumatoid arthritis.

The woman takes aspirin 4 times daily to treat her rheumatoid arthritis. Explanation: Excessive use of aspirin can cause tinnitus. A history of TIAs and cancer is not likely to be related, nor is the use of antihypertensives.

process where nurse influences a pt or helps pt to a better understanding through verbal/non-verbal communication

Therapeutic communication

What notation demonstrates the nurses understanding of effective documentation when the assessment findings identify thick, purulent drainage in both eyes of a client?

Thick, purulent drainage is noted at inner corner of both eyes. The abbreviations OD (right eye), OS (left eye), and OU (both eyes) are no longer used due to the potential for order errors. Instead, it is recommended to use "right eye," "left eye," or "both eyes."

A group of students is reviewing information about general assessment indicators of nutritional status. The students demonstrate a need for additional review when they identify which of the following as an indicator of adequate nutritional status?

Thinning, dry hair

During an assessment the nurse observes the condition shown in the client's mouth. What should the nurse suspect is occurring with this client?

This is gingival hyperplasia, or an overgrowth of gum tissue. It is seen in pregnancy, puberty, leukemia, and medications such as phenytoin. Gum tissue would not be overgrown if dentures are worn. This is not periodontal disease or evidence of significant dental caries.

Upon inspection of a Native American client's oral cavity, a nurse observes a bifid uvula. What should the nurse recognize about this finding?

This is often a normal finding in the Native American population.

Define initial assessment:

Thorough and accurate, complete rapidly, but not in a hurry, re-assess q4-8h (shift assessment)

A 58-year-old man who is HIV-positive has presented with thick, white plaques on his oral mucosa. What diagnosis would the nurse first suspect?

Thrush

A patient diagnosed with arthritis has been taking aspirin and now reports experiencing adverse effects. What adverse effect indicates that a decrease in dose may be necessary?

Tinnitus (Aspirin Explanation: At high doses, aspirin toxicity can produce bilateral tinnitus. Antivert and Dramamine is used for nausea and vomiting related to motion sickness. Antiemetics such as promethazine (Phernergan) suppositories help control the nausea and vomiting and the vertigo because of the antihistamine effect.)

A nurse is conducting a comprehensive nutritional assessment on a patient with suspected malnutrition. Why would it be important to assess this patient's ability to cook?

To assess if the patient has the ability to obtain or prepare food

While the nurse is obtaining a client's health history regarding the head and neck,the client tells the nurse about having a lump in the neck. In order to assess for associated manifestations of this problem, which of the following questions should the nurse ask next?

To assess manifestations associated with the lump in the neck, the nurse would ask if the patient has difficulty swallowing. Asking how long the client has experienced discomfort from the lump is associated with duration. Asking when the patient first noticed the lump assesses the onset of the lump. Asking if there is there more than one lump assesses the location of the lump.

You are teaching a physiology class for pre-nursing students. A student asks what the purpose of the upper airway is in regard to the lower airway. What would be your best answer?

To warm the inspired air.

The normal tongue has visible veins on the ventral surface and is pink or pale in color and moist. A normal variation seen in the older client is a fissured, topographical map-like tongue.

Tongue (normal findings)

Tonsils are red, enlarged (to 2+, 3+, or 4+), and covered with exudate in tonsillitis. They also may be indurated with patches of white or yellow exudate.

Tonsils (abnormal)

Tonsils may be present or absent. They are normally pink and symmetric and may be enlarged to 1+ in healthy clients. No exudate, swelling, or lesions should be present.

Tonsils (normal)

1+ Tonsils are just visible. 2+ Tonsils are midway between tonsillar pillars and uvula. 3+ Tonsils touch the uvula. 4+ Tonsils touch each other.

Tonsils grading

The nurse understands that malocclusion may be related to what?

Tooth loss Explanation: Malocclusion (imperfect positioning of the teeth when the jaws are closed) may occur with tooth loss.

The nurse teaches the client that overuse of this medication can cause rebound nasal congestion.

Topical decongestants can cause rebound nasal congestion.

Describe what the tympanic membrane looks like (color/structures):

Translucent, shiny, pearly-gray Cone shaped light reflex at 5 (right) and 7 (left) Retract/bulging (ear infection)

A 38-year-old woman presents with multiple small joints that are symmetrically involved with pain, swelling, and stiffness. Which of the following is the most likely explanation?

Trauma Gout Septic arthritis *Rheumatoid arthritis* Explanation: Rheumatoid arthritis is a systemic disease and accounts for multiple symmetrically involved joints. Septic arthritis is usually monoarticular, as are gout and trauma related joint pain. P552

Excoriation:

Traumatized/abraded area of skin caused by scratching or itching

A 50-year-old woman presents with "left hip pain" of several weeks duration. She is exquisitely tender when the nurse presses over her proximal lateral thigh. What is her most likely health problem?

Trochanteric bursitis

The nurse might expect the patient admitted with dehydration to have tachycardia. T/F

True

Which portion of the middle ear equalizes pressure? a) Cochlea b) Auricle c) Ossicles d) Tympanic membrane

Tympanic membrane Explanation: The tympanic membrane of the middle ear equalizes pressure. Ossicles, which are held in place by joints, muscles, and ligaments, assist in the transmission of sound. The auricle collects the sound waves and directs vibrations into the external auditory canal. The cochlea is a winding, snail-shaped bony tube that forma a portio of the inner ear and containss the organ of Corti; the transducer of hearing

When assessing the ears of older adults, it is necessary to remember that physiologic changes take place as people age. What is a physiologic change that takes place in the ear of an older adult?

Tympanic membrane is more opaque

While performing an otoscopic examination of a patient's ear, the APRN finds white patches on the tympanic membrane. How would the APRN note this finding in the chart?

Tympanic membrane visualized with areas of opacity noted

Visual Acuity Test:

Use Snellen Chart; position patient 20 feet from chart; the denominator is the last line the patient is able to read at least half way through; 20/200 = patient can read at 20 ft. what a normal person could read at 200 ft.

How can a nurse accurately assess the distant visual acuity of a client who is non-English speaking?

Use a Snellen E chart to perform the examination If a client does not speak English, is unable to read, or has a verbal communication problem, the Snellen E chart can be used to test the client's distant visual acuity. With this test, the client is asked to indicate by pointing which way the E is open on the chart. The six cardinal positions of gaze test eye muscle function and cranial nerve function. The Jaeger chart tests near visual acuity. Confrontation test is used to test visual fields for peripheral vision.

During an examination of the oral cavity, which technique by the nurse is most likely to improve visualization of the buccal mucosa?

Use a penlight and tongue depressor to retract the lips.

A teenager is brought to the clinic for a sports physical examination. The client states plans to play goalie on the community soccer team. What is the most important teaching opportunity presented for this client?

Use of safety equipment The nurse should assess with each client the use of safety equipment when playing sports. Proper eye protection can prevent many sports-related eye injuries. All options are points for client teaching for this client; however, the most important opportunity involves the use of safety equipment.

During a physical examination of an adult client, the nurse is preparing to auscultate the clients abdomen. The nurse should:

Use the diaphragm of the stethoscope

Which best describes audiometry?

Uses headphones and a box that delivers tones to each ear at variable frequencies and volumes.

Pitting:

Usually occurs from a nutritional deficiency, cirrhoses

Satellite lesions:

Usually occurs w/ yeast infection; fungal infection; main lesion sends out "spores" to start a new lesion elsewhere

An extension of the soft palate of the mouth, which hangs in the posterior midline of the oropharynx, is the

Uvula.

A 30-year-old client arrives at the community healthcare center complaining of dizziness and a feeling of the room spinning. Based on the client's symptoms, which condition best describes what the client most likely experiencing?

Vertigo

The clinic nurse is assessing an older adult client who fell. The nurse asks if the client ever feels dizzy or has problems with balance. For what is the nurse assessing?

Vertigo

When planning care for a client with an inner ear infection, the nurse will need to include interventions for which of the following potential problems?

Vertigo

Example of what lesion? Varicella and herpes simplex

Vesicular lesions

When visualizing the structures of the nose, the nurse recalls that air travels from the anterior nares to the trachea through the:

Vestibule Nasal passages Nasopharynx After entering the anterior nares, air enters the vestibule and passes through the narrow nasal passage to the nasopharynx.

A patient is found to have a smooth, glossy tongue. What vitamin deficiency might this indicate?

Vitamin B12 deficifency The tongue and buccal mucosa may appear smoother and shiny from papillary atrophy and thinning of the buccal mucosa. This condition is called smooth glossy tongue and may result from deficiencies of riboflavin, folic acid, and vitamin B 12.

Anatomy of the eye:

Vitreous humor: clear, gel-like substance that lies behind the lens, near the retina Aqueous: clear liquid, thinner; located near cornea and lens

Inspect and palpate the mouth and throat for:

Wear gloves! Lips (color, moisture, ulcers, scaling) Buccal mucosa (use tongue blade; color, ulcers) Teeth and gums (clench teeth; color, missing/position of teeth) Hard and soft palate Tongue (symmetry; sides, top, and bottom; use gauze to palpate) Oropharynx (say "ahhh" to see uvula rise and if midline) Tonsil grading: +1 visible, +2 halfway between tonsillar pillars and uvula, +3 touching the uvula, +4 touching one another

A client tells the nurse that it is difficult to hear normal conversation when in a room with a lot of noise. Which test should the nurse perform to assess the hearing ability of the client?

Weber

The submandibular glands open under the tongue through openings called

Wharton ducts. The submandibular glands, located in the lower jaw, open under the tongue on either side of the frenulum through openings called Wharton's ducts.

24 hour recall

What a patient ate or drank in the past 24hours, using open ended questions. Foods may vary from weekday to weekend. Only applicable for patients who can remember

indicates fluid overload best heard in the left lateral decubitus position sounds like Ken-TUCK-y

What does an S3 sound indicate? Where is it best heard?

high-pitched, musical sounds heard primarily during expiration but may also be heard on inspiration often heard in acute asthma or chrnoic emphysema

Wheeze (sibilant)

Resting vs. Intention Tremor:

When at rest the patient has a tremor, but may disappear when in use/intention tremor occurs when in use

The nurse is conducting a physical examination of a client who reports finding a lump in the neck. Which of the following questions should be included in when the nurse is collecting subjective data? (Select all that apply). A. When did you first notice the lump? B. How recently have you consumed alcohol? C. Do you have family members that experience lumps? D. Has the lump changed? E. Is it painful to touch?

When did you first notice the lump? How recently have you consumed alcohol? Has the lump changed?

When inspecting the nose with an otoscope, the nurse:

When inspecting the nose with an otoscope, the nurse should avoid touching the sensitive nasal septum and hold the otoscope to one side. The largest ear speculum available should be used, and it should be directed posteriorly.

The nurse is speaking to a group of seniors about health promotion and is preparing to discuss the ABCDEs of melanoma. Which of the following descriptions is correct for the ABCDEs?

a = asymmetry; b = irregular borders; c = color changes, esp. blue; d = diameter > 6 mm; e = evolution

The nurse is assessing a dark-skinned client who has been transported to the emergency room by ambulance. When the nurse observes that the client's skin appears pale, with blue-tinged lips and oral mucosa, the nurse should document the presence of

a great degree of cyanosis. Cyanosis may cause white skin to appear blue-tinged, especially in the perioral, nail bed, and conjunctival areas. Dark skin may appear blue, dull, and lifeless in the same areas.

findings: . symmetry · Anteroposterior to transverse diameter is 1:2 · Spinous process is straight

a normal thoracic cage

A 55-year-old male presents to the health care clinic with reports of decrease hearing over the past year. Which subjective data in client's review of systems should the nurse recognize as risk factors for hearing loss? Select all that apply. History of measles at age 3 years of age Drinks six cups of coffee daily Use of antihypertensive medication Uses ear plugs at work because of the loud noises Repeated episodes of otitis media as a child Increased cerumen production

a,e,f

what areas do you listen for bruits

abdominal aorta, ileac, femoral

When you are observing the aortic pulsations and they are exaggerated, what is this potentially a sign of?

abdominal aortic aneurysm

what does a pulsation mean

abdominal aortic anneurysm

what kind of finding is this? bruit- sound caused by turbulent blood flow

abnormal finding - carotid pulse

A nurse examines a client with a paralytic ileus. Which alteration in the bowel sounds should the nurse expect to find with auscultation of the client's abdomen? Absent Hyperactive Erratic Hypoactive

absent

Which function of the skin does this describe: allows limited excretion of some metabolic wastes and also absorbs chemicals

absorbtion, excretion

Define: Characterized by areas of dark, velvety discoloration in the folds and creases usually in the armpits, groin, and neck. Could be due to diabetes or prediabetes in children or young adults.

acanthosis nigricans

-A sign of pressure overload -Increased force and duration

accentuated apical impulse

The functional reflex that allows the eyes to focus on near objects is term

accommodation. Accommodation is a functional reflex allowing the eyes to focus on near objects. This is accomplished through movement of the ciliary muscles, causing an increase in the curvature of the lens.

Upon assessing the skin, the nurse finds pustular lesions on on the face. The nurse identifies that these could be what?

acne

Papular lesions

acne, warts, nevi, insect bites, early varicella

What condition is this? excessive secretion of growth hormone from pituitary after puberty creates an enlarged skull and thickened cranial bones.

acromegaly

While assessing an adult client's skull, the nurse observes that the client's skull and facial bones are larger and thicker than usual. The nurse should assess the client for

acromegaly

Define: coarse wrinkling, decreased elasticity, atrophy, speckled and uneven coloring, more pigment changes, and a yellowed, leathery texture occur.

actinic purpura (senile purpura)

Define: the process of paying close attention to what the patient is communicating

active listening

Define: basic self-care activities such as bathing, dressing, feeding, continence, etc.

activities of daily living (ADLs)

Define: pain that is short term, arises quickly and self-limiting, associated with physical cause.

acute pain

At what stage of life do skin problems such as pimples and blackheads appear, secretions of apocrine sweat glands increase, subcutaneous fat deposits increase, and secondary sex characteristics appear?

adolescence

observable response a person has to his or her own emotions

affect

responses are closed/restricted- disinterested

affect- blunted

At what stage of life does the skin lose it's elasticity, sweat and sebaceous glands decrease in number and function, skin breakdown occurs, functioning melanocytes decrease, liver spots appear, and actinic purpura appears?

aging adult

For what group should you take this information into consideration? -arteries may look twisted and predominant -may have mild tremors -lessened skin elasticity -missing teeth -less subcutaneous fat -less moisture in skin -more prominent facial bones

aging adults

What does the AUDIT screening tool indicate?

alcohol use disorders identification test, much more sensitive, picks out binge drinking habits

The nurse has assessed the nose of an adult client and has explained to the client about her thick yellowish nasal discharge. The nurse determines that the client understands the instructions when the client says that the yellowish discharge is most likely due to

an upper respiratory infection Exudate is common with infection and may range from large amounts of watery discharge to thick yellow-green, purulent discharge. Purulent nasal discharge is seen with acute bacterial rhinosinusitis.

Target lesions:

annular, but have specific red center; Ex. Lime disease, arrhythmia multiformae

Name this area (in triangle)

anterior triangle

what is pulsating mass above the umbilicus bruit

aortic aneurysm

open directly into the hair follicle, develop in areas abundant in hair follicles: scalp, axillae, inguinal areas

apocrine sweat glands

What are the names of the 2 types of sweat glands?

apocrine, eccrine

The nurse identifies the client has a positive Obturator sign. The nurse identifies this is due to what? Kidney pain Inflammation of the gallbladder Appendicitis Liver engorgement

appendicitis

During the health history of the nose and sinuses, a patient complains of having rhinorrhea. What question would the nurse ask to determine associated manifestations of this symptom?

are there any other symptoms? Asking if there are any other symptoms assesses the manifestations associated with rhinorrhea. Asking the patient about which side it occurs assesses the location. Asking how long it lasts assesses the duration. Asking the patient about the color of drainage assesses the characteristics of the symptom.

The small, involuntary muscle in the base of the hair follicle. "goose bumps"

arrector pili muscle

Pain: intermittent claudication to sharp, unrelenting/constant Pulses: diminished/absent Skin characteristics: dependent rubor Dry shiny skin Cool to cold temperature Loss of hair over toes and dorsum of feet Leg edema: minimal unless leg kept in dependent position

arterial insufficiency

While assessing the elbow of an adult client, the client complains of pain and swelling. The nurse should further assess the client for

arthritis

what could abdominal swelling in a patient mean

ascites

To assess for anemia in a dark-skinned client, the nurse should observe the client's skin for a color that appears

ashen. Pallor (loss of color) is seen in arterial insufficiency, decreased blood supply, and anemia. Pallid tones vary from pale to ashen without underlying pink.

The nurse is preparing to perform a head and neck assessment of an adult client who has immigrated to the United States from Cambodia. The nurse should first A. explain to the client why the assessment is necessary. B. ask the client if touching the head is permissible. C. determine whether the client desires a family member present. D. examine the lymph nodes of the neck before examining the head.

ask the client if touching the head is permissible.

listening to sounds produced by the body

assculation

During the health history, a client who has abdominal pain reports having occasional nausea and diarrhea. In which section of the health history should the nurse document this finding?

associated manifestations The nurse should document this finding in the associated manifestations section because this is a report on the experience of other symptoms associated with abdominal pain. In relieving factors, the nurse explores factors that aggravate or relieve the pain. In characteristic symptoms, the nurse should ask the client to describe the pain in his or her own words. Onset refers to when the abdominal pain started.

A client arrives complaining of nasal congestion, drainage of a thick, yellow discharge from the nose, difficulty breathing through the nose, headache, and pressure in the forehead. The nurse suspects sinusitis. Which of the following risk factors should the nurse assess for in this client?

asthma This client shows symptoms of sinusitis. Risk factors for sinusitis include a nasal passage abnormality, aspirin sensitivity, cystic fibrosis, chronic obstructive pulmonary disease (COPD), an immune system disorder, hay fever, asthma, and regular exposure to pollutants such as cigarette smoke. The other answers listed—chewing betel nuts, exposure to the sun, and heavy alcohol use—are all risk factors for oropharyngeal cancer, but not for sinusitis.

upper chambers of the heart thin walls, serve as a reservoir for blood (boost amount of blood moving to ventricles, atrial kick)

atria

what is a fixed lymph node?

attached to an underlying organ/tissue, a more serious issue

The nurse is planning to assess a client's abdomen. Which assessment technique should the nurse use after inspecting the area?

auscultation

Which temperature route does this statement describe: Can be used on patients who are unconscious

axillary

When providing client education on hearing, the nurse should remind clients to utilize ear plugs when they are what? (Select all that apply.) Working with children At train stations Using lawnmowers At concerts Cleaning their homes

b,c,d

Seen in COPD, emphysema 1:1 Ap diameter Hyperinflated lungs

barrel chest

most common type of skin cancer non-invasive, will not spread- seen often on the face/sun exposed surfaces

basal cell cancer

why are elderly patients more prone to UTIs

because protective bacteria in urinary tract will decline with age

wound-proliferative phase

begins at end of inflammatory phase, may last up to 4 weeks

wound remodeling phase

begins at end of proliferative phase, can last up to 2 years

What component of the general survey describes facial expression, mood and affect, speech, dress, and personal hygiene?

behavior

What is the single most important rule when discussing sensitive topics?

being nonjudgmental

Define: force of blood pushing against side of its container, vessel wall

blood pressure

what happens when you have an inflammatory bowel disease

bloody stool

what do you do to assess kidney tenderness

blunt percussion over CVA

What component of the general survey describes stature, nutrition, symmetry, posture, position, body build, contour, and obvious physical deformities?

body structure

While auscultating a client's abdomen, the nurse hears the client's stomach growling. The nurse knows that this is which type of bowel sound? Borborygmus Absent Erratic Hypoactive

borborygmus

what does a pulse strength of 4+ mean?

bounding (unable to obliterate) - fluid overload

How would you describe a heart rate <60bpm?

bradycardia

What exam goes in this order? 1. inspect 2. auscultation 3. palpate

carotid pulse exam order

A client is unable to externally rotate the left shoulder. What health problem should the nurse suspect is occurring with this client?

carpal tunnel syndrome rotator cuff tendinitis anterior dislocation of the humerus *rotator cuff tear* Exp: In a complete tear of the supraspinatus tendon, or a rotator cuff tear, active abduction and forward flexion at the glenohumeral joint are severely impaired, producing a characteristic shrugging of the shoulder and a positive "drop arm" test. Rotator cuff tendonitis is characterized by acute, recurrent, or chronic pain of the supraspinatus tendon. Carpal tunnel syndrome effects the wrist and not the shoulder. Anterior dislocation of the humerus is characterized by the shoulder seeming to slip out of the joint. P546

The thyroid gland secretes T3 and T4 which stimulate ________ __________

cellular metabolism

what are some indications of skin changes?

changes in mole or lesion non-healing sore/chronic ulcer pruritis (itching) rashes hair changes nail changes

A nurse asks the client to describe the pain associated with a headache by rating the pain on a scale from 1 to 10. This subjective data should be documented in which section of the assessment? A. characteristic symptoms B. associated manifestations C. relieving factors D. location

characteristic symptoms

person providing assistance is professionally accountable for the outcomes, empathetic, purposeful and time limited, dynamic, professional behavior

characteristics of therapeutic communication

At what stage of life does the epidermis thicken, darken, and become lubricated and hair growth accelerates?

childhood

Mask of pregnancy (pigmentation increases on the face)

chloasma

what is abdominal pain , NV/D, cramping, and weight loss

chrons disease

The nurse identifies this as trapping debris and propelling it toward the nasopharynx.

cilia Cilia capture and propel debris toward the nasopharynx. Turbinates are bony lobes that project from the lateral walls of the nasal cavity. The lacrimal duct receives drainage. Th columella divides the nostrils.

Define: understand their meaning, request clarification, as in "tell me exactly what you mean by "the flu" or "you said you were behaving just like your mother, what did you mean?"

clarification

where is the lg. intestine located

colon

When checking that the nails are translucent, with an even, pink nailbed underneath you are checking for _______

color

What are the 9 factors to look for when assessing lesions?

color, size, elevation, number, texture, location, distribution, shape, pattern

what are elderly patients at risk for

complications of diarrhea such as fluid volume deficit, dehydration, electrolyte imbalance due to more body fat then lean muscle

The nurse has performed the Rinne test on an older adult client. After the test, the client reports that her bone conduction sound was heard longer than the air conduction sound. The nurse determines that the client is most likely experiencing a) normal hearing. b) central hearing loss. c) conductive hearing loss. d) sensorineural hearing loss.

conductive hearing loss. Explanation: With conductive hearing loss, bone conduction (BC) sound is heard longer than or equally as long as air conduction (AC) sound (BC ? AC).

Define: What kind of shape/configuration of lesions is characterized by lesions that run together (EX. hives)

confluent

The nurse notes that the pupil of a client's left eye constricts when a light is shined into the right eye. How should the nurse document this finding?

consensual light response present in left eye The consensual light response occurs when one eye is exposed to light and the pupil of the other eye constricts. Since the light was shined in the right eye, the left pupil constricted. The left eye was not exposed to direct light. There is not enough information to determine if the pupils are equal or reacting to accommodation.

A college student presents to the health care clinic with reports no bowel movement for four (4) days, bloating, and generalized abdominal discomfort. She states she has not been eating and drinking correctly and is stressed because she has a final exam in two (2) days. A nurse assesses the abdomen and finds positive bowel sounds in all four quadrants, tenderness in the left lower quadrant with a few small round, firm masses. Rovsing's sign and the Psoas sign are negative. What nursing diagnosis can the nurse confirm for this client?

constipation related to decrease in fluid intake

The nurse plans to assess an adult client's kidneys for tenderness. The nurse should assess the area at the

costovertebral angle

The nurse plans to assess an adult client's kidneys for tenderness. The nurse should assess the area at the:

costovertebral angle.

A loss of taste discrimination occurs with a defect of

cranial nerve VII (facial).

The palate fails to rise and the uvula deviates to the side with

cranial nerve X (vagus) paralysis.

Define: thickened, dried-out exudate left when vesicles/pustules burst or dry up. Color can be red-brown, honey, or yellow, depending on fluid ingredients (blood, serum, pus). (impetigo, scab, weeping eczematous dermatitis)

crust

Define: A process that requires humility as individuals continually engage in self-reflection and self-critique as lifelong learners and reflective practitioners.

cultural humility

Define: A deliberative proactive behavior by healthcare providers who examine cultural situations through thoughtful reasoning, responsiveness, and discreet (attentive, considerate, and observant) interactions.

cultural sensibility

what syndrome is seen with darkish bluish pink striae marks associate with?

cushings syndrome

What does CAGE stand for when assessing alcohol use?

cutdown, annoyed, guilty, eye opener

Name this part of the nail: works like a gasket to cover and protect the nail

cuticle

Define: skin turning blue due to lack of oxygen

cyanosis

Define: encapsulated fluid filled cavity in dermis or subcutaneous layer; tensely elevating skin (sebaceous ____)

cyst

The nurse has tested the near visual acuity of a 45-year-old client. The nurse explains to the client that the client has impaired near vision and discusses a possible reason for the condition. The nurse determines that the client has understood the instructions when the client says that presbyopia is usually due to

decreased accommodation. Presbyopia (impaired near vision) is indicated when the client moves the chart away from the eyes to focus on the print. It is caused by decreased accommodation.

what sound signifies the absence of bowel motility that could occur after surgery but should resume gradually

decreased or absent bowel sounds

The nurse is planning to assess a client's abdomen for rebound tenderness. The nurse should palpate lightly while slowly releasing pressure. palpate deeply while quickly releasing pressure. ask the client to assume a side-lying position. perform this abdominal assessment first.

deep and quick

The pancreas of an adult client is located

deep in the upper abdomen and is not normally palpable.

The pancreas of an adult client is located posterior to the left midaxillary line and posterior to the stomach. below the diaphragm and extending below the right costal margin. high and deep under the diaphragm and can be palpated. deep in the upper abdomen and is not normally palpable.

deep in the upper abdomen and is not normally palpable.

chloasma (mask of pregnancy, striae)

developmental skin variations: pregnancy

Define: subjectivity of pain, education level, cultural view, gender expectations

developmental, cross-cultural, and gender considerations for verbal and nonverbal pain assessment

When using the ABCDE criteria for assessment of a mole, the nurse understands that which criteria could indicate a melanoma? (Select all that apply.)

diameter great than 6 cm asymmetry notched border

During a physical examination of an adult client, the nurse is preparing to auscultate the client's abdomen. The nurse should begin auscultation in the left upper quadrant. listen in each quadrant for 15 seconds. palpate the abdomen before auscultation. use the diaphragm of the stethoscope.

diaphragm

what can be an abnormal finding in someone with vascularity in their abdomen

dilated veins may be seen with cirrhosis of the liver, obstruction of the inferior vena cava, portal hypertension and ascites

Define: excessive bruising

ecchymosis

occur all over the body, open directly onto the surface of the skin

eccrine sweat glands

Define: excess fluid in interstitial spaces that is usually localized (due to injury) or systemic (often in dependent portions of the body) and causes skin to look puffy and feel tight

edema

During examination of the oral cavity, the nurse examines the salivary glands. Which area of the mouth should the nurse assess to inspect for the Wharton's ducts?

either side of the frenulum on the floor of the mouth. Stenson's ducts, not Wharton's ducts, are visible on the buccal mucosa across from the second upper molars. The right sides of the frenulum at the base of the gums and on the posterior aspect of the tongue bilaterally are not appropriate to inspect salivary ducts.

Define: responses may be as simple as "I understand", "that sounds upsetting", or "you seem sad" may also be non-verbal such as a gentle touch

empathy

A woman consumes pasta, grains, and other carbohydrates for

energy

When assessing heart rate bilaterally, what quality are you assessing?

equality

Define: superficial reddening of the skin, usually in patches, as a result of injury or irritation causing dilatation of the blood capillaries.

erythema

While assessing the eye of an adult client, the nurse observes an inward turning of the client's left eye. The nurse should document the client's

esotropia. Esotropia is an inward turn of the eye.

The correct technique to _____________________________________ is to use a square gauze pad to hold the client's tongue to each side.

examine the sides of the tongue

Name this artery

external carotid artery

Define: Ensure privacy, refuse interruptions, physical environment, dress, note-taking, tape and video recording

external factors

What condition is this? peripheral paralysis of the facial nerve (CNVII). Maybe due to injury at birth, inflammation of the middle ear branch of nerve from otitis media, or unknown causes (bells palsy). Full recovery occurs in 90% of cases.

facial palsy

Define: improper bp cuff size, improper positioning, age, sex, race, weight, diurnal rhythm, emotions, stress, exercise

factors that affect blood pressure

What abnormal physical response should the nurse be prepared to manage after noting pallor in a client?

fainting Pallor results from decreased redness in anemia and decreased blood flow, as occurs in fainting or arterial insufficiency. None of the remaining options present responses directly associated with pallor.

may indicate hyperthyroidism

fine, silky hair

Define: linear crack with abrupt edges; extends into dermis; dry or moist. (athlete's foot, cheilosis (corners of mouth from excess moisture))

fissure

An adult male client visits the clinic and tells the nurse that he believes he has athlete's foot. The nurse observes that the client has linear cracks in the skin on both feet. The nurse should document the presence of

fissures

A client has osteoarthritis of the elbow. Which assessment approach should the nurse expect to be impacted by this health problem?

flexion

On inspection of the abdomen, a nurse notes that the client's skin appears pale and taut. The nurse recognizes that this finding is most likely due to what process occurring within the abdominal cavity? Bleeding Inflammation Obstruction Fluid accumulation

fluid accumulation

Which of the following people need to be vaccinated for hepatitis A and B?

food service workers

a palpable vibration from the spoken voice felt over the chest wall- look for symmetry -use ulnar surface of hand

fremitus

a vibration of air in the bronchial tubes · Use ball or ulnar edge of one hand and palpate in all of the locations asking the client to say "99"- assess for symmetry and vibration Normal: decrease in the vibration towards the base of the lungs Abnormal: unequal tactile fremitus: consolidation; no vibrations

fremitus

Name this cranial bone

frontal

The nurse should suspect the client of having diabetic ketoacidosis if there is a __________ smell on the breath.

fruity

An older adult client who is admitted to the hospital with acute confusion has urinary incontinence. The nurse can accurately document this as which type of incontinence?

functional

A client with insulin-dependent diabetes visits the clinic and complains of painful hip joints. The nurse should assess the client carefully for signs and symptoms of

gait difficulties. *osteomyelitis.* scoliosis. arthritis. Exp: Having diabetes mellitus, sickle cell anemia, or SLE places the client at risk for development of musculoskeletal problems such as osteoporosis and osteomyelitis. P535

Define: Objective data that covers the general health state and obvious physical characteristics. It is an introduction for physical examination and helps to give an overall impression.

general survey

Equipment for an Integumentary assessment includes?

gloves penlight magnifying glass centimeter ruler gown, drape braden scale- predicting pressure ulcer risk PUSH scale- measure pressure ulcer healing

Define: What kind of shape/configuration of lesions is characterized by clusters of lesions (EX. contact dermatitis)

grouped

A nurse is instructing a client who suffers from peptic ulcer disease about the causes of this condition. Which of the following should the nurse mention as a common bacterial cause? Helicobacter pylori Escherichia coli Staphylococcus aureus Streptococcus pyogenes

h pylori

The nurse is planning to assess a client's near vision. Which technique should be used?

have the client read newspaper print held 14 inches from the eyes Near vision is tested by asking the client to read newspaper print held 14 inches from the eyes. Shining a light on the bridge of the nose tests the corneal light reflex. Moving the eyes in the direction of a moving finger tests for extraocular movements. Having the client read letters on a wall chart tests for central and distance vision.

Define: term for questions about personal habits, environmental/occupational hazards, safety measures, and intimate partner violence or abuse

health maintenance

Define: At what stage of the complete health history do you review self-esteem/self-concept, value-belief system, activity/exercise, sleep/rest, nutrition, interpersonal role, coping and stress management, environment/home safety hazards, and ADL/IADLs?

health patterns

To appropriately document ______ you would use a well-mounted or measuring pole on a scale (stadiometer). Make sure the patient is standing up straight with their body against the wall. After you get the readings in inches, then you will convert that into feet and document.

height

Vesicular lesions

herpes simplex, varicella, impetigo

What condition is this? obstruction of drainage of cerebrospinal fluid results in excessive accumulation, increasing intracranial pressure, and enlargement of the head

hydrocephalus

what is the term for an enlarged kidney

hydronephrosis

What condition do these symptoms represent? fatigue, weight loss without change in appetite, heat intolerance, depression, anxiety, irritability, menstrual irregularities, weakness, tremor, palpitations, anterior neck pain, insomnia

hyperthyroidism

what sound indicates decreased motility -seen with paralytic ileus after abdominal surgery and inflammation of the peritoneum, or late bowel obstruction

hypoactive bowel sounds

A 20-year-old client visits the outpatient center and tells the nurse that he has been experiencing sudden generalized hair loss. After determining that the client has not received radiation or chemotherapy, the nurse should further assess the client for signs and symptoms of

hypothyroidism

A client has an edematous face, hands, and legs. Which health problem should the nurse suspect this client is experiencing? scleroderma hypothyroidism hyperthyroidism Cushing's syndrome

hypothyroidism

Generalized hair loss may be seen in various systemic illnesses such as ___________________________________ and in clients receiving certain types of chemotherapy or radiation therapy.

hypothyroidism

What condition do these symptoms represent? fatigue, weight gain, cold intolerance, depression, menstrual irregularities, weakness, muscle cramps, joint pains, constipation, hoarseness, hypersomnolence

hypothyroidism

A client describes a 3-week history of hoarseness. The client also reports feeling fatigue and noticeable weight gain over the past month. Which cause should the nurse consider as most likely?

hypothyroidism Hoarseness lasting longer than two weeks accompanied by the additional reported symptoms of fatigue and weight gain suggest hypothyroidism. Gingivitis is an inflammation of the gums that often results in bleeding. Dysphagia is a medical term used to describe difficulty swallowing. This can be due to a neuromuscular or oropharyngeal deficit. Hoarseness can accompany dysphagia; however, weight gain and fatigue do not. Apthous ulcers are a common condition that are restricted to the oral cavity. Commonly called "canker sores," they may be associated with autoimmune problems that create a predisposition to developing oral ulcers.

Which function of the skin does this describe: people identify one another by unique combinations of facial characteristics, hair, skin color, and even fingerprints

identification

The nurse is assessing a client's abdomen. For which reason should the nurse perform deep palpation? identify abdominal organs detect abdominal tenderness discern muscular resistance complete a surface evaluation

identify abdominal organs deep palpation is to identify the organs , light is to identify tenderness

Identify the 8 components of the complete health history

identifying data, source of history, reason for seeking care, history of present illness, past health, family history, review of systems, health patterns

what age group experiences: Abdominal respirations Louder breath sounds Irregular rhythm with periods of apnea

infants

A child presents to the health care facility with new onset of a foul smelling, purulent drainage from the right nare. The mother states no other signs of an upper respiratory tract infection are present. What is an appropriate action by the nurse?

inspect nostrils with otoscope Because the drainage is unilateral, the most likely cause is a foreign body obstruction. He nurse should inspect the nostrils for patency and the presence of a foreign body. It is not a normal finding in children to have unilateral foul smelling drainage from the nose. This child will not need an antibiotic, so the nurse does not need to assess for allergies to medication. Blowing the nose may or may not dislodge the object and may cause further trauma to the nare.

what kind of inspection looks at color, condition, angle of attachment, abnormalities?

inspections of nails

Pressure ulcers are staged as I through IV. Put the following in order from stage I through stage IV.

intact, firm skin with redness ulceration involving the dermis full-thickness skin loss necrosis with damage to underlying muscle

The nurse is assessing the abdomen of an adult client and observes a purple discoloration at the flanks. The nurse should refer the client to a physician for possible liver disease. internal bleeding. abdominal distention. Cushing syndrome.

internal bleeding

Name this artery

internal carotid artery

Define: Liking others, ability to listen, empathy

internal factors

Name this vein

internal jugular vein

While assessing the abdominal sounds of an adult client, the nurse hears high-pitched tingling sounds throughout the distended abdomen. The nurse should refer the client to a physician for possible

intestinal obstruction

At what stage of the interview do you greet the patient and establish rapport, establish the agenda, ask open-ended questions, and identify all concerns?

introduction

what do you assess for when performing a light palpation

involuntary guarding indicates peritoneal irritation when a patient starts flinch

A patient with a zosteriform rash has a rash that

is distributed along a dermatome A zosteriform rash is distributed along a dermatome.

Define: yellowing of the skin and eyes due to excess bilirubin- usually due to liver disease

jaundice

A nurse observes tenderness over the costovertebral angle on the right side. The nurse recognizes this as an abnormal finding for which organ?

kidney

The nurse is assessing an elderly client and finds an exaggerated thoracic curve. This would be documented as what?

kyphosis

When the client reports a problem associated with the drainage of tears from the left eye, the nurse would focus the eye assessment on which eye structure?

lacrimal puncta The lacrimal gland lies mostly within the bony orbit, above and lateral to the eyeball. The tear fluid spreads across the eye and drains medially through two tiny holes called lacrimal puncta. The nurse can easily find a punctum atop the small elevation of the lower lid medially. The tears then pass into the lacrimal sac and into the nose through the nasolacrimal duct.

Where in the digestive tract is most of the water absorbed? Stomach Duodenum Large intestine Ileum

large intestines

Define: prolonged, intense scratching eventually thickens the skin and produces tightly packed sets of papules; looks like surface of moss (atopic dermatitis)

lichenification

A nurse is inspecting the abdomen of a young, fit client who has well-defined abdominal muscles. The nurse recognizes the vertical line that appears in the center of the client's abdomen as which of the following? Transverse abdominis Internal abdominal oblique Peritoneum Linea alba

linea alba

what organs are located in the RUQ " LGP"

liver gall bladder (not palpable) pancreas ( not palpable)

what organs are considered solid viscera

liver pancreas spleen adrenal glands kidney ovaries uterus

side effects of nausea and vomitting can also be seen in patients with

liver or pancreas disorders

what sounds does the bell of the stethoscope pick up?

low- pitched sounds, S3 and S4

Name this part of the nail: white, opaque, semilunar area at the proximal end of the nail. It lies over the nail matrix where new keratinized cells are formed. The nail folds overlap the posterior and lateral borders.

lunula

A patient complains of feeling like he is slowly losing his central vision. The nurse knows this symptom could represent

macular degeneration

An older client asks why vision is not as sharp as it used to be when the eyes are focused forward. What should the nurse realize this client is describing?

macular degeneration Macular degeneration causes a loss of central vision. Risk factors for macular degeneration are age, smoking history, obesity, family history, and female gender. Cataracts are characterized by cloudiness of the eye lenses. Glaucoma is an increase in intraocular pressure that places pressure on eye structures and affecting vision. A detached retina is the sudden loss of vision in one eye. This health problem may be precipitated by the appearance of blind spots.

flat non-palpable, less than 1 cm (ex: freckle)

macule (primary)

When documenting that a patient has freckles, the appropriate term to use is?

macules

While assessing an adult client, the nurse observes freckles on the client's face. The nurse should document the presence of

macules Freckles are flat, small macules of pigment that appear following sun exposure.

As the nurse palpates the lymph nodes of the neck, hard and fixed nodes are noted in the supra-clavicular region. This finding is consistent with which condition? A. malignancy B. inflammation C. enlargement D. hypothyroidism

malignancy

What does a horizontal line between two people mean in a genogram?

married

While assessing an adult client's abdomen, the nurse observes that the client's umbilicus is deviated to the left. The nurse should refer the client to a physician for possible gallbladder disease. cachexia. kidney trauma. masses.

masses

The nurse is preparing to examine the sinuses of an adult client. After examining the frontal sinuses, the nurse should proceed to examine the

maxillary sinuses The frontal sinuses (above the eyes) and the maxillary sinuses (in the upper jaw) are accessible to examination by the nurse.

what are the side effects of having abdominal surgery

may have adhesions which can cause complications

Define: pressure forcing blood into tissues

mean arterial pressure

A nurse cares for a client with a distended abdomen due to peritonitis. Which parameter should the nurse measure to assess improvement? Palpate the abdomen Perform percussion for tympany Measure abdominal girth Auscultate for bowel sounds

measure abdominal girth

what do you do if the abdomen is distended

measure the girth

The nurse assesses a client with noisy breathing including a gasping sound with respiration. The nurse notes tracheal deviation from the usual midline position. Which conditions should the nurse assess for further? Select all that apply. mediastinal mass atelectasis pneumothorax goiter inflammation of preauricular node

mediastinal mass atelectasis pneumothorax goiter

where is the bladder located

midline, behind the pubic bone

What type of headache has these characteristics: pain, nausea, and visual changes are typical of classical form

migraine

PHQ-9 score of 5-9 indicates?

mild- watchful waiting

PHQ- 9 score of 10-14 indicates?

mild/moderate - pt education, counseling, active treatment

what unit is used to stage edema?

millimeters

left, 5th, intercostal space, mid clavicular line (point of max impulse)

mitral area/ apex landmark

PHQ-9 score of 15-19 indicates?

moderate- active treatment for most

palpation of the skin includes?

moisture temperature texture turgor surface characteristics

the way the person feels

mood

Name this part of the nail: clear, with fine longitudinal ridges that become prominent in aging. Nails get their pink color from the underlying nail bed of highly vascular epithelial cells

nail plate

is it normal to feel lymph nodes?

no, examiners should not be able to feel lymph nodes

Define: solid, elevated, hard or soft over 1cm. May extend deeper into dermis than papule (xanthoma, intradermal nevi)

nodule

An edema where there is a local infection or trauma that cannot be indented easily by compression is called

nonpitting (brawny)

A BP with a systolic <120 and diastolic <80 can be described as ______________

normal

How would you describe a heart rate between 60-100bpm?

normal

what does a pulse strength of 2+ mean?

normal (obliterate with moderate pressure)

PHQ-9 score of 0-4 indicates?

normal - no action

what kind of finding is this? no blowing or swishing sounds

normal finding - carotid pulse

Define: Observable or measurable data or signs obtained through observations, physical examination, and testing

objective data

information that is seen, heard, felt, or smelled by an observer; signs

objective data

The nurse is planning to perform an ear examination on an adult client. After explaining the procedures to the client, the nurse should a) observe the client's response to the explanations. b) show the client the otoscope. c) ask the client to remove his contact lenses. d) ask the client to remain standing.

observe the client's response to the explanations. Explanation: As you prepare the client for the ear examination, carefully note how the client responds to your explanations

what can pencil shaped stools mean

obstruction

client opens eyes to loud voice, responds slowly with confusion, and seems unaware of environment (might need to shout)

obtunded

During the abdominal examination a nurse supports the client's right knee and ankle. The nurse flexes the client's hip and rotates the leg externally and internally. At this point, the client reports pain in the right lower quadrant. This test is positive for which sign? Rovsing's Murphy's Obturator Psoas

obturator

During the abdominal examination, a nurse supports the client's right knee and ankle. The nurse flexes the client's hip and rotates the leg externally and internally. At this point, the client reports pain in the right lower quadrant. This test is positive for which sign?

obturator

what test is done when assessing pain in the RLQ , when, the right hip and knee is flexed and rotated internally and externally indicating irritation fo the obturator muscle due to appencititis or a perforated appendix

obturator sign

Name this cranial bone

occipital

Describe parietal pain?

occurs when the parietal peritoneum becomes inflamed, as in appendicitis and peritonitis

What does OLDCHARTS stand for?

onset, location, duration, character, history, associated manifestations, relieving or exacerbating factors, treatment, severity

What does OPQRSTU stand for?

onset, provocative/palliative, quality, region/radiation, severity, timing, understanding

Which instruction to the client will help facilitate examination of the temporomandibular joint by the nurse? Ask the client to: A. open the mouth B. sit upright C. sit without moving D. perform a chewing action

open the mouth

The optic nerves from each eyeball cross at the

optic chiasma. At the point where the optic nerves from each eyeball cross—the optic chiasma—the nerve fibers from the nasal quadrant of each retina (from both temporal visual fields) cross over to the opposite side.

Which temperature route does this statement describe: Accurate and convenient

oral

What are the various routes of taking temperature?

oral, axillary, rectal, tympanic, temporal

Define: a decrease in blood pressure that happens soon after standing or sitting up. When a person stands up, gravity causes blood to pool in the legs. People at risk are those on diuretics, who have had a heart attack, with anemia, or due to abrupt standing, multiple meds, dehydration and low bp

orthostatic hypotension (orthostasis)

A female client visits the clinic and tells the nurse that she began menarche at the age of 16 years. The nurse should instruct the client that she is at a higher risk for

osteomyelitis. *osteoporosis.* lordosis. rheumatoid arthritis. Explanation: Women who begin menarche late or begin menopause early are at greater risk for development of osteoporosis because of decreased estrogen levels, which tend to decrease the density of bone mass. P535

The nursing instructor is discussing the administration of nasal spray with the nursing students. What information is most important to include in this discussion?

overuse of nasal spray may cause rebound congestion The use of topical decongestants is controversial because of the potential for a rebound effect. The patient should hold his or her head back for maximum distribution of the spray. Only the patient should use the bottle.

activation of the atria, artial depolarization

p wave

What condition is this? localized bone disease of unknown origin that softens, thickens, and deforms bone. Affects 3% of adults over 40 and 10% of adults over 80 and occurs more often in males.

paget's disease

what are the 5 P's?

pain pallor (pale color) parathesia (pins and needles) pulselessness paralysis

position, texture, size, consistency, fluid, creptus

palmar

what are the three hand surfaces?

palmar ulnar dorsal

The nurse is planning to assess a client's abdomen for rebound tenderness. The nurse should

palpate deeply while quickly releasing pressure.

use of hands and fingers to gather information through sense of touch

palpation

look for scalp tenderness, masses, hair texture (ex: dry/course- consider hypothyroidism- fine/silky- hyperthyroidism) ** consider whats normal for the patient, has there been any changes?

palpation (hair)

what organs are located in the LUQ

pancrea spleen stomach small intestine ** all not palpable

what are some causes of abdominal pain

pancreatis food stomach ulcers

Define: something you can feel. Solid, elevated, circumscribed, less than 1cm. Caused by superficial thickening in epidermis. (mole, wart)

papule

The nurse is assessing the bowel sounds of an adult client. After listening to each quadrant, the nurse determines that bowel sounds are not present. The nurse should refer the client to a physician for possible fluid and electrolyte imbalances. aortic aneurysm. paralytic ileus. gastroenteritis.

paralytic ileus

what is severe, steady pain when the parietal peritoneoum (orgnans) becomes inflammed

pareital

What does a vertical line between two people mean in a genogram?

parent, child

Name this cranial bone

parietal

While assessing an adult client's head and neck, the nurse observes asymmetry in front of the client's ear lobes. The nurse refers the client to the physician because the nurse suspects the client is most likely experiencing a/an enlarged thyroid. lymph node abscess. neurologic disorder. parotid gland enlargement.

parotid gland enlargement.

Define: at what stage of the comprehensive health history do you ask about allergies, medications, childhood illnesses, adult illnesses, and health maintenance

past medical history (past health)

Define: macules that are over 1cm. (mongolian spot, vitiligo, measles)

patch

flat non-palpable greater than 1 cm (ex: birthmark)

patch (primary)

Define: patients who are self-confident and understand the recommendations are most likely to adopt offered advice, make lifestyle changes, or take medications as prescribed

patient empowerment

what is a dull or burning pain or indigestion between breasts and above the umbilicus

peptic ulcer

Which function of the skin does this describe: sensory surface holding the neurosensory end-organs for touch, pain, temperature and pressure

perception

tough, inextensible, loose fitting, fibro-serous sac that attached to the great vessels and surrounds the heart

pericaridum

The purpose of testing capillary refill by depressing the nail edge and releasing, noting the return of color is to test for __________ ___________

peripheral circulation

what else slows down in elderly patient

peristalsis slows, causing constipation

pinpoint, round spots that appear on the skin as a result of bleeding

petechiae

What component of the general survey describes age, sex, level of consciousness, skin color, facial features, and signs of distress.

physical appearance

Identify the 4 components of the general survey (based on the powerpoint)

physical appearance, body structure, mobility, behavior

________ _______ that may occur because of poorly controlled pain: disability, impairment, disruption of ADLs, abnormal vital signs, nonverbal cues such as grimacing, wincing, restlessness, etc.

physical changes

temporary increase in blood flow pregnancy, anemia

physiological murmurs

When assessing hydration, the nurse will

pinch a fold of skin just below the midpoint of one of the clavicles and allow the skin to recoil to normal

An edema where the interstitial fluid is mobile is described as

pitting

A client's abdominal muscles are tense when lying supine for an abdominal assessment. What should the nurse do to ensure the client's comfort during the assessment?

place a small pillow under the clients knee

The nurse is preparing to examine an adult client's eyes, using a Snellen chart. The nurse should

position the client 609.6 cm (20 ft) away from the chart Used to test distant visual acuity, the Snellen chart consists of lines of different letters stacked one above the other. The letters are large at the top and decrease in size from top to bottom. The chart is placed on a wall or door at eye level in a well-lighted area. The client stands 20 feet from the chart and covers one eye with an opaque card (which prevents the client from peeking through the fingers). Then the client reads each line of letters until he or she can no longer distinguish them.

Name this lymph node (9)

posterior cervical

Name this area

posterior triangle

At what stage of the interview do you self-reflect, review medical and nursing records, set goals for the interview, review clinical behavior and appearance, and adjust the environment?

pre-interview

What are the 4 phases of the Interview?

pre-interview, introduction, working, termination

Name this lymph node (1)

preauricular

The nurse is planning to assess a client's lymph nodes. Which set of nodes should the nurse assess first? A. submental B. preauricular C. supraclavicular D. superficial cervical

preauricular

what can cause?: increase tidal volume diaphragm rises and costal angle widens to accommodate the enlarging uterus SOB and dyspena (especially in 3rd trimester)

pregnancy

When assessing the skin- linea nigra, chloasma, and striae gravidum are normal findings in what population?

pregnant women

A BP with a systolic 120-139 and diastolic 80-89 can be described as _______________

prehypertension

Braden Scale

pressure ulcer risk assessment - sensory perception, moisture, activity, mobility, nutrition, and friction and shear. Scores of 14-18 indicates a high risk for pressure ulcers

Partial thickness (some skin loss or blistering)

pressure ulcer stage 2

These are ways to _______ _____: -review medication lists and assess for side effects like fainting and dizziness -avoid throw rugs, remove excess objects from high traffic areas -wear well-fitting, non-slip shoes

prevent falls

changes in color or texture that are generally present at birth or acquired over time, such as a birthmark or an age spot de novo most characteristic of the disease process

primary lesions

The nurse notes that a client's nails are greater than a 160-degree angle. What should the nurse assess as a priority for this client?

pulse oximetry A nail angle greater than 160 degrees indicates clubbing which is caused by chronic hypoxia. Measuring the client's pulse oximetry would be a priority. Heart sounds, bowel sounds, and body temperature will not provide information to determine the cause for the clubbed nails.

Which instructions should the nurse provide to the client taking a sublingual medication?

put the medication underneath the tongue The highly vascular floor of the mouth is a good location for absorption of sublingual medications. The client should be taught to place the medication under the tongue for best absorption.

What 4 qualities should you assess for respirations?

rate, rhythm, depth, effort

what is a blumberg sign?

rebound tenderness

what kind of test is being done when you apply deep palpation away from the painful area and when the pressure is released (blumbergs sign) it will indicate appendicitits

rebound tenderness

Which temperature route does this statement describe: Good for those who cannot close their mouth or are unconscious but is uncomfortable

rectal

what is pain that travels or refers from the primary site and becomes localized at the distant site

reffered pain

A client reports using pain medication and sitting in a dark room on the onset of a migraine headache. In which part of the subjective section of the physical examination should the nurse document this information? A. onset B. location C. treatment D. relieving factors

relieving factors

Define: A system of beliefs or a practice of worship

religion

Photoreceptors of the eye are located in the eye's

retina. The innermost layer, the retina, extends only to the ciliary body anteriorly. It receives visual stimuli and sends it to the brain. The retina consists of numerous layers of nerve cells, including the cells commonly called rods and cones. These specialized nerve cells are often referred to as "photoreceptors" because they are responsive to light

The nurse should ask about or assess which associated factors when a patient complains of cluster headaches? Select all that apply.

rhinorrhea ptosis miosis lacrimation

To palpate for tenderness of an adult client's appendix, the nurse should begin the abdominal assessment at the client's

right lower quadrant

To palpate for tenderness of an adult client's appendix, the nurse should begin the abdominal assessment at the client's left upper quadrant. right upper quadrant. right lower quadrant. left lower quadrant.

right lower quadrant.

When palpating the abdomen, the nurse may be able to feel the lower edge of the liver in which quadrant?

right upper

When palpating the abdomen, the nurse may be able to feel the lower edge of the liver in which quadrant? right lower left lower left upper right upper

right upper

The nurse is caring for a client suffering from liver damage. The nurse is most likely to palpate the border of the enlarged liver in which abdominal quadrant?

right upper quadrant

To percuss the liver of an adult client, the nurse should begin the abdominal assessment at the client's

right upper quadrant.

what does the branden scale determine? what are the 6 elements of the scale?

risk for skin breakdown 1. sensory perception (respond to pressure related discomfort) 2. moisture (degree to which skin is exposed to moisture) 3. activity (degree of physical activity) 4. mobility (change or control body position) 5. nutrition (usual food intake) 6. friction and shear ** know the 6

While assessing the range of motion in an adult client's shoulders, the client expresses pain and exhibits limited abduction and muscle weakness. The nurse plans to refer the client to a physician for possible

rotator cuff tear

Define: desiccated flakes of skin, dry or greasy, silvery or white, from shedding of dead excess keratin cells (psoriasis, drug reaction)

scale

Define: after a skin lesion is repaired, normal tissue is lost and replaced with connective tissue (collagen) (acne, healed area after surgery)

scar

What gland is this: secretes a protective lipid substance called sebum, which is secreted through the hair follicles and lubricates the skin and hair

sebaceous

Define: excessive moisture/oiliness

seborrhea

Define: clusters of melanocytes after sun exposure

senile lentigines (liver spots)

what happens to elderly patients tolerance to pain

sensitivity to pain decrease with aging

When viewing the index fingers at their profile, they should be about 160 degrees. What are you looking for when you do observe this?

shape, contour

Define: objective abnormality, detectable on physical exam or in laboratory reports

sign

What type of headache has these characteristics: pain is usually behind the forehead and/or cheekbones

sinus

The nurse is performing the assessment shown. What is the nurse assessing in this client?

sinuses Transillumination is a traditional method by which clinicians assess sinus cavities. This method has limited clinical significance and provides inconsistent results. This technique is not used to assess the nasal turbinates, the septum, or the sense of smell.

most of the exam done in this position; auscultation of heart and lungs; HEENT exam; parts of neurological exam

sitting position

what are organs that maintain their shape

solid viscera

Define: Recording who furnishes the information during the interview, judging the reliability of the informant, and noting special circumstances such as the use of an interpreter.

source of history

Define: All behaviors that give meaning to life and provide strength to the individual

spirituality

The nurse understands this abdominal organ is responsible for storing red blood cells and platelets.

spleen

Which organ that resides in the abdominal cavity stores red blood cells and platelets, produces new red blood cells and macrophages, and activates B and T lymphocytes? Spleen Gallbladder Liver Pancreas

spleen

skin cancer that begins in the epidermis but may grow into deeper tissue; does not generally metastasize to other areas of the body usually non - invasive

squamous cell carcinoma (SCC)

What is the symbol for a male in a genogram?

square

For people with diabetes, renal disease, or age 60+ a BP with a systolic 150-159 and diastolic 90-99 can be described as

stage I hypertension

A pressure ulcer with these signs is at what stage: partial-thickness skin erosion with loss of epidermis or also the dermis

stage II

The nurse assesses an older adult bedridden client in her home. While assessing the client's buttocks, the nurse observes that a small area of the skin is broken and resembles an erosion. The nurse should document the client's pressure ulcer as

stage II. Stage II pressure ulcer is a partial thickness loss of dermis presenting as a shallow open ulcer with a red-pink wound bed, without slough. May also present as an intact or open/ruptured, serum-filled blister. Presents as a shiny or dry shallow ulcer without slough or bruising; bruising indicates suspected deep tissue injury. This stage should not be used to describe skin tears, tape burns, perineal dermatitis, maceration, or excoriation.

A pressure ulcer with these signs is at what stage: full-thickness pressure ulcer extending into the subcutaneous tissue. May see subcutaneous fat but not muscle, bone or tendon.

stage III

A pressure ulcer with these signs is at what stage: full-thickness pressure ulcer involves all skin layers and extends into supporting tissue. Exposes muscle, tendon, or bone.

stage IV

typical position for exam of male genitals & inguinal hernia

standing position

In regards to the skin hair and nails, this would be considered what type of information? -previous history of skin disease -change in mole -change in pigmentation (size and color) -excessive dryness (xerosis) or moisture (seborrhea) -pruritus (itching) -excessive bruising (ecchymosis) -rash or lesion migration -hair loss -systemic conditions -change in nails -environmental/occupational hazards -self-care behaviors

subjective

Name this lymph node (5)

submandibular

Name this lymph node (4)

submental

Clients with end-stage liver disease have a ___________ odor in their breath.

sulfur

Define: it communicates to the patient that you have been listening carefully

summary

Name this lymph node (7)

superficial cervical

what position is used for palpation of abdominal organs, cardiac exam?

supine position

Name this lymph node (10)

supraclavicular

The nurse is assessing a client with a bladder disorder. Where would the nurse expect the pain to be?

suprapubic

The nurse is preparing to assess the neck of an adult client. To inspect movement of the client's thyroid gland, the nurse should ask the client to

swallow a small sip of water.

Define: subjective sensation person feels from disorder

symptom

Define: maximum pressure felt on artery during left ventricular contraction

systolic pressure

A nurse suspects that a client has gastroesophygeal reflux disease (GERD). Which risk factors must be present for the nurse to confirm this? Select all that apply.

taking multiple medications hiatal hernia

Which function of the skin does this describe: allows heat dissipation through sweat glands and heat storage through subcutaneous insulation

temperature regulation

Name this cranial bone

temporal

Which temperature route does this statement describe: must take multiple readings for accuracy

temporal (temp)

Name this artery

temporal artery

A client complains of a headache over both temporal areas. What type of headache should the nurse suspect the client is experiencing? A. cluster B. tension C. migraine D. hypertensive

tension

What type of headache has these characteristics: pain is like a band squeezing the head

tension

What type of hair is this: thick, coarse, pigmented

terminal

At what stage of the interview do you signal the end of the interview, ask if the patient has any questions, summarize the patient's problems, and review the plan of care?

termination

what area do you percuss the liver

the RLQ at the MCL upwards till you hear dullness

what is the purpose of a deep palpation

to assess for severe tenderness, or pain that may be peritonitis or an infection, tumor, diseases liver or gallbladder

An upright position in which the patient leans forward onto two arms stretched forward and thrusts the head and chin forward. considerable amount of work is required to expire air, forced expiratory volume decreases substantially- increases residual volume impairs gas exchange

tripod position

When the spleen enlarges, the nurse would not be surprised to percuss dullness over the stomach.

true

Define: larger than a few centimeters in diameter, soft or firm, deeper into dermis; may be benign or malignant. (lipoma)

tumor

An older client visits the clinic accompanied by his daughter. The daughter tells the nurse that her father has been experiencing severe headaches that usually begin in the morning and become worse when he coughs. The client tells the nurse that he feels dizzy when he has the headaches. The nurse refers the client for further evaluation because these symptoms are characteristic of a A. migraine headache. B. cluster headache. C. tension headache. D. tumor-related headache.

tumor-related headache.

Mr. Maxwell has noticed that he is gaining weight and has increasing girth. Which of the following would argue for the presence of ascites? Dullness that remains despite change in position Dullness centrally when the client is supine Tympany that changes location with client position Bilateral flank tympany

tympani that changes location with the client position

Which temperature route does this statement describe: Speed, convenience, safety, and noninvasiveness

tympanic

Define: deeper depression extending into dermis, irregular shape; may bleed; leaves scar when heals (stasis ulcer, pressure sore, chancre)

ulcer

chronic use of antacids/ H2 blocers (protonix, mylanta, tums) can lead to what kind of stomach problems?

ulcers

An adult client tells the nurse that his father had cataracts. He asks the nurse about risk factors for cataracts. The nurse should instruct the client that a potential risk factor is

ultraviolet light exposure. Exposure to ultraviolet radiation puts the client at risk for the development of cataracts (opacities of the lenses of the eyes). Consistent use of sunglasses during exposure minimizes the client's risk.

While assessing an adult client's abdomen, the nurse observes that the client's umbilicus is enlarged and everted. The nurse should refer the client to a physician for possible:

umbilical hernia.

In performing an otoscopic examination of a client's tympanic membrane, the nurse observes through the membrane the tip of a landmark at the center of the membrane. Which of the following landmarks is this? a) Pars flaccida b) Umbo c) Pars tensa d) Handle and short process of the malleus

umbo Explanation: The malleus is the nearest auditory ossicle that can be seen through the translucent membrane and includes a handle and short process, which are closer to the edge of the membrane, and the umbo, or base, which is a center point landmark of the tympanic membrane. Pars flaccida is the top portion of the membrane, which appears to be less taut than the bottom portion. Pars tensa is the bottom of the membrane, which appears to be taut.

A pressure ulcer with these signs is at what stage: pressure ulcer shows slough (stringy matter attached to wound bed) or eschar (black or brown necrotic tissue)

unstageable

The nurse assesses a client's indwelling urinary catheter bag and observes cloudy urine. The client also complains of lower back pain. What is the nurse's best action? Flush the catheter tubing with sterile normal saline. Record the findings as expected for a client with an indwelling catheter. Prepare to obtain a urine specimen for culture. Encourage the client to increase PO fluid intake.

urine specimen for culture

if a client has difficulty explaining their pain, whats another way to describe the symptoms

use descriptive words such as a laundry list- dull, sharp, stabbing, aching- use numeric scales

During a physical examination of an adult client, the nurse is preparing to auscultate the client's abdomen. The nurse should

use the diaphragm of the stethoscope.

To assess an adult client's skin turgor, the nurse should

use two fingers to pinch the skin under the clavicle. To assess turgor, gently pinch the skin over the clavicle with two fingers.

Define: make the patient feel affirmed to validate or acknowledge the legitimacy of the emotional experience

validation

Pain: aching, cramping Pulses: present but may be difficult to palpate through edema Skin: pigmentation in area of medial/lateral malleolus Thicken and tough Reddish to blue in color Associated with dermatitis Edema: moderate to severe

venous insufficiency

Define: elevated cavity containing free fluid, up to 1cm; a blister. Clear serum flows if wall is ruptured (herpes, chickenpox, shingles)

vesicle

A clear, fluid-filled, raised lesion; commonly called a blister, cyst, less than 1 cm (ex; herpes zoster)

vesicle (primary)

The nurse is preparing to examine the skin of an adult client with a diagnosis of herpes simplex. The nurse plans to measure the client's symptomatic lesions and measure the size of the client's

vesicles. Vesicles are circumscribed elevated, palpable masses containing serous fluid. Vesicles are less than 0.5 cm. Examples of vesicles include herpes simplex/zoster, varicella (chickenpox), poison ivy, and second-degree burn.

A client tells the nurse he has been having gray-colored stools after recent travel out of the country to an area with known poor sanitation. The nurse needs to investigate the possibility of which condition?

viral hepatitis

A client tells the nurse he has been having gray-colored stools after recent travel out of the country to an area with known poor sanitation. The nurse needs to investigate the possibility of which condition? alcohol hepatitis viral hepatitis intrahepatic jaundice toxic liver damage

viral hepatitis

A nurse is assessing a client with a history of alcohol abuse. The client reports right upper quadrant pain. Which type of pain is the client experiencing?

visceral

what type of pain is dull, aching, burning, cramping, or collicky sensations when the organs become distended or stretched

visceral pain

To measure ______ ____________ place the tape measure over bare skin just above the hip bones. Have the patient take a breath and measure the waist after exhalation. Record this number.

waist circumference

what does a pulse sound of 1+ mean?

weak, diminished (easy to obliterate)

The nurse is caring for an adult client who is in a cast because of a fractured arm. To promote healing of the bone and tissue, the nurse should instruct the client to eat a diet that is high in

whole grains. *vitamin C.* vitamin B. vitamin E. Exp: Adequate protein in the diet promotes muscle tone and bone growth; vitamin C promotes healing of tissues and bones. P536

At what stage of the interview do you identify the patient's story, identify and respond to emotional cues, gain clues to the patient's perspective of the illness, expand and clarify the story, generate hypotheses, create a shared understanding of the problem, and negotiate a plan based on a complete picture of the problem?

working

The risk for melanoma is increased in people older than?

50 years

Functionally, the most important range for hearing is the range of human speech. What is the range of human speech?

500 Hz to 3000 Hz

Transient Ischemic Attack:

A brief stroke-like attack; unilateral facial droop, arm drift, weakness or paralysis on one half of the body, difficulty speaking, and confusion

where is the epitrochlear lymph node?

located in the subcutaneous connective tissue on the medial aspect of the elbow, about 4-5 cm above the humeral epitrochlea

abnormal swishing sound caused turbulent blood flow through the heart or great vessels 1- 6 grade scale- 1: least worrisome 6: the loudest

murmur

You are assessing a patient for acute cholecystitis. What sign would you assess for? Psoas sign Cutaneous hyperesthesia Obstipation sign Murphy sign

murphy sign

what test is done for cholecystitis

murphys sign

what test is done when the RUQ exhibits sharp pain causing the client to hold their breath leading to inflammation of the gall bladder associated with acute cholecystitis

murphys sign

Myopia

nearsightedness

what position helps examine the spine and legs?

prone position

Define: excessive itching

pruritus

Used for rectal examination, enema admin, rectal temp. The patient is on his/her left side with right knee flexed against the abdomen and the left knee slightly flexed.

sim's position

What is the difference between solid viscera and hollow viscera?

solid retain their shape always like a liver, while hollow adjust to the contents inside like the stomach.

A BP that with a systolic 160+ and diastolic 100+ can be described as ___________

stage II hypertension

Name this duct

stensen's duct

Name this gland

sublingual gland

low pitched snoring, heard primarily during expiration, may clear with coughing. often head in bronchitis

wheeze (sonorant)

*Exophthalmos:

Protruding eyes

responses are closed, have no emotion

affect- flat

baldness, area of hair loss

alopecia

Temporal lobe:

Auditory reception center; alterations in hearing, taste, or smell

Scotomas

Blind spot (one black spot)

Rebound headaches:

Caused by overuse of medications to treat headaches

Signs of a stroke:

Change in mental status (confused/difficulty speaking), sudden onset of numbness/weakness, dizziness/trouble walking, blindness in one/both eye(s), "This is the worst headache of my life!"

what does COLDSPA stand for?

Character Onset Location Duration Severity Pattern Associated factors

which element of COLDSPA is the description on the problem?

Character (COLDSPA)

What are the 2 types of sweat glands?

Eccrine glands (sweat glands that release saline solution) Apocrine glands (produce adipose-like substance that is released in the axillae, nipples, anal/private part area, naval)

S1, the "lub" sound. What is happening?

Closure of the mitral and tricuspid valves, opening of the aortic and pulmonic valves

Cataracts:

Clouding of the lens of the eye

Inspect and palpate the skin for:

Color, moisture, temp., texture, thickness, mobility and turgor, edema, vascularity + bruising, lesions

Inspect and palpate hair (including body hair) for:

Color, quantity, distribution, texture, lesions, flaking, parasites

Inspect and palpate the nails for:

Color, shape, lesions, rounded in shape with a 160 degree angle (profile sign = clubbing)

Parietal lobe:

Focuses on sensation

Multiform lesions:

Confluent, target-like lesions

Pale conjunctiva:

Conjunctiva is pale color usually due to anemia; not enough blood is getting to the conjunctiva

Diplopia:

Double vision

*Ptosis:

Drooping eyelid (lazy eye)

what element of COLDSPA asks "how long does it last?"

Duration (COLDSPA)

Extra-ocular muscles test:

Follow finger in the 6 cardinal directions of vision

period between ventricular depolarization and the beginning of ventricular repolarization

EKG: ST segment:

ventricular repolarization, ventricles return to a resting state , recovery phase

EKG: T wave

Convergence:

EOM move eyes in closer together; pupils constrict; eyes moving in towards the nose using the EOM

The nurse should prioritize assessments related to overhydration for a client experiencing which health problem?

Early congestive heart failure

Places where patients are susceptible to pressure ulcers:

Ears, shoulder, elbow, hip, knee (outer and inner), ankles, heel

The linea alba is located where?

Middle of the ventral abdominal wall

Where is the linea alba located?

Middle of the ventral abdominal wall

Have patient state ___________ to test for speech and language issue?

No ifs, ands, or buts

In addition to noting the physical characteristics of the thyroid gland, which of the following signs would be most important to consider in determining if the client has hypothyroidism? A. Increased heart rate B. Increased blood pressure C. Laboratory tests D. Feeling anxious

Laboratory tests

Subarachnoid Hemorrhage:

Less common, but is due to an aneurysm (a ballooning and weakened area in an artery) between the base of the cerebral cortex and the arachnoid layer of the meninges

Example of what lesion? Acne, furuncles and carbuncles

Pustular lesions

A client with a body mass index of 28 tells the nurse she is concerned about her risk for hypertension. What can the nurse recommend to this patient?

Reducing her weight by 5% can lower her risk

Which of the following is consistent with obturator sign?

Right hypogastric pain with the right hip and knee flexed, and the hip internally rotated

The colon originates in this abdominal area: the

Right lower quadrant

Annular:

Ring/circular shape

Nails that are slightly curved or flat with smooth, rounded, and clean edges are normal findings. True or False?

True

Discrete:

Tend to remain separated from one another

The nurse is assessing a client complaining of swelling in the neck. While palpating the neck, the nurse finds a 2-cm lump that is fixed and hard. Why does this finding require emergency investigation?

This could be a sign of cancer

Mrs. Anderson presents with an itchy raised rash that appears and disappears in various locations. Each lesion lasts for many minutes. Which most likely accounts for this rash? The most unusual aspect of this condition is that the lesions move from place to place.

This is a typical case of urticaria

Fourth intercostal space, left side, at sternal border

Tricuspid landmark

While assessing the head and neck of an adult client, the client tells the nurse that she has been experiencing sharp shooting facial pains that last from 10 to 20 seconds but are occurring more frequently. The nurse should refer the client for possible

Trigeminal neuralgia

When preparing to examine a patient's sclera and conjunctiva during an eye examination, the nurse should instruct the patient to move both eyes to look in which direction?

UP The correct technique to use when examining a patient's sclera and conjunctiva during an eye examination is to instruct the patient to look up. Having the patient look down, to the right, or to the left will not provide visualization of the sclera or conjunctiva during the examination.

While assessing an adult client's abdomen, the nurse observes that the client's umbilicus is enlarged and everted. The nurse should refer the client to a physician for possible

Umbilical hernia

Weber test: (not accurate test)

Use tuning fork and place on top, middle of skull (should be able to hear sound bilaterally)

Petechiae, purpura, and ecchymosis

Vascular lesions; bleeding that occurs in the skin; petechiae = small lesions; purpura/ecchymosis = larger lesions; don't blanch

Short, pale, and fine hair that is present over much of the body is termed

Vellus

Name the two types of hair:

Vellus: short, fine, inconspicuous, unpigemented (over most of our body) Terminal: coarser, thicker, conspicuous, usually pigmented (on head, pubic hair)

A client complains, "I feel like the whole room is spinning around me, and it makes me nauseous sometimes." What term should the nurse use to document the client's symptom?

Vertigo

The nursing instructor is teaching a pre-nursing pathophysiology class. The class is covering the respiratory system. The instructor explains that the respiratory system is composed of both the upper and lower respiratory system. The nose is part of the upper respiratory system. The instructor continues to explain that the nasal cavities have a vascular and ciliated mucous lining. What is the purpose of the vascular and ciliated mucous lining of the nasal cavities?

Warm and humidify inspired air.

What is SADPERSONS?

a suicide risk assessment tool

Which instruction to the client will help facilitate examination of the temporomandibular joint by the nurse?

Open the mouth

III (3), IV (4), VI (6): Oculomotor, Trochlear, Abducens

PERRLA, EOMS, palpebral fissures (associated w/ Down Syndrome or Fetal Alcohol Syndrome)

Define Admission Assessment:

PMHx, allergies, home meds, immunizations, physical assessment data, cultural/spiritual

A nurse assesses a client with a distended abdomen. Which action by the nurse demonstrates the correct way to assess the client for ascites?

Percuss the flanks from bed upward toward the umbilicus

"normal" pink to light brown color/texture most common compliant - often reflect underlying systemic issues

coloring of nails

client remains unresponsive to all stimuli, eyes stay closed

coma

Which function of the skin does this describe: emotions are expressed in the sign language of the face and in the body posture

communication

What does CAGE stand for?

cut down, annoyed, guilty, eye opener

Define: at what point in the complete health history do you create a genogram?

family history

Hyeropia

farsightedness

Bones contain yellow marrow that is composed mainly of

fat

What condition is this? symptoms range from craniofacial abnormalities to cognitive impairment

fetal alcohol syndrome

assist positioning of feet into stirrups ex; pelvic exam, pap smears

lithotomy

Your patient describes her stool as soft, light yellow to gray, mushy, greasy, foul-smelling, and usually floats in the toilet. What would you suspect is wrong with your patient?

malabsorption syndrome

The sigmoid colon is located in this area of the abdomen, the:

left lower quadrant.

To palpate the spleen of an adult client, the nurse should begin the abdominal assessment of the client at the left upper quadrant. right lower quadrant. right upper quadrant. left lower quadrant.

left upper quadrant

To palpate the spleen of an adult client, the nurse should begin the abdominal assessment of the client at the

left upper quadrant.

what is a normal capillary refill time?

less than 2 seconds

Name this lymph node (6)

tonsillar

A client presents to the health care clinic with reports of inability to concentrate at work and daily frontal headaches for the past two weeks. What additional information should the nurse ask this client?

"Are you experiencing sinus pressure and congestion?' The client has a recent onset of a frontal headache and the nurse should collect information on additional findings of a sinus infection. Family history of headaches will not provide information about the current headache. High blood pressure causes a headache in the occipital area. A previous injury will not explain the recent acute onset of headache that the client is now experiencing.

Which question should the nurse ask when assessing a client for a possible detached retina?

"Are you seeing flashing lights?" Flashing lights suggest detachment of vitreous from retina. Slow central loss of vision is associated with macular degeneration. Peripheral loss in advanced open-angle glaucoma. Bilateral loss is often related to a chemical exposure.

A nurse is interviewing a client as part of a routine examination of his ears and hearing. The nurse notes that this client has high blood pressure. Which of the following questions regarding his hearing should the nurse ask that is associated with his high blood pressure?

"Do you experience any ringing, roaring, or crackling in your ears."

A nurse is interviewing a client whose chief complaint is temporomandibular joint pain. Which of the following questions should the nurse ask regarding a causative factor?

"Do you grind your teeth?"

A client describes headaches as severe and lasting for days. Which question would be most appropriate to use to determine if these headaches are migraines?

"Do you have any visual changes before the headache?"

A client presents with otalgia and yellow-green discharge from the external ear canal. Which question should the nurse ask to determine the cause of this problem?

"Have you had any recent trauma to the inside of your ear?"

A client reports the onset of tinnitus. What question should the nurse ask the client to further assess this condition?

"Have you taken any antibiotics in the past few weeks?"

A nurse is assessing a 49-year-old client who questions the nurse's need to know about sunburns he experienced as a child. How should the nurse best explain the rationale for this subjective assessment?

"Having bad sunburns when you're a child puts you at risk for skin cancer later in life."

A nurse observes silvery, white striae on the abdomen of a middle-aged female client during the examination of the abdomen. What is an appropriate question to ask this client in regards to this finding?

"How many times have you been pregnant?"

The patient asks the nurse why the nurse put the tuning fork on the bone behind the ear. Which is the best response by the nurse?

"It identifies a problem with the normal pathways for sound to travel to your inner ear."

A patient calls the clinic and tells the nurse that the doctor told her that she has "otalgia." The patient cannot remember what the doctor explained this to be. How would the nurse most appropriately respond?

"Otalgia is pain in the ear"

Vitamin D

"Sunshine vitamin" Groups at risk: Older adults, those that smoke

otitis externa

"Swimmer's ear" Inflammation of outer ear and ear canal

Torticollis:

"Wryneck"; hematoma in one sternomastoid muscle, probably injured by intrauterine malpostion, results in head tilt to one side and limited neck ROM to opposite side. Feel a firm, discrete, non-tender mass in mid-muscle on the involved side. Requires treatment on mass becomes fibrotic and permanent, which can lead to visual problems; occurs in both children and adults

The results of a client's Rinne test are as follows: bone condcution > air conduction. How should the nurse explain these findings to the client?

"You have a conductive hearing loss."

The nurse suspects that a client is experiencing osteoarthritis. What information about the client's pain caused the nurse to make this clinical determination? Select all that apply.

*Early morning stiffness* *Worse in rainy weather* *Improves with rest* Located in the right hip Worse after sitting

The nurse is assessing the range of motion (ROM) of a patient's joints. What would the nurse use to assess flexion and extension of a joint if the patient complains of pain on examination?

*Goniometer* P540 Angulator Scoliometer Calibrator

A registered nurse is working with a nurse practitioner at a local walk-in clinic. The nurse practitioner needs to perform an objective assessment of a client's left ear. What equipment would the RN make sure is in the room? Select all that apply.

- High-pitched tuning fork -Otoscope

The school nurse notes that the client carries her left shoulder higher than her right shoulder. You should recognize the likely presence of what health problem?

-*Scoliosis* P562 -Torn rotator cuff -Dislocated shoulder -Broken clavicle

Mrs. Fletcher presents to the office with chronic unilateral pain when chewing. She does not have facial or scalp tenderness. Which of the following is the most likely cause of her pain?

-*Temporomandibular joint syndrome* -Temporal arteritis -Tumour of the mandible -Trigeminal neuralgia Explanation: Explanation: Temporomandibular joint syndrome is a very common cause of pain with chewing. Ischemic pain with chewing, or jaw claudication, can occur with temporal arteritis, but the lack of tenderness of the scalp overlying the artery makes this less likely. Trigeminal neuralgia can be associated with extreme tenderness over the branches of the trigeminal nerve. While a tumour of the mandible is possible, it is much less likely than the other choices. P534

A client visits the clinic and tells the nurse that she has joint pain in her hands, especially in the morning. The nurse should assess the client further for signs and symptoms of

-*arthritis.* -osteoporosis. -a neurologic disorder. -carpal tunnel syndrome. Exp: Pain and stiffness in the joints is associated with arthritis. P535

Joints may be classified as cartilaginous, synovial, or...?

-immobile. -*fibrous.* P530 -articulate. -flexible.

Skeletal muscles are attached to bones by:

-ligaments. -fibrous connective tissue. -*tendons.* P528 -cartilage.

what is a symptom seen with GI tract disease

-nausea -vomittng

How would you document a pulse with a force that is absent or unpalpable?

0

Upon assessment of the tonsils, the nurse finds them to be obstructing 30% of midline. This nurse would document this as what?

1+

Edema with mild pitting (2mm), slight indentation, and no perceptible swelling of the leg is noted as:

1+ (edema)

A nurse is working with a patient whose ideal body weight is 130 lb. At what weight would this client first be considered overweight?

143 lbs

In what order should you take these steps for the physical examination of the skin and nails? 1. palpate nails for capillary refill (should refill in 1-2 secs) 2. inspect skin for color, thickness, hair distribution, vascularity, brusing, and lesions 3. inspect nails for color, shape, contour, clubbing and consistency 4. palpate skin for temperature, moisture, texture, thickness, edema, mobility, and turgor

2, 4, 3, 1

In what order should you take these steps for physical examination of the neck (this is excluding lymph nodes)? 1. palpate trachea for tracheal shift by placing index finger on the midline and moving from side to side, both sides should be symmetrical 2. test range of motion by asking the patient to move their head from side to side, up and down 3. inspect trachea for deviation from the midline 4. palpate thyroid gland posteriorly and anteriorly checking for enlargement, consistency, symmetry, and presence of nodules 5. inspect thyroid for enlargement using tangential lighting 6. inspect neck for symmetry, head position, lumps, and acanthosis nigricans 7. test CNVI by asking the person to shrug their shoulders against resistance

6, 2, 7, 3, 1, 5, 4

Depression:

A long-term depressed mood, w/ lack of pleasure; disturbed sleep and appetite; feelings of hopelessness, guilt, worthlessness, sadness, loneliness and despair, suicide ideation

Emerges from an existing primary lesion, such as the keloids that can emerge from the site of a healed wound.

A secondary lesion

Which of the following clients is most likely to be diagnosed with migraine headaches?

A woman whose headaches come on suddenly and are somewhat relieved by a quiet , dark room

Which risk factor for traumatic brain injury (TBI) should a nurse include in a discussion about prevention for a group of adolescents? A. Concussions in sports and motor vehicle accidents cause the largest number of TBIs in teens B. Most firearm incidents are accidental C. Females have twice the risk that males do D. Falls occur more frequently in the younger population

A. Concussions in sports and motor vehicle accidents cause the largest number of TBIs in teens Rationale: Among kids and teens, concussions in sports and motor vehicle accidents account for the largest number of TBIs. It is not surprising that males are more likely to sustain a TBI than females due to more risk-taking behaviors and contact sports or hazardous occupations. Firearm incidents are not listed as a significant cause of TBIs. For older adults, falls and maltreatment account for most TBIs.

The abdominal contents are enclosed externally by the abdominal wall musculature- three layers of muscle extending from the back, around the flanks, to the front. The outer muscle layer is the external

Abdominal Oblique

A client presents to the health care clinic with reports of a 3-day history of fever, sore throat, and trouble swallowing. The nurse notes the client to be febrile with temperature 101.5° F, tonsils are 2+ and red, transillumination of the sinuses is normal. Which nursing diagnosis should the nurse confirm based on this data?

Acute Pain The nursing diagnosis of Acute Pain can be confirmed because it meets the major defining characteristic of verbalization of sore throat. Impaired swallowing is not related to impaired neurological or neuromuscular function.

Accommodation:

Adaptation of the eye for near vision; pupils should constrict and eyes move in toward each other; the ability of the lens to curve according to distance change

A 35-year-old archaeologist comes to the office for a regular skin examination. She has just returned from her annual dig site in Greece. She has fair skin and reddish-blonde hair. She has a family history of melanoma. She has many freckles scattered across her skin. From this description, which of the following is not a risk factor for melanoma in this client?

Age The risk for melanoma is increased in people older than 50 years; this client is 35 years old. The other answers represent known risk factors for melanoma. Especially with a family history of melanoma, she should be instructed to keep her skin covered when in the sun and use strong sunscreen on exposed areas.

Checking for collateral circulation in hand -Depress radial and ulnar arteries together and have person open and close fist. blood should return via ulnar artery for normal circulation. No return for occluded ulnar artery

Allen's test procedure & purpose

he student nurse learns that examining the skin can do all of the following except?

Allow early identification of neurologic deficits Examination of the skin can reveal signs of systemic diseases, medication side effects, dehydration or overhydration, and physical abuse; allow early identification of potentially cancerous lesions and risk factors for pressure ulcer formation; and identify the need for hygiene and health promotion education.

Postical:

Altered state of consciousness after an epileptic seizure

The nurse is planning to instruct a group of adolescents on ways to prevent traumatic brain injuries. What should be included in these instructions?

Always use seat belts

1. The nurse is preparing to examine the sinuses of an adult client. After examining the frontal sinuses, the nurse should proceed to examine the

Ans: Maxillary sinuses

The nurse identifies the client has a positive Obturator sign. The nurse identifies this is due to what?

Appendicitis

A common condition that is restricted to the oral cavity. May be associated with autoimmune problems that create a predisposition to developing oral ulcers.

Apthous ulcers. Commonly called "canker sores".

A client asks a nurse to look at a raised lesion on the skin that has been present for about 5 years. Which is an "ABCDE" characteristic of malignant melanoma?

Asymmetrical shape

A nurse is teaching a client who suffers from peptic ulcers how to reduce the risk of reoccurrence. Which of the following should the nurse recommend?

Avoid excessive alcohol intake

Level of Consciousness:

Awake, Alert, Oriented (person, place, time, to the situation)

While performing an examination of the head and neck, a nurse notices left sided facial drooping. The nurse recognizes this as what condition?

Bell's Palsy

For a nursing exam, students must label a diagram using the correct medical terminology. Where would the students label the metacarpophalangeal joint?

Between the hand and the finger

Name the deep tendon reflexes:

Biceps, tricep, brachioradialis, patellar (knee), achilles, clonus (abnormal)

Cyanosis:

Blue-pigmented color; often seen in new borns; not normal in an adult

Sensory/Motor Homunculus:

Body parts whose movements are more important, occupy more space in the brain

what lines the cheeks?

Buccal mucosa

A 27-year-old woman comes to the emergency department reporting severe right lower quadrant pain. Her temperature is 101.5°F (38.6°C), BP 122/80 mm Hg, pulse 95 beats/min, and respirations 22 breaths/min. What might the nurse suspect the client has? A. Chronic gallbladder disease B. Gastric cancer C. Acute appendicitis D. Hepatitis A

C. Acute appendicitis Rationale: In classic appendicitis, the client reports pain beginning at the umbilicus and moving to the RLQ. If you ask the client to cough, he or she reports pain in the RLQ. The client has local tenderness on palpation in the RLQ, at the McBurney point. A rectal examination, or in women, a pelvic examination, will reveal local tenderness, especially if the appendix is retrocecal.

A client comes to the Emergency Department with bruises on her upper and lower body and appears to be withdrawn. The injuries do not appear consistent with the explanations for them. The client's boyfriend refuses to leave the examination room and is overly protective of her. The nurse suspects: A. Anorexia nervosa B. Hypertension C. Human violence D. Inability of the client to perform ADLs

C. Human violence Rationale: The indications should raise the nurse's suspicions of abuse of the client by the boyfriend. Commonly, abusers are overly protective in the presence of others and will not leave the examination room. Hypertension, inability to perform ADLs and the eating disorder anorexia nervosa are not indicated in this scenario of bruising and withdrawal.

Which type of vessels filter pathogens from the body and drain the fluid that has moved outside of the circulation back into the vessels? A. Veins B. Arteries C. Lymphatic D. Aortic

C. Lymphatic Rationale: Lymphatic vessels filter potential pathogens from the body. They also drain the fluid that has moved outside of the circulation back into the vessels. Arteries carry oxygenated blood from the heart to the body. Veins carry unoxygenated blood from the body to the lungs. Aortic is an adjective for aorta, which is the large vessel carrying oxygenated blood away from the heart.

A client is being assessed for a headache. Symptoms include throbbing and severe pain lasting for the last 8 hours. The client also has a history of vomiting with the headache. What type of headache could these findings indicate? A. Benign B. Tension C. Migraine D. Cluster

C. Migraine Rationale: A throbbing, severe, unilateral headache that lasts 6 to 24 hours and is associated with photophobia, nausea and vomiting suggests migraine. The scenario does not indicate tension, cluster or benign headaches.

The nurse applies the pulse oximeter to the client's finger. What measurement is appropriate for this device? A. Hemoglobin level 13.9 mg/dL B. White blood cell count 7800/mm3 C. Oxygen saturation 97% on room air D. Capillary glucose level 112 mg/dL

C. Oxygen saturation 97% on room air Rationale: Pulse oximetry measures the arterial oxygenation saturation, or SpO2. A probe is placed on the client's finger or earlobe. The toe is used for infants and young children. This device does not measure hemoglobin level, white blood cell count or blood glucose level.

Which cranial nerve innervates the skin, mucous membranes, and sinuses of the face as well as the muscles of mastication? This is also the largest cranial nerve.

CNV (trigeminal)

Which cranial nerve mediates the facial muscles, helping us to make facial expressions?

CNVII (facial)

Which cranial nerve innervates the sternocleidomastoid and trapezius muscles and serves a purely somatic motor function?

CNXI (accessory)

A nurse is working with a client from Asia who has just been diagnosed with oropharyngeal cancer. Which culture-related risk factor should the nurse most suspect in this client?

Chewing betel nuts.

A nurse recognizes that a client may be at risk for malnutrition when which lifestyle behavior is present?

Chronic dieting

Goiter:

Chronic enlarged thyroid gland. Common in wide regions of the world (mountainous regions) where soil is low in iodine, which is essential in formation of thyroid hormones

Scleroderma:

Chronic hardening and tightening of the skin and connective tissues.

A nurse is instructing a client on how to assess himself for herpes simplex lesions by their configuration. Which configuration should the nurse tell the client to look for?

Clustered

Stuporous:

Completely unaware of their surroundings; may respond to pain, but not much else; maybe some verbal stimulation

Which risk factor for traumatic brain injury (TBI) should a nurse include in a discussion about prevention for a group of adolescents?

Concussions in sports and motor vehicle accidents cause the largest number of TBI's in teens

The transmission of sound waves through the external ear and the middle ear is known as

Conductive hearing

When palpating the lymph nodes of the neck, the nurse assesses for which of the following characteristics?

Consistency, delineation, mobility, tenderness

Pupillary reaction:

Constriction in both eyes when you shine a light on eye; do this 4 times

An inflammatory response to an antigen that has contact with exposed skin. Initial contact causes stimulation of the histamine receptors, which results in the classic erythematous and pruritic lesions.

Contact dermatitis

Which characteristic of the gums should a nurse expect to assess in a client who is healthy? A. Enlarged, reddened B. A grey-white line C. Red, bleeding D. Pink, moist, firm

D. Pink, moist, firm Rationale: Pink, moist, firm gums are normal findings of the gums. The nurse may find enlarged, reddened gums as an adverse effect of the phenytoin treatment. Red, swollen, bleeding gums are seen in gingivitis, scurvy and leukemia. A grey-white line along the gum line is seen in cases of lead poisoning.

A patient asks a nurse if any foods promote eye health. What food would the nurse include as a response?

Deep-water fish Foods that promote eye health include deep-water fish, fruits, and vegetables (e.g., carrots, spinach).

Parkinson's:

Deficiency of the NT dopamine and degeneration of the basal ganglia in the brain. Immobility of features produces a face that is flat and expressionless, "masklike", with elevatd eyebrows, staring gaze, oily skin, and drooling

Hypothyroidism/Myxedema:

Deficiency of thyroid hormone; reduces metabolic rate and when severe, causes no pitting edema or myxedema. S/S include: fatigue and cold tolerance/puffy, edematous face, especially around eyes, puffy hands and feet, coarse facial features, and cold, dry skin

Aphasia:

Difficulty speaking

Sequence the steps of the physical examination of the abdomen in the order the nurse should follow. All options must be used.

Drape the client. Ask the client about pain. Warm hands and membrane of stethoscope. Stand at the client's right side. Begin palpation, auscultation, and percussion.

a discoloration of the skin resulting from bleeding underneath, typically caused by bruising

Ecchymosis

A client diagnosed with a peritonsillar abscess exhibits 4+ tonsils and is not able to eat or drink. What is the nurse's priority concern for this client?

Ensure a patent airway 4+ tonsils obstruct 75%-100% to midline in the back of the throat. Potential or actual acute airway obstruction requires immediate intervention and is priority.

The nurse performing an abdominal assessment should proceed in what order? (Place all items in correct order.)

Ensure a warm, comfortable environment. Ask the client to lie supine. Inspection Auscultation Percussion Palpation

Layers of skin:

Epidermis: outer layer; protective barrier Dermis: underneath; nerve, blood vessels, and hair follicles Subcutaneous: layer of fat that provides protection

3rd Intercostal space, left sternal border

Erbs Point landmark

Hemangioma:

Ex. port wine stain; caused by proliferation of blood vessels under the skin

XI (11): Spinal Accessory

Examines sternocleidomastoid and trapezius muscles (equal size, resistances, shoulder shrug)

Cushing syndrome:

Excessive secretion of adrenocorticotrophin hormone (ACTH) and chronic steroid use, the person develops a plethoric, rounded, "moonlike" face, prominent jowls, red cheeks, hirsutism on the upper lip, lower cheeks, and chin, and rash on chest

Scale:

Flakes/plates of dead skin fall from an acute injury to the skin; sometimes seen in cirrhoses

Eschar:

Hard plaque that covers an ulcer where tissue necrosis occurs underneath

How do you palpate the thyroid?

Have pt. lean neck slightly forward or backward, place fingers on the cricoid cartilage and have patient swallow and you feel the thyroid rise and fall; normal is you do NOT feel the thyroid

A nurse is instructing a client who suffers from peptic ulcer disease about the causes of this condition. Which of the following should the nurse mention as a common bacterial cause?

Helicobacter pylori

A nurse is instructing a client who suffers from peptic ulcer disease about the causes of this condition. Which of the following should the nurse mention as a common bacterial use?

Helicobacter pylori

During the physical examination of the mouth, the nurse identifies vesicular eruptions along the client's lips and surrounding skin. The nurse should document which problem?

Herpes simplex.

Which finding should a nurse recognize as normal when assessing the ears of an elderly client?

High-tone frequency loss

What structure is found midline in the tracheal area just beneath the mandible?

Hyoid bone

An older adult client is admitted to the hospital after a fall during which the client's head was injured. While performing the admission assessment, the nurse finds a large ecchymosis over the C7-T1 area. The client reports tenderness on palpation and movement. What would be an appropriate nursing diagnosis for this client?

Impaired comfort related to possible neck injury

What is pyrosis?

Indigestion

Tonsillitis:

Inflamed tonsils; erythema in the throat; white or yellow exudate on tonsils and pharynx; enlarged, swollen uvula

Tic (Tourettes Syndrome):

Involuntary, compulsive, repetitive twitching of a muscle group (ex. wink, head movement)

Reflexes:

Involuntary, stereotypical responses; have to have at least 2 nerves: sensory and motor

USDHHS and the USDA

Jointly push guidelines for healthy eating. Revise guidelines every 5 years.

right sided heart raises this, pulsation descends with inspiration, can be indicative of fluid overload

Jugular venous pressure

A 60-year-old client is concerned about developing cataracts in her eyes. She asks the nurse whether there is anything she can do to reduce her risk. Consumption of which of the following foods should the nurse recommend to the client for this purpose? Select all that apply.

Kale Eggs Oranges Lutein and zeaxanthin found in green leafy vegetables, eggs, and other foods reduce the risk of chronic eye diseases, including age-related macular degeneration and cataracts. Foods rich in these nutrients include kale, spinach, collards, turnip greens, corn, green peas, broccoli, romaine lettuce, green beans, eggs, and oranges. Consumption of red wine, turkey, and skim milk are not associated with a reduced risk for cataracts.

A nurse would like to assess an elderly client's general functional status in performing daily chores. Which of the following should the nurse implement to make this assessment?

Katz Activities of Daily Living tool

A hospitalized client who suffered a recent stroke hasn't started a diet yet and has referrals in to speech therapy, occupational therapy, and physical therapy. What is the nurse's best action at mealtime?

Keep the client NPO until speech therapy has seen client. Explanation: The client should remain NPO until evaluated by speech therapy. Occupational therapists do not specialize in swallowing assessments. Physical therapy does not need to be cancelled and should be continued. The nurse, not the nursing assistant, is responsible for assessment.

Where in the digestive tract is most of the water absorbed?

Large Intestine

Broca's Aphasia:

Located in frontal lobe; expressive aphasia; patient understands what others tell them and know what they want to say, but they have difficulty producing the words

the half moon shaped, white area at the base of the nail abnormalities can indicate some vitamin deficiencies

Lunula of nail

Your patient describes her stool as soft, light yellow to gray, mushy, greasy, foul-smelling, and usually floats in the toilet. What would you suspect is wrong with your patient? Malabsorption syndrome Crohn disease Lactose intolerance Ulcerative colitis

Malabsorption syndrome is characterized by stool that is typically bulky, soft, light yellow to gray, mushy, greasy or oily, sometimes frothy, and particularly foul-smelling, and it usually floats in the toilet. like diarrhea

The most serious form of skin cancer will metastasize - ex; can spread to the brain

Melanoma

A client is being assessed for a headache. Symptoms include throbbing and severe pain lasting for the last 8 hours. The client also has a history of vomiting with the headache. What type of headache could these findings indicate?

Migraine

Anatomy of the nail:

Note the body of the nail (nail plate), lunula, and cuticle

Mr. Martin is a 72-year-old smoker who comes to the clinic for a follow-up visit for hypertension. With deep palpation a pulsatile mass about 4 cm in diameter is palpable. What should the examiner do next?

Obtain abdominal ultrasound

Primary lesions:

Occur with onset of disease

Beau lines:

Occurs from trauma to nail plate causing a dent; eventually grows out and disappears

A patient comes to the clinic and reports pain when he touches his ear. With what is this finding most consistent?

Otitis externa

For which client condition would the nurse most likely expect a capillary refill time longer than 2 seconds?

Peripheral vascular disease

The nurse asses an adult male clients abdomen and observes diminished abdominal respiration. The nurse determines that the client should be further assessed for:

Peritoneal irritation

Scar:

Permanent fibrotic change on the skin after damage to the dermis; shows up differently depending on color of skin

Erythema:

Pink-pigmented color; sometimes seen with a fever

Which precaution should a nurse take to ensure the safety of a client when performing the Romberg test?

Place arms around the client without touching

How should the nurse perform blunt percussion over the liver?

Place left hand on right lower rib cage, strike it with ulnar side of right fist

creatinine excretion

Reflects muscle mass

Decorticate vs. Decerebrate:

Relates to scoring of the Glasgow Coma Scale

Which nursing diagnosis is most appropriate for an elderly client with poor dentition?

Risk for Imbalanced Nutrition: Less Than Body Requirements

dementia/ Alzheimers screening tools

SLUMS test

Name the sutures of the skull:

Sagittal (separates the parietals) Coronal (separates the frontal from the parietals) Lamboid (separates parietals from the occipital)

When assessing the tympanic membrane, a variation of normal is a tympanic membrane with what?

Sclerosis

abnormal lateral curvature of the spine, reduced AP diameter

Scoliosis

Hallucinations:

Sensory perceptions for which there are no external stimuli; may strike any sense: visual, auditory, tactile, olfactory, gustatory

Inspect/palpate the skull for:

Size, shape, and symmetry/scalp for deformities, tenderness, and temporal artery

A client diagnosed with Sjogren syndrome should be given which instructions?

Sjogren syndrome is a chronic inflammatory disorder characterized by decreased lacrimal and salivary gland secretion. Eye drops and hard candy can provide relief from dryness.

Erosion:

Slightly depressed area of skin, where all the top layers of the epidermis is lost; usually from burns

During an assessment, the patient describes vomiting moderate amounts that "smell like poop." The nurse might suspect

Small bowel obstruction

The nurse is caring for a client being treated for a nutritional deficiency. The nurse expects that the client has a dysfunction of which abdominal body part?

Small intestine

Two routes for sensory pathways:

Spinothalamic tract: controls fine touch, position, and vibration Posterior dorsal tract ("posterior horn"): perceive pain, temp., and crude touch

A client is having trouble turning her head to the side. Which of the following muscles should the nurse most suspect as being involved? A. Sternocleidomastoid B. Trapezius C. Masseter D. Temporalis

Sternocleidomastoid

Aging adult and the head/neck:

Temporal arteries twisted and prominent; senile tremors (mild rhythmic tremor of the head), face looks smaller if teeth are missing, perform ROM slowly

Syncope:

Temporary loss of consciousness caused by a fall in blood pressure.

A nurse is utilizing the Braden Scale for Predicting Pressure Sore Risk during the admission assessment of an older adult client. What assessment parameter will the nurse evaluate when using this scale?

The client's ability to change position

Which action by the nurse is consistent with the Rinne test?

The nurse strikes the tuning fork and places it on the patient's mastoid process to measure bone conduction

What is psoas test?

The one where they kick up and you hold it down or they kick back while on their side.

When examining the head, the nurse remembers that the anatomic regions of the cranium take their names from which of the following sources?

The underlying bones

When examining the head, the nurse remembers that the anatomic regions of the cranium take their names from which of the following sources? A. Noted anatomists B. The underlying bones C. Their anatomical positions D. The underlying vascular network

The underlying bones

A 4-year-old child presents to the health care clinic with circular lesions. Which of the following conditions should the nurse most suspect in this client, based on the configuration of the lesions?

Tinea corporis (ring worm)

Rovsing's sign is a test of referred rebound tenderness in appendicitis.

True

True/False: Subjective and objective data are both important parts of an assessment. Subjective data are things the client or his or her family tells the nurse.

True

True/False: The nurse tests the six cardinal directions to test extraocular movement of the eye.

True

True/False: When assessing a client with asthma, the nurse would expect to hear wheezing.

True Rationale: Wheezing indicates narrowing of the airways due to spas or obstruction. Wheezing is associated with CHF, asthma (reactive airway disease) or excessive secretions.

Heterochromia:

Two different colored iris

While assessing an adult clients abdomen, the nurse observes that the clients umbilicus is enlarged and everted. The nurse should refer the client to a physician for possible:

Umbilical hernia

Dyskinesia:

Uncontrolled movement of the body

Which area of the body should a nurse inspect for possible loss of skin integrity when performing a skin examination on a female who is obese?

Under the breast The nurse should inspect the area under the breast for skin integrity in obese clients. The area between the skin folds is more prone to loss of skin integrity; therefore, the presence of skin breakdown should be inspected on the skin on the limbs, under the breasts, and in the groin area. Perspiration and friction often cause skin problems in these areas in obese clients. The areas over the chest and abdomen and on the neck are not prone to skin breakdown.

Anisocoria:

Unequal pupil size

Hirsutism:

Unwanted male-pattern hair growth in women

BMI

Weight in KG divided by Height in meters, divided by height in meters Guide to maintaining ideal weight to height ratio

(nail fold/cuticle) -"normal" 160 degrees abnormalities - clubbing >180 degrees- indicative of hypoxia brittle nails onycholysis- think yellow fungus

angle of attachment- nails

What does ABCDE(E) stand for when teaching skin self examination?

asymmetry, border, color, diameter (>6mm), elevation, evolution

an audible vascular sound associated with turbulent blood flow

bruit

Define: larger than 1cm in diameter; superficial in epidermis; thin-walled and ruptures easily. (friction blister, burns, pemphigus, contact dermatitis)

bulla

S2 "dub" heart sound. What is happening?

closure of aortic and pulmonic valves, opening of mitral and tricuspid valves

What type of headache has these characteristics: pain is in and around the eye

cluster

low pitched bubbling, moist sounds early inspiration to early expiration (Velcro separating) Pneumonia, pulmonary edema, pulmonary fibrosis

coarse sounds

ability to take in information and act on it, revolves around neurological assessment

cognition

advanced age- higher risk for dementia

cognition primary risk factors

The nurse plans to assess an adult client's kidneys for tenderness. The nurse should assess the area at the left upper quadrant. costovertebral angle. external oblique angle. right upper quadrant.

costovertebral angle. The angle found on your back made by the bottom ribs meeting at an angle

Name this cartilage

cricoid

What kind of variations include? darker pigmentation - higher predisposition to keloids oral mucosa best site to determine changes mongolian spots fair skinned- increased risk for skin cancer

cultural variations

Which characteristic feature of the tympanic membrane should a nurse anticipate finding in a client with otitis media? a) Pearly, translucent with no bulging b) Yellowish, bulging with fluid bubbles c) Gray, translucent with retraction d) Red, bulging with an absent light reflex

d (Explanation: A client with acute otitis media would have a red, bulging eardrum with absent light reflex. A pearly, translucent membrane with no bulging is a normal finding in the tympanic membrane. A yellowish, bulging membrane with bubbles is seen in serous otitis media. A gray, translucent membrane with retraction is a normal finding in the tympanic membrane.)

For what group should you take this information into consideration? -cyanosis, jaundice, erythema, and carotenemia may present differently -acanthosis nigricans is more difficult to see; use penlight -lower risk for melanoma

darker skinned individuals

What does it mean when a person's shape has a line through it in a genogram?

deceased

Define: resting pressure that blood exerts constantly between each contraction

diastolic pressure

Define: What kind of shape/configuration of lesions is characterized by distinct, individual lesions that remain separate (EX. skin tags, acne)

discrete

What does two diagonal lines through a horizontal line between two people in a genogram mean?

divorced

The nurse has tested an adult client's visual fields and determined that the temporal field is 90 degrees in both eyes. The nurse should

document the findings in the client's records. Validate the eye assessment data that you have collected. This is necessary to verify that the data are reliable and accurate. Document the assessment data following the health care facility or agency policy.

What condition is this? trisomy 21. Small head at birth, flat facial features, flat nasal bridge, upward slanting eyes, abnormal ears

down's syndrome

may indicate hypothyroidism

dry or course hair

The nurse is performing percussion on a client's abdomen. What would the nurse expect to hear over the liver of the right upper quadrant?

dullness

what is epigastric pain, pain that occurs a few hours after eating and is relieved when eating again

duodenal ulcers

A client reports having chronic discomfort in the upper abdomen. The nurse can be certain that this client is experiencing dyspepsia when which symptoms are present?: A. dysphagia B. epigastric burning C. satiation early in a meal D. feeling fullness after eating E. regurgitation

epigastric burning satiation early in a meal feeling fullness after eating

An avid swimmer presents with ear pain. Her history includes pain and drainage from the left ear. On examination, she has pain when the ear, including the tragus, is manipulated. The canal is narrowed and erythematous with some white debris in the canal. The rest of the examination is normal. What diagnosis would be most appropriate for this client?

external otitis

Define: moving from open-ended questions to focused questions

guided questioning

separate assessment / inspect at one inch intervals

hair assessment

Define: at what stage of the comprehensive health history do you use OLDCHARTS/OPQRSTU to assess symptoms

history of present illness (HPI)

Define: name, address, phone number, age, birth date, birthplace, sex, marital status, race, ethnic origin, occupation, education

identifying data

associated with right ventricular hypertrophy cause by pulmonic valve disease, pulmonic HTN, and chronic lung disease

lift/heave

A nurse assesses a client who reports abdominal pain. Which technique should the nurse use during the physical examination to detect tenderness?

light palpation

What life stage does a person have thin, smooth, elastic skin that is more permeable than an adult, thus a greater risk of fluid loss?

newborn (infant)

To palpate the spleen of an adult client, the nurse should

place the right hand below the left costal margin.

Which function of the skin does this describe: minimizes injury from physical, chemical, thermal, and light-wave sources; stops invasion of microorganisms and loss of water and electrolytes from within the body.

protection

What 4 qualities are considered when assessing pulse?

rate, rhythm, force, equality

Describe referred pain?

ravels, or refers, from the primary site and becomes highly localized at the distant site (Weber 481)

Define: Brief statement of the issue in patient's own words- exact words recorded and enclosed in quotation marks

reason for seeking care

The nurse suspects that a client has Cushing's syndrome. What assessment finding did the nurse use to make this clinical determination? red cheeks mask-like face swelling around the eyes elongated prominent forehead

red cheeks

What finding supports the diagnosis of carbon monoxide poisoning?

reddish lips

what test is pain in the RLQ when applying pressure to the LLQ (rovsings sign) indicating appendicitis

referred rebound tenderness

Define: a simple repetition of the patient's last words

reflection

Name this muscle

sternomastoid (sternocleidomastoid)

How would you describe a heart rate 101bpm>

tachycardia

Sinus Bradycardia in athletes is due to the heart adapting to the physical stresses that it is put under by the athlete's physical activity. This causes the heart to become more efficient producing a greater stroke volume, which in return allows the heart to circulate the same amount of blood with fewer contractions.

why athletes have lower heart rates

Which function of the skin does this describe: allows cell replacement of surface wounds

wound repair

What components of sound does the cochlea interpret? (Select all that apply.)

• Amplitude • Frequency Explanation: The cochlea interprets two components of sound: amplitude (volume) and frequency (pitch). The cochlea does not interpret tone, direction, or decibel.

The nurse is preparing to perform a nutritional assessment for a client. Which of the following questions would be most appropriate to use when initiating the assessment?

"Can you tell me what you've eaten in the last 24 hours?"

Aphthous ulcer:

"Canker sore"; common; a vesicle at first and then a small, round, "punched out" ulcer with a white base surrounded by a red halo. Quite painful and last for 1-2 weeks. Cause is unknown, although associated with stress, fatigue and food allergy.

During a physical examination of the head and neck, a client reports frequently feeling cold. What additional questions should the nurse ask for more information about the client's symptoms? (Select all that apply.)

"Do you dress more warmly than other people? "Do you use more blankets that others at home? "Do you perspire less than others?"

Which question asked by the nurse is assessing problems with vertigo?

"Do you ever have problems with balance"

Which question asked by the nurse is assessing problems with tinnitus?

"Do you experience buzzing in your ears?"

Anatomy of the nose:

Note the nasal septum, turbinates (ridges on the inside), tip, bridge of the nose, vestibule (just inside)

What precaution should the nurse take when measuring a client's abdominal girth to screen for cardiovascular risk factors?

Place the tape measure behind the client and measure at the umbilicus

Superficial elevated lesion, 1cm or larger

Plaque (primary)

Psoriasis is an example of what lesion?

Plaque lesions

During the physical examination of a client, a nurse detects a thick and tender temporal artery. Which additional assessment should the nurse perform to rule out the possibility of temporal arteritis?

Vision Acuity

II (2): Optic

Visual acuity

Occipital lobe:

Visual receptor center; if stroke occurs, may go blind or have other vision issues

A patient describes probable night blindness. Intake of what vitamin should be evaluated?

Vitamin A

The nurse assessing for unilateral hearing loss by using a tuning fork. What test is the nurse performing?

Webers test

The molars are responsible for?

grinding and final chewing before swallowing

Abdominal organs are supplied with blood from what vessels?

the abdominal aorta and its major branches, which are the left and right iliac arteries

Putting on gloves and retracting the client's lips and cheeks is a technique used to examine _________________________

the gums and teeth

why do you want to position yourself on the right side for a cardiac exam?

the nurse can achieve a better view of the precordium, it may also allow you to view any lifts more easily

what does alcohol use affect

the pancreas

The colon originates in this abdominal area: the right lower quadrant.

the right lower quadrant.

where are the kidney assessed from

they are assessed from the posterior and anterior left and right flank

what is a split S2 sound? is it normal?

this sound is normal in young athletic people. it is accentuated by inspiration and sitting

palpable vibration on the chest wall accompanying severe heart murmur, palpated over the 2nd and 3rd intercostal space, may indicate severe aortic stenosis

thrill

Name this cartilage

thyroid

Name this gland

thyroid gland

screening tool, not diagnostic for depression

PHQ-9

Pustular lesions

acne, furuncles, carbuncles

what number correlates with moderate edema? (4mm depression disappears 10-15 seconds)

2+

What are some of the challenges of notetaking? Select all that apply: A- it breaks eye contact B- it increases the patient's sense of importance C- it enhances observation of the patient's nonverbal behavior D- it shifts the attention away from the patient E- it interrupts the patient's narrative flow F- may be threatening during the discussion of sensitive topics

A, D, E, F

What is the most important lifestyle changes a client can make to improve cardiovascular health? A. Quitting smoking B. Eating a diet high in fat C. Getting less exercise and more rest D. Living a more sedentary lifestyle

A. Quitting smoking Rationale: Nurses work with clients over time to modify lifestyle choices that reflect healthy behaviors. The most important are stopping smoking, reducing high blood pressure and reducing high cholesterol.

Occur after acute illness and eventually grow out.

Beau's lines

Strabismus:

Cross eyed

Presbyopia:

Decrease in the power of accommodation w/ aging

Alopecia:

Loss of hair in patches

Myopia:

Nearsighted; globe is longer

I (1): Olfactory

Sense of smell

Which of the following is a symptom related to vertigo?

Spinning sensation

clarity/ content/ perceptions

Thought processes

true or false, liver size decreases after age 50?

True

The incisors are responsible for?

biting food

What is the symbol for a female in a genogram?

circle

Name this lymph node (8)

deep cervical

hunchback, increased AP diameter

kyphosis

Name this facial bone

mandible

In general, a BMI 30> is considered _____

obese

vibration

ulnar

pain on swallowing

odynophagia

Ulceration:

Necrosis of the epidermis, dermis, and sometimes the subcutaneous tissue

Women who are pregnant

Need an additional 300/500 cal per day with emphasis on protein sources. Vitamin and mineral supps may be required

Grey matter:

No myelin; grey because of the grey nucleus; 40% of brain

is it normal to hear S3 and S4 sounds?

Nope

amino acids can be metabolized into what three things?

energy, glucose or fat

The nurse observes an inward turning of the lower lid in a 77-year-old patient. The nurse documents

entropion

What layer of the skin is this: outer, thin layer; forms a protective barrier

epidermis

what terms will help to identify where pain is

epigastric umbilical hypogastric /suprapubic

where is the abdominal aorta located

in the epigastric area, above the umbilicus

Chorea:

Sudden, rapid, jerky, purposeless movement involving limbs, trunk, or face. Dance-like form of dyskinesia

A client complains of pain, numbness, and tingling in the upper extremities for several weeks before coming to the clinic for evaluation. What is the nurse's best action?

Suggest a referral to orthopedic spine specialist

Glasgow Coma Scale:

Test LOC (eye opening, verbal response, best motor response); best score 15/15

Babinski:

Test by stroking lateral aspect and across the ball of the foot; abnormal finding is extension/fanning of toes

Rinne test: (not accurate test)

Test the difference between air vs. bone conduction; air conduction last longer than bone conduction; vibrate the tuning fork on the mastoid process, have pt. tell you when they can no longer hear it, then remove tuning fork and hold it up to the ear (pt. should still be able to ear vibrations through air conduction); normal = AC:BC = 2:1

Clients with respiratory infection have __________ odors in their breath.

foul

bumps- indentations of nails (pitting) - abnormal, seen often in psoriasis pts.

condition of nails

tapping surface of body to determine density- tappign produces vibrations and sound waves

precussion

full thickness tissue loss with visible fat necrotic subcutaneous tissue, deep crater

pressure ulcer stage 3

Full-thickness pressure ulcer involves all skin layers and extends into supporting tissue. Exposes muscle, tendon, or bone, and may show slough (stringy matter attached to wound bed) or eschar (black or brown necrotic tissue).

pressure ulcer stage 4

A person who is confined to a bed or immobilized are more at risk for ________ ______. Common sites are on the heel, ischium, sacrum, elbow, scapula, vertebrae, ankle, knee, hip, rib, and shoulder.

pressure ulcers

what number correlates with moderately severe edema? (6mm depression lasts >1 minute)

3+

Edema with deep pitting (6mm), indentation that remains for a short time and leg that looks swollen is noted as:

3+ (edema)

In what order should you palpate the lymph nodes? Remember that you will be using a gentle circular motion with your fingerpads, moving skin over underlying tissue in each area. 1. submental- midline behind tip of mandible 2. supraclavicular- just above and behind the clavicle at the sternomastoid muscle 3. preauricular- in front of the ear 4. occipital- base of the skull posteriorly 5. submandibular- 1/2 way between the angle and tip of mandible 6. deep cervical- deep under sternomastoid 7. postauricular- superficial to mastoid process 8. posterior cervical- along anterior edge of trapezius in posterior triangle 9. tonsillar (jugulodigastric)- under angle of mandible 10. superficial cervical- overlying (superficial to) sternomastoid muscle

3, 7, 4, 9, 5, 1, 10, 6, 8, 2

Clubbing of the nails indicates? Clubbing is identified when the angle of the nail to the finger is more than 160 degrees.

Chronic hypoxia

Upon examination of the ear with an otoscope, the nurse documents the skin of the ear canal as thickened, red, and itchy. The nurse would expect this finding with a diagnosis of

Chronic otitis media

Hyperopia:

Farsighted, near objects are blurry; globe is shorter

what is the term for kidney stones

renal calculi

percussion tones from vibrations (lungs)

resonance

Define: At what stage of the complete health history do you address each body system from head to toe and address health promotion?

review of systems

A nurse examines a client with complaints of a sore throat and finds that the tonsils are just visible. Using a grading scale of 1+ to 4+, how should the nurse appropriately document the tonsils?

1+

Frontal lobe:

Controls personality, behavior, emotions, and intellectual functioning; when damaged, people have problems with executive functioning (trouble w/ things an administrator), changes w/ personality

Bipolar disorder:

A disorder associated with episodes of mood swings ranging from depressive lows to manic highs

Fasciculation:

A single muscle begins twitching as individual fibers within it

Which assessment of the tongue should a nurse recognize as abnormal?

A smooth, red, shiny tongue without papillae is indicative of a loss of vitamin B 12 or niacin.

When palpating the neck, performing which of the following techniques will help differentiate lymph nodes from a band of muscles?

Attempting to roll the structure up and down and side to side

Ruptured TM:

Bulging, inflammed

activation of atria, atrial depolarization

EKG: P wave

activation of ventricles, ventricular depolarization and atrial repolarization

EKG: QRS complex

Mr. Kruger, 84 years old, presents with a smooth lower abdominal mass in the midline, which is minimally tender. There is dullness to percussion up to 6 cm above the symphysis pubis. What does this most likely represent?

Enlarged bladder

Lymphadenopathy:

Enlargement of the lymph nodes (>1 cm) from infection, allergy, or neoplasm

After completing a shift assessment on clients, the nurse reviews the early morning laboratory results. One of the clients has a potassium level of 3.0. The nurse knows that this potassium level can cause what?

Fatal cardiac dysrhythmias

A client visits a community clinic reporting severe allergies causing a "crackling sensation" in the ear. The physician diagnoses serous otitis media. Which of the following is a characteristic of this condition?

Fluid collects in the middle ear causing an obstruction of the auditory tube.

taking non steroidal-anti inflammatory drugs (NSAIDS) e.g aspirins, ibuprofen, or steroids cause

GI bleeding

A client complains of epigastric pain and tarry stools. The nurse should suspect which of the following as the underlying cause?

Gastric ulcer

Define Health Assessment:

Gathering information about the health status of the patient, analyzing and synthesizing those data, making judgments about nursing interventions based on the findings and evaluating patient care outcomes

VII (8): Acoustic

Hearing acuity: Rinne (AC:BC), Weber, whisper test

The nurse is presenting an educational event for a local civic group about the risk factors for neck cancer. What would the nurse list? (Select all that apply.) Female gender Male gender Coffee drinker Tobacco use Age older than 50 years

Male gender Tobacco use Age older than 50 years

The nurse is testing the client for extension of the wrist and notes weakness on the right side. This assessment finding is consistent with what disease of the nervous system?

Multiple Sclerosis

anemia and shock

Pallor around the lips

A client complains of recurring headaches that are worse when first waking in the morning and with coughing or sneezing. What would be the nurse's most appropriate action?

Perform a focused assessment

A 55-year-old woman with a history of type 2 diabetes went through menarche at age 19 and menopause 2 years ago. Which of the preceding is a risk factor for osteoporosis?

Postmenopausal status

ventricular depolarization

QRS complex

screening tool for dementia/Alzheimers

SLUMS

Keloid:

See more in dark skinned individuals; exaggeration of the connective tissue when healing

Photophobia:

Sensitivity to light

Parkinson's gait:

Shuffling gait

When assessing a client with Graves disease, how would you expect the thyroid gland to be?

Soft

Nodules:

Solid, raised lesion that is half a cm; can be epidermis, dermis, and sub-q tissue; Ex. basal cell carcinoma

The client is facing the nurse with his forearm turned so that his palm is up. What movement is the client exhibiting?

Supination

Nephrotic syndrome:

Swelling of the face, especially around the eyelids and lips

Pupillary light reflex:

The eye that do NOT have direct light, but is constricting in response to the light on the retina of the other eye; consensual, direct;

The nurse is admitting a 79-year-old man for outpatient surgery. The patient has bruises in various stages of healing all over his body. Why is it important for the nurse to promptly document and report these findings?

The patient may have been abused. Multiple ecchymoses may be from repeated trauma (falls), clotting disorder, or physical abuse.

The rich blood supply of the nose serves to

The rich blood supply of the nose warms the inspired air as it is moistened by the mucous membrane.

what are the openings to the middle ear? Select all that apply.

The tympanic membrane The round window The oval window The eustachian tube

A decrease in oxyhemoglobin will result in documentation of pallor.

True

Graphesthesia:

Use a pen or finger and draw a letter or number into the patients palms and they should ID the number or letter

Recommended protective measures to avoid skin cancer include which of the following?

While monthly self-examination and awareness of signs of skin cancer may aide in early detection, only avoiding sun will prevent and protect against skin cancer. Clinical examinations are recommended annually.

A nurse performs a hearing test on an elderly client. Which result should the nurse recognize as an indication that presbycusis is present? An inability to hear:

Whispered sounds

Icterus:

Yellow sclera; usually occurs with jaundice

crackles/rales- can be fine or coarse

adventitious sounds

A waist circumference >40 for men and >35 for women poses an increased risk for ______________ _______

cardiovascular disease

Define: yellow/orange pigmentation of the skin due to too much carotene in the blood

carotenemia

Name this bone

clavicle

suicidal/homicidal thoughts, delusions

content

temperature

dorsal

Name this vein

external jugular vein

deep breathing, >20 deep RR

hyperpnea

Name this facial bone

maxilla

what is a kidney infection

pylenephritis

how do you test for arterial insufficiency?

raise legs, examine soles of feet, rubor color is a big indicator

what does SBIRT screen for?

screening for risky substance use behaviors

Name this duct

wharton's duct

Edema that has no pitting is noted as:

0+

during an inspection of the abdomen what are the major causes of distenstion - "6 F's"

1. fat 2. feces 3. fetus 4. fibroids 5. flatulence 6. fluid

How many lobes does the left lung have?

2 lobes LUL LLL; sternal notch

Mental illness is...

A physical condition just like asthma or arthritis

Fissure:

A linear cleavage of the skin that goes into the dermis; common at the corners of your mouth

A nurse is going to perform a physical examination on a pregnant woman. Which of the following are normal findings in a pregnant woman? Select all that apply: A- chloasma B- striae gravidarum C- ecchymosis D- linea nigra

A, B, D

What is the normal pathway of hearing:

Air conduction

what are the anatomical landmarks for cardiac palpation/ auscultation?

All Pigs Eat Too Much Aortic Pulmonic Erbs Point Tricuspid Mitral Area/ Apex

Which change in auscultation of bowl sounds should the nurse recognize as most diagnostic of an intestinal obstruction?

An increase in the pitch

What medications may cause epistaxis?

Anticoagulants

When collecting a client's medical history, the nurse should ask if the client is taking which medications that most likely contribute to complaints of recurrent epistaxis? (Select all that apply.)

Anticoagulants, antihistamines, and herbal supplements are common drugs that result is epistaxis as an adverse reaction. Hormones and central nervous system agents can lead to nasal congestion, but are not as common for leading to epistaxis.

How would you document edema that has moderate pitting and indentation that subsides rapidly? A- 1+ B- 2+ C- 3+ D- 4+

B (2+)

Which of the following would the nurse use as the primary assessment for a client's pain? A. The client's spiritual view of pain B. The client's report of pain C. Psychosocial questions related to perceptions D. Current pain therapies used

B. The client's report of pain Rationale: Pain is a subjective phenomenon, and thus the main assessment lies in the client's reporting of the pain. The client's spiritual views, current therapies used and psychosocial questions about the client's perception may provide additional information about the pain. However, the client's pain is whatever the person says it is.

Ankle Clonus:

Basic reflex; push the ankle up quickly, their foot vibrates rapidly; abnormal finding; hyper-stimulation

An alternate pathway that bypasses the external and middle ear is called what? a) Bone conduction b) Air conduction c) Neuro conduction d) Sensory conduction

Bone conduction Correct Explanation: An alternate pathway, known as bone conduction, bypasses the external and middle ear and is used for testing purposes

What is bone conduction?

Bones of the skull vibrate; used for testing

OU, OD, OS:

Both eye, right eye, left eye

Which of the following is a skin condition that is characterized by areas of dark, velvety discoloration in the body folds and creases? A- liver spots B- melasma C- acanthosis nigricans D- eczema

C (acanthosis nigricans)

When a nurse in conducting an interview, the nurse should do which of the following? Select all that apply: A- face away from the patient when conducting the interview B- speak to the translator when there is a language barrier C- ask open-ended questions D- eliminate any noise to avoid distractions E- provide the patient with false assurance or reassurance

C, D

A nurse takes a client's vital signs. Which of the following is considered a vital sign? A. Visual acuity B. Urinary output C. Blood pressure D. Mental status

C. Blood pressure Rationale: Vital signs are a person's temperature, pulse, respiration and blood pressure. Mental status, visual acuity and urinary output are not considered vital signs, even though they are frequently assessed.

The nurse is palpating a client's cervical vertebrae. Which vertebra can be easily palpated when the neck is flexed and should help the nurse locate the other vertebrae?

C7

You note a lesion during a skin assessment. Which is the best way to document this finding? A- raised, irregular lesion the size of a quarter, located on dorsum of left hand B- open lesion with no drainage or odor, approximately 14in in diameter C- pedunculated lesion below left scapula with consistent red color and no drainage or odor D- dark brown raised lesion, with irregular border, on dorsum of right foot, 3cm in size, with no drainage

D

a behavioral or psychological syndrome or pattern that occurs in an individual, reflect an underlying psychobiologic dysfunction

DSM-5 mental disorder

Orthostatic Hypotension:

Decrease in systolic blood pressure of 20 mmHg or a decrease in diastolic blood pressure of 10 mmHg within three minutes of standing when compared with blood pressure from the sitting or supine position

Ophthalmoscope exam:

Decrease lighting, right hand to examine right eye and vice versa, keep both your eyes open, note red reflex

An older client presents with symptoms of pain on urinating. The nurse recognizes that older adults are at increased risk for urinary tract infections for which of the following reasons?

Decreased activity of protective bacteria in the urinary tract

Paralysis:

Decreased or loss of motor power caused by problem with motor nerve or muscle fiber. Patient is unable to move one side/part of their body

Aging adult eyes:

Decreased tear production (dry eyes), drooping eyelids, pupils get smaller, lens loses elasticity, lose ability to accommodate, cataracts, glaucoma, macular degerenation

Well-vascularized, connective tissue layer containing collagen and elastic fibers, nerve endings, and lymph vessels. It is also the origin of sebaceous glands, sweat glands, and hair follicles.

Dermis

The medical term used to describe difficulty swallowing. This can be due to a neuromuscular or oropharyngeal deficit. Hoarseness can accompany dysphagia; however, weight gain and fatigue do not.

Dysphagia

A hospitalized client continues to exhibit residual effects of a stroke. Which symptom is the priority concern?

Dysphagia can lead to aspiration and is the priority concern to maintain a patent airway. A weak gait can lead to falls but is not priority over airway. Right ptosis, or eyelid drooping,and facial weakness can inhibit certain facial movements but this is not a priority concern over airway.

Brainstem:

Effects breathing, heart rate, blood vessels constriction, and alertness

Cerebellum:

Equilibrium, balance, and coordination

Linear lesions:

Ex. burrows or poison ivy

Nystagmus:

Fine, oscillating movements best seen around the iris; denotes a CNS impairment

Clubbing

Fingernails are wider; nails are rounded; sign of heart failure

A nurse receives report from the shift nurse that a client has new onset of peripheral cyanosis. Where should the nurse focus the assessment of the skin to detect the presence of this condition?

Fingers and toes Peripheral cyanosis is usually a local problem with manifestations of cyanosis, a blue-tinged color to the skin, caused by problems resulting in vasoconstriction. Changes in color around the mouth are called circumoral. Bluish tints to the chest and abdomen cyanosis is called central cyanosis.

An 85-year-old retired housewife comes with her daughter to establish care. Her daughter is concerned because the client has experienced frequent falls in recent months. As part of the physical examination, the nurse asks the client to walk across the examination room. Which of the following is not part of the stance phase of gait?

Foot arched

White or yellow papules appearing on the cheeks, tongue and lips.

Fordyce granules

List the sinuses:

Frontal*, maxillary*, ethmoid, and sphenoid * = only these are palpable

What is the most common type of hyperthyroidism?

Grave's disease

Confluent:

Growing together

A client presents to the health care clinic with reports of pain in the hands and right wrist. Additional history reveals that the client is a factory worker who spends all day performing the same repetitive task. The nurse performs Phalen's test and Tinel's test with positive results. The hand grips are unequal, with the right weaker than the left. What nursing diagnosis can the nurse confirm from this data?

Impaired Physical Mobility

A client's electronic health record states that he has been diagnosed with sensorineural hearing loss. Which condition should the nurse most likely identify as a cause?

Inner ear problems

The nurse is preparing to assess the abdomen of a hospitalized client 2 days after abdominal surgery. The nurse should first:

Inspect the abdominal area

A child presents to the health care facility with new onset of a foul-smelling, purulent drainage from the right nare. The mother states that no other signs of an upper respiratory tract infection are present. What is an appropriate action by the nurse?

Inspect the nostrils with an otoscope.

A nurse is working with a client who is Buddhist. Based on what the nurse knows of the common dietary practice of this religion, which nutrient should the nurse make sure that the client is not deficient in?

Iron

Monique is a 33-year-old administrative assistant who has had intermittent lower abdominal pain approximately one week a month for the past year. It is not related to her menses. She notes relief with defecation, and a change in form and frequency of her bowel movements with these episodes. Which of the following is most likely?

Irritable bowel syndrome

Deep, purple lesions. The lesions may be raised or flat.

Kaposi sarcoma

The sigmoid colon is located in this area of the abdomen: the

Left lower quadrant

A nurse inspects a client's skin and notices several flat, brown color change areas on the forearms. What is the proper term for documentation of this finding by the nurse?

Macule A macule is a flat, non-palpable skin color change that may manifest as brown, white, tan, red, or purple. Freckles and port wine birthmarks are examples of a macule. A circumscribed elevated mass containing fluid is called a vesicle or bulla, depending on it size. A nodule is a solid, palpable mass. A papule is an elevated, palpable, solid mass that is smaller in diameter than a nodule.

V (5): Trigeminal

Motor: clench teeth, jaw strength (scale 0-5) Sensory: test light touch on forehead, cheeks, chin

VII (7): Facial

Motor: facial symmetry w/ smile, frown, puff cheeks Sensory: not routinely test. Tests sense of taste

IX (9) and X (10): Glossopharyngeal and Vagus

Motor: uvula and soft palate movement, gag reflex, and hoarseness of voice Sensory: taste on POSTERIOR third of tongue

White matter:

Myelin is present; 60% of brain; contains extended axons, so it goes long distances

The nurse observes a young client holding a newspaper up close to read. Which condition does the nurse suspect this client suffers from?

Myopia

Where should a nurse place the hands to palpate the submandibular lymph nodes?

On the medial border of the mandible

total time for ventricular depolarization and repolarization

QT interval

The nurse correctly identifies the gallbladder is located where?

RUQ

The nurse is caring for a 63-year-old client who can neither read nor speak English. What would be the appropriate chart to use to assess this patient's vision?

Snellen E

Palpules (palpules are palpable):

Solid, raised lesion w/ distinct borders that is less than 1 cm in diameter; variety of shapes

How many layers of muscle are in your abdominal area?

The abdominal contents are enclosed externally by the abdominal wall musculature, which includes three layers of muscle extending from the back, around the flanks, to the front. (Weber 473)

The nurse prepares to assess the anterior triangle of a client's neck. Where should the nurse palpate this area on the diagram?

The anterior triangle is located in the area below the mandible, lateral to the sternocleidomastoid muscle and medial to the midline of the neck.

Composed of four distinct layers: the stratum corneum, stratum lucidum, stratum granulosum, and stratum germinativum. The outermost layer consists of dead, keratinized cells that render the skin waterproof.

The epidermis

produces recurrent vesicular eruptions of the lips and surrounding skin.

The herpes simplex virus

A client with a cervical spine injury has chronic pain. What would be the most appropriate initial nursing intervention for this client?

The first step would be for the nurse to assess characteristics of the pain. Surgery or pharmacologic interventions would be considered by the whole health care team after more information was gathered. Option C represents a nursing diagnosis, not an intervention.

The nurse assesses the frontal sinus where?

The frontal sinuses are located above the eyes. The maxillary sinuses are located above the jaw.

What are the three layers and what order.

The outermost layer is the external abdominal oblique, the middle layer is the internal abdominal oblique, and the innermost layer is the transverse abdominis (Weber 473) (transfers is deepest)

An nurse practitioner is assessing the tympanic membrane of a client who has come to the clinic. What would the nurse practitioner expect to visualize if the client has a normal otoscopic evaluation?

The short process of the malleus

talks to you, interacts, responsive, matains posture

alert

The meibomian glands secrete

an oily substance to lubricate the eyes. Meibomian glands secrete an oily substance that lubricates the eyelid.

The apocrine glands are dormant until puberty and are concentrated in the axillae, the perineum, and the

areola of the breast The apocrine glands are associated with hair follicles in the axillae, perineum, and areola of the breast. Apocrine glands are small and non-functional until puberty at which time they are activated and secrete a milky sweat.

collapse of alveoli

atelectasis

what does having a history of UTI's mean

becoming resistant to antibiotics

Define: pain that is defined in several ways; often diagnosed when pain continues for more than 3-6 months

chronic pain

The nurse is taking the health history of a client who takes a calcium channel blocking medication for hypertension. The client reports a sensation of incomplete evacuation when having a bowel movement about three times per week. For which problem should the nurse further assess the client?

constipation

What layer of the skin is this: connective tissue (tough-resists tearing), hair follicles, sebaceous glands, sweat glands, nerves, blood vessels

dermis

Hair follicles, sebaceous glands, and sweat glands originate from the

dermis. The dermis is a well-vascularized, connective tissue layer containing collagen and elastic fibers, nerve endings, and lymph vessels. It is also the origin of sebaceous glands, sweat glands, and hair follicles.

acne, pubic/axillary hair

developmental skin variations: adolescents

cradle cap/ atopic dermatitis

developmental skin variations: infants

atrophy, decrease in melanocyte function, graying

developmental skin variations: older adults

Define: activities that require more complex, higher-level of thinking and organizational skills such as shopping, cooking, using the phone, driving, housekeeping, managing money, etc.

instrumental activities of daily living (IADLs)

process of observation, begins with initial meeting- continues through the hx and physical exam

inspection

look for alopecia (areas of baldness), texture, distribution, cleanliness, dryness or oiliness, parasites, and lesions

inspection (hair)

What inspection includes? Color (varies, look for symmetry) Odor Integrity Any Lesions

inspection of skin

where is kidney tenderness assessed from

it is assessed at costovertebral angle at the 12th rib

similar to papule, greater than 1 cm, dermis.subcutaneous flat

nodule (primary)

occlude the artery

obliterate

Name this lymph node (2)

posterior auricular (postauricular)

Plaque lesions

psoriasis, lichen simplex

2nd ICS, left of sternum- base

pulmonic landmark

Define: turbid fluid (pus) in the cavity. Circumscribed and elevated (acne, impetigo)

pustule

The subacromial bursae are contained in the

shoulder joint

Light skin and excessive sun or UV exposure are risk factors for ____ ______

skin cancer

what organs change their shape depending on their contents

small intestine gall bladder stomach colon bladder

Most absorption of nutrients occurs where in the GI?

small intestines

Name this gland

submandibular gland

what stage of lymphadema has no obvious signs or symptoms, edema not evident?

stage 0

what stage of lymphadema? swelling is present, affected area pits with pressure, elevation relieves swelling, skin is smooth (spontaneously reversible)

stage 1

what stage of lymphadema? skin tissue firmer, may look tight, shiny and feel spongy. Pitting may or not be present. Elevation does not completely alleviate swelling (spontaneously irreversible)

stage 2

what stage of lymphadema? LE has progressed to lymphostatic elephantiasis stage- limb is very large, affected area nonpitting, often permanent eczema, skin is firm and thick. Increased risk for infections/ulcerations (irreversible!)

stage 3

A pressure ulcer with these signs is at what stage: intact skin appears red but unbroken

stage I

What is borborygmus?

stomach growling

A client visits the clinic because she experienced bright hematemesis yesterday. The nurse should refer the client to a physician because this symptom is indicative of

stomach ulcers.

A 32 year old warehouse worker presents for evaluation of low back pain. He notes a sudden onset of pain after lifting a set of boxes that were heavier than usual. He also states that he has numbness and tingling in the left leg. He wants to know if he needs to be off work. what test should you perform to assess for a herniated disc?

straight leg raise test

why may stria marks be seen on some peoples abdomen

striae marks may be seen in someone who has had ascites, which will stretch the skin. Ascites usually results from liver failure of liver disease

honking wheeze (kids with croup)

stridor

what does a pulse strength of 3+ mean?

strong (obliterate with firm pressure)

client awakens to vigorous shake or painful stimuli, returns to unresponsive sleep

stupor

What layer of the skin is this: stores fat for energy, provides insulation for temperature control, and aids in protection with its cushioning effect.

subcutaneous

Complaints of pain, dizziness, lumps, swelling, head injury, or history of head and neck injury are __________ information.

subjective

Define: Information from the patient's point of view including their feelings, perceptions, and concerns (symptoms)

subjective data

Visceral pain is associated with a hollow abdominal organ such as the intestine. Visceral pain is

usually difficult to localize

Define: to put them more at ease, tell them when you are changing directions during the interview

transitions

Name this muscle

trapezius

used for acute hypotension. promotes venous circulation, postural drainage of basal lung lobes

trendelenburg

In general, a BMI <18 is considered ___________

underweight

What type of hair is this: short, fine, unpigmented

vellus

The nurse suspects carpal tunnel syndrome after examining a patient in the clinic. A test result that would suggest this diagnosis would be

weak opposition of the thumb


Related study sets

ProEducate Chapter 3 - Interests and Estates

View Set

Urinary system ch.9 (male & female)

View Set

Pre-AP Computer Science 1 Semester 1 Examination Review (ExpoJava Ch. 2-4)

View Set

C172 Lesson 5 (Quizzes, Exercises, Labs)

View Set

Article 6 of US Constitution And Commerce Clause

View Set

Operation of three phase induction motors (12hrs)

View Set